You are on page 1of 188

• i =N1.00k, ii = N5.

00k
UNILORIN POST-UTME
• i = 60k, ii = N4.40k
PAST QUESTIONS
• i = 80k, ii = N4.80k

ALL GENERAL • i = 50k, ii = N4.50k


ANS C
QUESTIONS COMBINED
• A man buys recharge card for N500. He
• A labourer is paid N5,400:00k per sells it for N430. What is the
month. What is his total salary in 1 percentage loss?
3/4years?
• 7%
• N113,40:00
• 9%
• N64,800:00
• 14%
• N97,200:00
• 20%
• N129,600:00 ANS C
ANS A
• The title of the traditional ruler of
• Which of the following will sink when Benin is called................................
placed on water?
• Obi
• Ball
• Obong
• Plastic Cup
• Oba
• Balloon
• Igwe
• Eraser ANS C
ANS D
• Which of these is not a method of
• The following are domestic birds preventing iron from rusting?
except
• Lubrication
• Duck
• Galvanization
• Pigeon
• Melting
• Turkey
• Painting
• Vulture ANS C
ANS D
• Which of the following is not a rainbow
• A boy buys a biscuit for N4 and sells it colour?
at a profit of 20%. Find the actual price
and the selling price? • Indigo

1
• Purple • Physician

• Orange • Dentist
ANS D
• Green
ANS B • A bricklayer is Paid N1000:00k for
5days work. What is his pay for 22days
• A person who flies an aircraft is work?
called..........................
• N22,000:00k
• A captain
• N50,000:00k
• A craftologist
• N4,400:00k
• A pilot
• N5,000:00k
• An aeronautics engineer
ANS C
ANS C
• In parliamentary system of
• Kwame Nkrumah Museum is located government, members of the
in.................................. parliament are required to report the
• Tanzania proceedings of the house to
their..............................
• Kenya
• Local government chairman and
• Mauritania governors

• Ghana • Constituencies
ANS D
• Political parties and party
• An example of confluence town in stalwarts
Nigeria is...................................
• Traditional rulers
• Onitsha ANS B

• Obudu • One essential duty of a citizen to


his/her state is to...........................
• Lokoja
• Vote and support the government
• Port Harcout
in power
ANS C
• Recite the national anthem and
• A trained person that specializes in the
the pledge
treatment of diseases and disorders of
the teeth is called................ • Pay his tax

• Cardiologist • Encourage other citizens to be


loyal ANS C
• Dermatologist

2
16. Under whose regime were Delta and • President Barrack Obama is
Adamawa States created? the.................. president of the United
States of America and first African-
(a)Gen Sanni Abacha American president of U.S.
(b)Gen yakubu Gowon (a)35th
(c)Gen Muritala Muhammed (b)44th
(d)Gen Ibrahim Babangida (c)40th
ANS D
(d)39th
• The body charged with the responsibility
ANS B
of regulating and protecting consumers
against illicit foods and drugs in Nigeria • A large area of water surrounded by land
is called.......................................
(a)Consumer protection council
(a)A dam
(b)Standard Organization of Nigeria
(b)An Island
(c)National Drug Law Enforcement
agency (c)An Isberg

(d)National Agency for Food, Drug (d)A lake


Administration and Control ANS D ANS B

• The senate president during the second • A written piece of paper which indicates
republic was......................... that goods bought and services rendered
have been paid for is
(a)Dr. Joseph Wayas called...............................
(b)Senator Anyim Pius Anyim • An Invoice
(c)Senator Iyorchia Ayu • A receipt
(d)Chief Adopous Wabara • A wage bill
ANS A
• A trade document
• We eat Carbohydrate food to give
ANS B
us..........................
• The device for stepping up or down
(a)Energy electricity voltage on transmission line is
(b)Good sight called......................

(c)Good teeth • Electricity meter

(d)Strong bones • Transmission station


ANS A • Transformer

3
• Step-down panel A) 354
ANS C
B) 357
• The major raw material for pottery
is............................. C) 358

(a)Metal D) 353 ANS A

(b)Ceramics 28. While preparing for the post jamb


aptitude test, Felix read through pages 35 to
(c)Cement 78 and 102 to 127 of an English text book.
How many pages did he read altogether?
(d)Clay
ANS D A) 58

• If the total sales for a bag shop in a B) 70


certain year were #150, 000 what were
sales in July, if July sales were half the C) 68
monthly average? D) 64 ANS C
A) #6,250 29. How many numbers between 1 and 60
B) #8,650 begins or end with 5

C) #12,500 A) 5

D) #7,550 ANS A B) 8

26. In UITH, Dr Steve has worked more night C) 15


shift than Dr. Greg who has worked five night D) 18 ANS C
shifts. Dr Okon has worked fifteen night shifts
more than Dr. Steve and Dr. Greg combined. 30. A researcher conducted in the laboratory
Dr. Uche has worked eight night shifts less showed that 55% rats died of liver disorder
than Dr. Steve. How many night shifts has Dr, when exposed to sulphur dioxide but 96% of
Steve worked? rats who died of liver disorder were not
exposed to sulphur dioxide. This means that,
A) Ten
A) Small portion of studied rats was
B) Nine exposed to Sulphur dioxide
C) Eight B) There is only one cause of fatal liver
D) Seven ANS B disease in rats

27. At a basket match, Roland was sitting in C) Sulphur dioxide is not particularly
seat 356. Peter on the right of Roland in seat dangerous to the liver of rats
355. Samuel was on the left of Roland. D) Most rats will not suffer from exposure
Samson was by the left of Samuel. Which seat to low level of Sulphur dioxide.
is Samson sitting? ANS A

4
31. Three people witnessed a lady who was 35. Two-third of a class passed English
robbed in a cyber café and they were asked to Language as a subject. The number of
describe the mugger. Which of the following students who failed English Language will
best described the mugger? be?

A) He was tall, of average weight and A) 1/3


middle-aged
B) 1/2
B) He was tall, thin and young
C) 1/4
C) He was average height, thin and
middle-aged D) 3/4 ANS A

D) He was tall, thin and middle-aged 36, “My Heart Jumped into ANSmyD mouth”
What is the meaning of the above statement?
32. If 15 cans of food are needed for seven
adults for two days, the number of cans A) To be confused
needed to feed four adults for seven days B) To be hungry
is?
C) To be afraid
A) 25
D) To be discouraged ANS C
B) 15
37. It was Friday on Jan 1, 2010. What was
C) 20 the day of the week Jan 1, 2011?

D) 30
A) Sunday
ANS D B) Wednesday
C) Thursday
33. Which one of the following key
D) Saturday ANS
techniques is necessary for all the others to
D
be effective?
38. Today is Monday. After 63 days, it will be:
A) Communication

B) Teamwork
A) Wednesday
C) Leadership B) Monday
C) Tuesday
D) Technical know-how ANS A D) Saturday ANS B
39. AT 10% DISCOUNT, AN ITEM VALUED AT
34. How many Lunar months make one year
N45000 NOW COST
A) 13
• 4050
B) 11
• (B) 40500
C) 12
• (C) 40505
D) 14 ANS A

5
• (D) 40550 ANS (C) DIFF
D
(D) OASIS
40. IN WHICH COUNTRY IS MISSISSIPPI ANS D
RIVER
44. WHICH OF THE FOLLOWING HAS THE
• USA HIGHEST POPULATION IN THE WORLD

• (B) ARGENTINA (A)INDIA

• (C) BRAZIL (B) RUSSIA

• (D) CANADA. ANS (C) CHINA


A
(D) USA ANS C
41. ON A ROAD MAP, A LINE SEGMENT OF
45. THE SCIENTIFIC WAY OF REFERRING
4 INCHES LONG REPRESENTS A DISTANCE OF
12 MILES. HOW MANY MILES LONG IS 4.5 TO THE ACTIVITY OF THE EAR IS
INCHES. (A)AUDIO VISUAL
• 12.5 (B) AUDIO
• (B) 13.5 (C) VISUAL
• (C) 14.5 (D) CHEMORECEPTOR ANS B
• (D) 15.5 46. THE UPPER LEGISLATIVE ARM OF
ANS B GOVERNMENT IN NIGERIA IS HEADED BY

(A)SPEAKER
42. THE BREATH AND LENTH OF A (B) PRESIDENT
RECTANGLE ARE L AND 2L RESPECTIVELY.
WHAT IS THE PERIMETER OF THE (C) LEADER
RECTANGLE?
(D) SECRETARY ANS B
(A) L

(B) 3L
47. THE DEGREE OF HOTNESS AND
(C)5L COLDNESS OF A PLACE IS CALLED

(D ) 6L (A)RAINFALL
ANS D
(B) HUMIDITY
43. THE PLACE WHERE WATER IS FOUND
(C) TEMPERATURE
IN THE DESERT IS CALLED
(D) WEATHER ANS C
(A)LAKE

(B) DAM
6
48. A PLACE WHERE TWO RIVERS MEET IS (D) AMPHIBIANS ANS B
CALLED
53. WHICH OF THE FOLLOWING IS NOT AN
(A)CONFLUENCE EXCRETORY PRODUCT?

(B) JUNCTION (A)SWEAT

(C) JOINT (B) UREA

(D) DELTA ANS A (C) URINE

49. THE PRIMARY SOURCE OF ENERGY IS (D) FAECES ANS D


CALLED
54. THE NAME OF A YOUNG LION IS CALLED
(A)SUNLIGHT
(A) KITTEN
(B) WATER
(B) KEVLIN
(C) FOOD
(C) CUB
(D) HEAT ANS A
(D) CUDE
50. THE FIRST COLOUR OF THE RAINBOW IS ANS C
CALLED
55. THE NAME OF AN EQUIPMENT FOR
(A)GREEN MEASURING PRESSURE IS CALLED

(B) VIOLET (A) BAROMETER

(C) INDIGO (B) THERMOMETER

(D) RED ANS D (C) GAUGE

51. WHICH OF THE FOLLOWING IS NOT A (D) HYGROMETER


MAMMAL ANS A

(A)WHALE 56. WHAT IS THE ANGLE IN DEGREES ON A


STRAIGHT LINE
(B) DOG
(A) 90
(C) CAT
(B) 60
(D) SHARK ANS D
(C) 180
52. AVIAN FLU IS A DISEASE IN WHICH OF THE
FOLLOWING (D) 30
ANS C
(A)MAMMALS
57. WHAT O.005KILOGRAMMES TO
(B) BIRDS GRAMMES
(C) PICES (A) 5000
7
(B) 500 • East Africa
ANS D
(C) 50
• A hexagonal kite has --- sides.
(D) 5
ANS D • 5

58. WHAT IS THE VALUE OF DCXL • 6

(A) 640 • 4

(B) 504 • 7
ANS B
(C) 540

(D) 600
ANS A The followings are true concerning a
trapezium except
59. Choose the odd one among the
followings, • It does not have any line of
symmetry
• Cougar
• All the sides are of different
• Hyena
lengths
• Cheetah • All the angles are of different
• Leopard sizes
ANS B • It has two angles equal but
• Which among the following does not others different. ANS D
belong to the group, • A man bought 100 eggs at N1500.00,
• Brigadier 10 eggs were damaged. He sold the
rest at N20.00 each. Calculate the
• Colonel percentage profit made.

• Navy Captain • 17

• Group Captain • 15
ANS A
• 20
• In which part of Africa will you find
Tanzania • 12
ANS C
• West Africa
• In a leap year a baker baked 500,000
• North Africa loaves of bread, how many loaves did
the baker bake in February, if his
• Southern Africa
working days are Mondays to
Saturdays?
8
• 1592 • 4th

• 39800 • 2nd

• 38208 • 5th
ANS A
• 46168
ANS C • The following diseases are
transmitted by mosquitoes except
• In Nigeria democracy day is marked
on • Yellow fever

• June 12 • Dengue fever

• May 29 • West Nile fever

• June 29 • Hepatitis fever


ANS D
• May 27
ANS B • In a coding system numbers 0 to 9 are
coded a to j respectively, what will be
• The -------------- originated the division
the code for the number 4238
of time as base of 60
• ebdj
• Babylonians
• fdei
• Americans
• ecdi
• Romans
• dbch
• Arabs ANS C
ANS A
• Obtain the missing number in the
• In the computer browser address following sequence: 2, 3, 5, 8, ----, 17,
field, what does the abbreviation
23, 30
www stands for
• 13
• world wide web
• 12
• word wide web
• 11
• web wide world
• 14
• wide web world ANS B
ANS A
• A retailer sells at a discount of 15% of
• The earth is the ------ planet from the the marked price, if a customer pays
sun N16,700.00 for an item, what is the
• 3rd marked price of the item?

• N 19,650.00
9
• N 13,750.00 • Muritala Muhammed

• N 20,700.00 • Ibrahim Babangida


ANS A
• N 22,700.00
ANS A • Which is the odd one out among the
followings,
• Given that G is directly proportional
to the square of H. If G is 5 when H is • African National Congress
3, Find H when G is 100
• Social Democratic Party
• 150
• National Republican Convention
• 125
• All Peoples Party
• 180 ANS A

• 225 • The last military head of state in


ANS C Nigeria was

• Late Idi Amin Dada was from which of • Sanni Abacha


the following countries
• Ibrahim Babangida
• Kenya
• Abubakar Abdulsalam
• Tanzania
• Muritala Muhammed
• Uganda ANS C

• Congo Brazzaville • GSM telephony was introduced into


ANS C Nigeria communication system during
the regime of
• River Nile has its source from
• Ibrahim Babangida

• Lake Chad • Olusegun Obasanjo

• Lake Victoria • Umaru Yar’adua

• Lake Tanganyika • Goodluck Jonathan


ANSB
• Lake Albert
ANS B • The following Nigerian footballers
have won the African Footballer
The first Nigerian who was the head of state Award except
when Nigeria first won the African Cup of
Nations, • Victor Ikpeba

• Shehu Shagari • Nwankwo Kanu

• Muhammadu Buhari • Rasheed Yekini


10
• Austin Okocha • Calabar
ANS D ANS C

• Kofi Annan is well known for his role • The time zone operative in Nigeria is
as
• GMT
• President of Ghana
• +1 GMT
• Secretary General of United
Nations • – 1 GMT

• Secretary General of the African • +2 GMT


Union ANS B

• President of the United Nations • The rotation of the earth on its axis is
ANS B used in the measurement of

Complete the followings; Ooni and Osun, • Time


Sultan and Sokoto, Obi and Anambra, El • Weather
Kanemi and --------
• Temperature
• Kaduna
• Pressure
• Borno ANS A
• Katsina • The Nigerian currency in use before
• Yobe the change to the Naira was the
ANS B • Pound
• In which part of Nigeria will you find • Dollar
relics of groundnut pyramids
• Riyal
• Kano
• CFA
• Jos ANS A
• Sokoto • An oncologist is a medical practitioner
• Maiduguri specialized in the treatment of
ANS A • Tumours
• Nnamdi Azikwe International Airport • Fibroids
is found in
• Obesities
• Enugu
• Ulcers
• Port Harcourt ANS A
• Abuja

11
• In which of the following African • Pluto
countries will you find a confluence of
two oceans • Mercury

• South Africa • Venus

• Nigeria • Moon
ANS D
• Somalia
• Which of the followings is a star?
• Senegal
ANS A • Earth

• Choose the odd one out among the • Sun


followings: • Moon
• Peru • Nebula
• Venezuela ANS B

• Bolivia • The River Niger has its source from

• Indonesia • Fouta Djallon Island


ANS D • Lake Guinea
• The capital city of Rwanda is • Upper Volta
• Khartoum • Timbuktu
ANS A
• Kabul

• Kigali • The earth is mainly covered with

• Kingston • Water
ANS C • Land
• The geographic equator passes • Equal amount of land and water
through which of the following
countries • Mountains
ANS A
• Uganda
• Car is to road as train is to
• Zambia • Wheels
• B. Rails
• Nigeria • C. Surface
• D. Locomotive
• Cameroon ANS B
ANS A
• Which is the odd one out?
• Which of the followings is not a planet • Hockey
• B. Exercise
12
• C. Tennis C. Jim
• D. Football D. Jane
ANS B ANS C

• A forest is to a tree as a tree is to a 105. The day after tomorrow is three days
before a Monday. What day is it
• Leaf
today?
• B. Orchard • Monday
• Tuesday
• C. Jungle • Wednesday
• D. Plant • Thursday
ANS A ANS C

• A rancher plans to add a post 106. Which is the next number is the
between every two posts of an open- series? 18, 13, 9, 6, 4,.
ended (straight) fence. If he currently A. 1
has 10 posts in the fence, how many
does he need to insert? B. 2
• 5
C. 3
• B. 9
• C. 10 D. 4
• D. 20 ANS C
• ANS B
107. Steam is to water as liquid is to
A. Ice
102. Complete the series English, French,
B. Solid
Hindi, Arabic
C. Vapour
A. Swahili
D. Snow
B. Language
ANS B
C. Dialect
D. Africa 108. What will be the total weight of 10
ANS A poles, each of the same weight? If:
• One-fourth of the weight of each pole
103. If you were to add all odd numbers is 5 kg.
between 1 and 11 (both inclusive),
the result would be • The total weight of three poles is 20
A. Even kilograms more than the total weight
B. Odd of two poles.
C. Impossible to say A. I alone is sufficient while II
ANS A alone is not sufficient to answer the
question
104. When Jack, James, Jim and Jane stand B. II alone is sufficient while I
by age, Jack being the youngest alone is not sufficient
stands first while James brings up the
rear. However, when they stand by C. Either I or II is sufficient
height, Jim being the shortest stands D. Neither I nor II is sufficient
first while James comes to the third E. Both I and II are sufficient
spot. In both lines Jane remains at the
second position. Who is immediately ANS E
younger than James?
109. How much was the total sale of a
A. Jack
company if:
B. James
13
• The company sold 8000 units of All dogwoods in the park are
product A each costing N 25.00 flowering trees.
• This company has no other product If the first two statements are true,
line. the third statement is

A. I alone is sufficient while II A. true


alone is not sufficient to deduce the B. false
answer C. uncertain
B. II alone is sufficient while I D. None of the above
alone is not sufficient
C. Either I or II is sufficient ANS A
D. Neither I nor II is sufficient 113. Ade runs faster than Ola.
E. Both I and II are sufficient Ayo runs faster than Ade.
Ola runs faster than Ayo.
ANS D If the first two statements are true,
the third statement is
A. true
110. Tanya is older than Eric.
Cliff is older than Tanya. B. false
Eric is older than Cliff. C. uncertain
If the first two statements are true, D. None of the above
the third statement is
A. true ANS B
B. false 114. All the tulips in Zoe's garden are white.
C. uncertain All the pansies in Zoe's garden are
yellow.
All the flowers in Zoe's garden are
either white or yellow
D. None of the above
If the first two statements are true,
the third statement is
ANS B
A. true
111. Blueberries cost more than
strawberries. B. false
Blueberries cost less than raspberries. C. uncertain
Raspberries cost more than both D. None of the above
strawberries and blueberries.
If the first two statements are true, ANS C
the third statement is
115. Fact 1: All drink mixes are beverages.
A. true Fact 2: All beverages are drinkable.
B. false Fact 3: Some beverages are red.
C. uncertain If the first three statements are facts, which
D. None of the above of the following statements must also
be a fact?
ANS A I: Some drink mixes are red.
112. All the trees in the park are flowering II: All beverages are drink mixes.
trees. III: All red drink mixes are drinkable.
Some of the trees in the park are
dogwoods. A. I only
14
B. II only
C. I and III only
118. Fact 1: Mary said, "Ann and I both
D. None of the statements is a
have cats."
known fact. ANS
Fact 2: Ann said, "I don't have a cat."
C
Fact 3: Mary always tells the truth,
but Ann sometimes lies.
116. Fact 1: All chickens are birds.
If the first three statements are facts, which
Fact 2: Some chickens are hens.
of the following statements must also
Fact 3: Female birds lay eggs.
be a fact?
If the first three statements are facts, which
I: Ann has a cat.
of the following statements must also
II: Mary has a cat.
be a fact?
III: Ann is lying.
I: All birds lay eggs.
II: Hens are birds.
A. I only
III: Some chickens are not hens.
B. II only
A. I only C. I and II only
B. II only D. All the statements are facts.
C. II and III only
ANS D
D. None of the statements is a
known fact. ANS
119. Fact 1: Pictures can tell a story.
C
Fact 2: All storybooks have pictures.
Fact 3: Some storybooks have words.
If the first three statements are facts, which
117. Fact 1: Eyeglass frames cost between
of the following statements must also
$35 and $350.
be a fact?
Fact 2: Some eyeglass frames are
made of titanium. I: Pictures can tell a story better than
Fact 3: Some eyeglass frames are words can.
made of plastic. II: The stories in storybooks are very simple.
III: Some storybooks have both words and
If the first three statements are facts, which
pictures.
of the following statements must also
be a fact?
A. I only
I: Titanium eyeglass frames cost more than
plastic frames. B. II only
II: Expensive eyeglass frames last longer C. III only
than cheap frames. D. None of the statements is a
III: Only a few eyeglass frames cost less than known fact. ANS
$35. C

A. I only 120. Fact 1: Some pens don't write.


B. II only Fact 2: All blue pens write.
C. II and III only Fact 3: Some writing utensils are
pens.
D. None of the statements is a
known fact. ANS
D
15
If the first three statements are facts, which • If it is snowing and sunny then
of the following statements must also Pedro goes hunting
be a fact? ANS B
I: Some writing utensils don't write. 123. A group of friends live in a house
II: Some writing utensils are blue. divided into one flat per floor. Tony is
III: Some blue writing utensils don't write. in the flat below Julie and Madeleine
is in the flat above Sarah. Sarah is in
A. I only the flat below Tony and Julie lives
B. I and II only with Roger. Peter lives on the top
floor. Who is in the bottom flat?
C. II and III only
D. None of the statements is a A. Tony
known fact. ANS B. Julie
B C. Madeleine
D. Sarah
121. At the baseball game, Henry was E. Peter
sitting in seat 253. Marla was sitting ANS C
to the right of Henry in seat 254. In
the seat to the left of Henry was 124. A group of friends live in a house
George. Inez was sitting to the left of divided into one flat per floor. Tony is
George. Which seat is Inez sitting in? in the flat below Julie and Madeleine
A. 251 is in the flat above Sarah. Sarah is in
the flat below Tony and Julie lives
B. 254 with Roger. Peter lives on the top
C. 255 floor. Apart from Julie and Roger, who
D. 256 else shares a flat?

ANS A A. Tony
B. Julie
C. Madeleine
122. Pedro goes hunting or fishing every D. Sarah
day. If it is snowing and windy, then E. Peter
Pedro goes hunting. If it is sunny and ANS A
not windy then Pedro goes fishing.
Sometimes it can be snowing and
sunny. Which of the following 125. Jane can play the piano but not the
statements must be true? flute while Jeremy plays the violin and
the flute. Shelly plays the violin but
• If it is not sunny and it is snowing not the piano and Josephine plays the
then Pedro goes hunting flute but not the violin. If each child
plays two of the three instruments,
• If it is windy and Pedro does not which one is likely to be similar to
go hunting then it is not snowing Jeremy?
• If it is windy and not sunny then
A. Shelly
Pedro goes hunting
B. Jane
• If it is windy and sunny then C. Josephine
Pedro goes hunting ANS B
126. Which of the following cannot be used
to travel from Portharcourt to Kano?
16
• Motor car • It was dark. The flood
covered the bridge. The
• Train lorry crashed into the
• Boat stream.

• Aeroplane • It was dark. The lorry


crashed into the stream.
ANS D The flood covered the
bridge.
127. If you were one of the students
instructed to fill a basket with water, knowing ANS C
fully well that a basket is a well perforated 130. which sequential order best describe the
object. How would you fill it?
events in the following?
• It is impossible • Tomorrow may be too
• Will use big bucket to fill it late. You may regret time
wasted. Make hay while
• Immerse it in a pool of water sun shines.

• Immerse it in a pool of water • Make hay while sun


and quickly take it out shines. You may regret
ANS C time wasted. Tomorrow
may be too late.
128. In the understanding of the actual
arrangement, the sun revolves round the • Make hay while sun
earth shines. Tomorrow may be
too late. You may regret
• Always
time wasted.
• Often
• You may regret time
• Never wasted. Make hay while
sun shines. Tomorrow
• Sometimes may be too late.

ANS C ANS C

129. Which of the following gives the best 131. The largest known planet is
logical sequence of events? …………………….?

• The lorry crashed into the • Sun


stream. The flood covered
• Uranus
the bridge. It was dark.
• Jupiter
• The lorry crashed into the
stream. It was dark. The
flood covered the bridge.

17
• Earth • vHausas

ANS C • Igbo’s

132. The largest continent on earth is • Fulani’s


……………………? • Yoruba
• Africa
ANS C
• Australia
136. Choose the two statements that
• North America would necessarily make the given
piece of information true:
• Asia
Fifty candidates sat for the
ANS D examination.

133. 50 men can build a house in 60 days. I) Twenty candidates answered under
How many more men of equal half the questions
II) There were ten failures in the
strength and ability must be put on so
examination
as to finish a similar house in 40 days? III) Over half the questions were
answered by thirty candidates
• 20 men IV) Forty candidates were successful.
• 30 men
• I & IV B. II & IV C. II &
• 35 men III D. III & IV ANS B

• 25 men 137. Third World countries are found in

A. Europe, Asia and Soviet Union


ANS D
B. Africa, Asia and Latin America
134. A group of letters that is added to the
beginning of a word in order to C. North and South America
change its meaning e.g ‘’un’’, ‘’anti’’,
is ……………………………? D. Asia

• Suffix Ans: B

• Antonym 138. A tricycle is

• Prefix • Vehicle
• Lorry
• Pronouns • Three concentric circles
• Truck
ANS C
Ans: A
135. Pastoral nomadic is a common 139. Paper is made from
characteristics of the ………………..
• wood
18
• Leaves 144. Which correspondence is wrong
• Saw dust matching?
• Pulp
A. nose-breathing-air
Ans: A
140. “He must carry his cross” mean B. mouth-laughing-sound

• She is a Christian C. eyes-seeing-heat


• He is a disciple
• He is a minister of God D. feet-motion-distance
• He suffers by destiny ANS: C

ANS: D 145. Which of the following is the correct


term for entering an aircraft?
141. There is a big hole in my pocket
means A. enter

A. My pocket is leaking B. board

B. my money comes and goes C. transit


away fast.
D tranceed
C. My pocket is torn Ans: B

D. my pocket is not in good 146. Which of the following is wrong?


shape ANS: B
A. Plane crash
142. He got the job and suddenly became
B. boat sink
swollen headed
C. Car accident
A. The job is full of hazards
D. ship wreck
B. The job made him become
Ans: B
proud.
147. The officials accompanying a VIP for
C. the job made him a big man
protection purpose are referred to as
D The job made him wealthy
A. Police
ANS: B
B. Escort
143. Which of the following is arranged in
wrong ascending order? C. Soldier

A. Feet-Body -Head D. Mobile


Ans: B
B. Middle-Top-Bottom
148. Someone who persist in doing the
C. Root-Stem-Leaves
wrong thing even after he has been
D. Sea-Land-Mountain punished is said to be
ANS: B
19
A. stubborn B. Low Antenna Net Work

B. heady C. Low Attenuated Net Work

C. recalcitrant D. logarithmic Advanced Net


Work Ans. A
D. disobedient
153. Two digit percentage inflation means
Ans: C
A. less than two decimal places
149. Random Sampling is connected more
with B. from 10-99

A. Mathematics C. more than two decimal places

D. above twenty
B. Additional Mathematics
Ans. B
C. Algebra 154. The expression 2 x 2 x 2 is equal to 8 can
be expressed as
D. Statistics.
Ans: D A. quadruple 2

150. FM in radio operation means B. triple 2

A. Far medium transmission C. cubic 2

B. Mega Frequency transmission D. thrice 2 Ans.


C
C. Modulated Frequency
155. The head of a parliamentary system of
transmission Government is
D. Modern Frequency A. The Prime Minister
transmission Ans: C
B. The President
151. Inferiority complex is used to describe
C. The Queen
A. A mathematical complex
D. The King
inferior to algorithm. Ans. A
B. A feeling that one is not as 156. A 3 -3 bedroom flat in Abuja costs 1
good or intelligent as other people. million naira per annum. The same flat in
Ilorin, the Kwara State capital will cost
C. A mathematical complex
describing an inferior integral A. Much less than 1 million naira

D. A life inferior complex B. exactly the same amount


Ans: B C. 0.5 million naira

152. The term LAN in Computer Systems D. about 700 thousand naira.
means Ans. A

A. Local Area Net Work

20
157. Seconds, minutes, hours are units with (d) 660
definite converts faction which of the Ans A
following is not
161. Write in full figures 106
A. inch, feet yard
(a) 60
B. naira, dollar, pound

C. centimeter, meter, kilometer (b) 6.0

D. centigrade, fahcuhelt, Celsius


(c) 106
Ans. B

158. The figure “O O” is a representative of (d) 1,000,000


the figures “ X X”. How many such figures
Ans D
are between the numbers 10 and 100

A. 00 • This small pest is commonly

B. 90 found on body or clothes of


people living in dirty condition:
C. 9
• lice
D. 19
Ans.C • mite
• mosquito
159. Mrs. Felicia Ojo borrowed ten (10) tiers
• cricket
of gari from her neighbour, Mrs Comfort
Ans A
Chinedu and promises to return it three fold,
• This insert is the odd one among
the following month. How many tiers of gari
these four:
will she return?
• cricket
• 3
• grasshopper
• 13
• locust
• 30
• cockwash
• 10 Ans C
Ans D
• Divide 666 cowries between 6
• This utensil is a different class
women traditional gods
• kerosene stove
worshipers. How much does
• Gas cooker
each receive?
• Electric cooker
(a)111 • Frying pan

(b) 110 Ans D


• These are local forming tools
(c) 66
except one

21
• cutlass A. Tues.25th May
• hoe B. Wed. 29 April
• shovel
C. Mon. 1st June
• spoon
Ans D D. Mon. 27 April ANS A

166. I am 10 years old my sister is 4, in how 169. Mary and Jane are both clever. Jean
many years shall I be twice as old as she will and Jane are tall. Mary and Jean are dark. My
be? friend is tall and clever. Who is she?

A. 3 A. Mary

B. 4 B. Jane

C. 2 C. Jean and Jane

D. 5 D. Mary and Jane ANS B


ANS C
170. Audu is taller than Laide but shorter
167. Bola had fewer sums right than ojo, than Deji. Deji is of the same height as
ojo had fewer sums right than Dele. Which Taiwo.Who is the shortest?
had the most right answers?
A. Audu
A. Bola
B. Laide
B. Ojo and Bola
C. Deji
C. Dele
D. Taiwo ANS B
D. Bola and Dele ANS C
171. Bisi is older than Ore, Dele is older
168. My holiday starts in six day’s time. than Bisi.Who is the youngest of all?
Yesterday was my birth day –Friday june
23rd.On which day of the week will my holiday A. None
start?
B.Bisi
th
A. Fri.june 30
C. Ore
th
B. Wed.30 June D.Dele. ANS C
th
C. Tues.29 june

D. Sat.26th july. ANS A 172. Abayo, Tope, Femi and Seyi are
friends. Only Abayo likes going to the
168. Yemi was not at school at Friday last. theatre.Only Tope and Seyi like the cinema.
She was first absent three days before that Only Femi and Seyi like dancing. Which boy
.Today is Monday 31st may.When was yemi likes neither dancing nor cinema?
first absent? Give the day and date.
A.Tope
22
B.Seyi A. Boxers

C.Abayo B. Hands

D.Femi. ANS C C. feet

173. I do not like grapes or bananas but I D. Legs ANS C


have bought some fruits to eat. Do you think
it is grapes, bananas, both or neither? Choose the correct word to complete the
analogies.
A. Grapes
177. Up is to down as before is to:
B. Bananas
A. later
C. Both
B. after
D. Neither ANS D
C. now
174. There is a boat with a ladder attached
to it and the ladder is eight metres tall if the D. next ANS B
water rises four metres how much of the Choose the correct word to complete the
ladder will be on to of the water? analogies.
A. 12 metres 178. Sew is to needle as dig is to:
B. 48 metres A. garden
C. 8 metres B. plant
D. 10 metres ANS C C. poke
175. I thought the time was ten minutes to D. spade ANS D
two ,but I was looking at the clock in a mirror.
What time was it really? Choose the correct word to complete the
analogies.
A. ten minutes to two
179. School is to pupils, as army is to:
B. ten minutes after two
A. teachers
C.ten to ten
B. chiefs
D. Ten to three. ANS B
C.soldiers

D. barracks ANS D
ANALOGIES AND ASSOCIATIONS.
Choose the correct word to complete the
Choose the correct word to complete the analogies.
analogies.
180. Feather is to bird as fin is to:
176. Gloves are to hands as shoes are to:

23
A. finish A. Breathing

B. fish B. Flying

C. crab C. Aeroplane

D. finies ANS B D. Airfield ANS C

Choose the correct word to complete the Choose the correct word to complete the
analogies. analogies.

181. Tall is to short as broad is to: 184. Poem is to a poet, as --- is to a


novelist:
A. wide
A. writer
B. long
B. author
C. straight
C. story
D. narrow ANS D
D. novel ANS D
Choose the correct word to complete the
analogies. Choose from the alternatives, the word
which is missing from the pairs
182. Orange is to peel as nut is to :
185. Merchant and selling ,artist and:
A. kernel
A. picture
B. rind
B. painting
C. shell
C. paints
D. fruit ANS A
D. brushes ANS B
Choose the correct word to complete the
analogies. Choose from the alternatives, the word
which is missing from the pairs
183. Hour is to time as kilometre is to:
186. Paper and wall, carpet and:
A. Metre
A. rug
B. Day
B.lino
C. Speed
C. floor
D. Distance ANS D
D. room ANS C
Choose the correct word to complete the
analogies. Choose from the alternatives, the word
which is missing from the pairs
183. Water and ship, air and:
187. Bees and ----Pigs and sty:
24
A. hive A. pencil

B. kernel B. paper

C. Hole C. fork

D. Sty ANS A D. writing ANS D

Choose from the alternatives, the word Choose from the alternatives, the word
which is missing from the pairs which is missing from the pairs

188. Chop is to steak as --- is to mutton 192. Trumpet is to blow as gong is to:
and :
A. bellow
A. cutlet
B. strike
B. ham
C. pluck
C. pork
D. pull ANS B
D. veal ANS A

Choose from the alternatives, the word


which is missing from the pairs SYNONYMS

Choose a word which has the same or nearly


189. Hold and hand, Kneel and:
the same meaning as the first word.
A. Ankle
193. Feeble:
B. Foot
A. Strong
C. Heel
B. weak
D. Knee ANS D
C. Tired
Choose from the alternatives, the word
which is missing from the pairs D. Little ANS B

Choose a word which has the same or nearly


190. Bed and mattress, chair and:
the same meaning as the first word.
A. Table
194. Astonish:
B. Pillow
A. asunder
C. Cushion
B. attack
D. Stool ANS C
C. surprise
Choose from the alternatives, the word
which is missing from the pairs D. Sensible ANS C

191. Knife is to cutting as pen is to:


25
Choose a word which has the same or nearly Choose a word which has the same or nearly
the same meaning as the first word. the same meaning as the first word.

195. Miserable: 199. Dear:

A. Mischief A. precious

B. Wretched B. cheap

C. Mistake C. nice

D. Wicked ANS B D. useless ANS A

Choose a word which has the same or nearly Choose a word which has the same or nearly
the same meaning as the first word. the same meaning as the first word.

196. Stubborn: 200. Elegance:

A. obstinate A. grace

B. sturdy B. display

C. Fearful C. safety

D. strong ANS A D. ugly ANS A

Choose a word which has the same or nearly Choose a word which has the same or nearly
the same meaning as the first word. the same meaning as the first word.

197. Permanent: 201. Thankfulness:

A. temporary A. greatness

B. old B. gratitude

C. lasting C. meanness

D. complete ANS D. goodness ANS B


C
Choose a word which has the same or nearly
Choose a word which has the same or nearly the same meaning as the first word.
the same meaning as the first word.
202. Study:
198. Boss:
A. Learn
A. Governor
B. Enjoy
B. Servant
C. Teach
C. Mistress
D. Remember ANS A
D. master ANS D
26
Choose a word which has the same or nearly A. Satan
the same meaning as the first word.
B. monkey
203. Prohibit:
C. gazelle
A. advertise
D. tortoise ANS B
B. allow
Complete the following simile:
C. forbid
208. He is as industrious as:
D. destroy ANS C
A. an ant
Choose a word which has the same or nearly
the same meaning as the first word. B. elephant

204. Excitement: C. worker

A. Enthusiasm D. messenger ANS A

B. Zeal Complete the following simile:

C. Fuss 209. As merciless as a:

D. Success ANS A A. grave

B. grasshopper
Complete the following simile:

205. Her gown is as green as: C. Catastrophe

A. grace D. Catapult ANS A

Complete the following simile:


B. grass

C. flag 210. As brittle as:

D. butter ANS B A. Pure water

Complete the following simile: B. Glass

C. Bintu
206. Olu is as hungry as a:
D.Wood ANS
A. orphan
B
B. wolf
Complete the following simile:
C. termites
211. As beautiful as:
D. Thug ANS B
A. rainbow
Complete the following simile:
B. brass
207. He is as mischievous as:
27
C. peacock B. Blunt

D. lamb ANS A C. Keen

Choose a word opposite in meaning to the D. Even ANS B


first word
Choose a word opposite in meaning to the
212. Above: first word

A. Over 216. Conductor:

B. Below A. Driver

C. High B. Conductress

D. Low ANS C. Passenger


B
D. Wire ANS C
Choose a word opposite in meaning to the
first word Choose a word opposite in meaning to the
first word
213. Liberty:
217. Spendthrift:
A. Freedom
A. Miser
B. Captivity
B. Loan
C. Stadium
C. Foolish
D. Guilty ANS
B D. Happy ANS A

Choose a word opposite in meaning to the Choose a word opposite in meaning to the
first word first word

214. Dissent: 218. Profane:

A. Beautiful
A. Dissent

B. Undissent B. Sacred

C. Assent C. Profuse

D.Indissent ANS C D. Stiff. ANS B

Choose a word opposite in meaning to the 219. A team of eight lumberjacks cut an
first word average of 15,000 cubic feet of timber in a
week. How many cubic feet will four
215. Sharp: lumberjacks cut in four weeks?

A. Clever A.30, 000

28
B. 25,000 B. Books

C. 32,000 C. Music

D. 16,000 ANS D. Drum ANS B


A
224. Football is to a footballer as ........to a
220. A promotional discount of 15% is offered cyclist
on a new coat which previously cost #18,000.
What is the discounted price of the coat? A. Rider

A. #15,300 B. Motorbike

B. #14,000 C. Bicycle

C. #15,500 D. Motorcycle ANS


C
D. #16,000 ANS A
225. Ade is to arrange 5S55ES in a reverse
221. Seun‘s friend was knock down by a car, orders. Which one of the following is the
he tried to copy the car plate number which correct arrangement?
reads
A. 5S55SE
HA539KST. Which one of the following is
a correct copy of the car plate number? B. SE55S5

A. HA 359KST C. ESE55S5

D. ES555S ANS
B. HA 593KST
B
C. HA 539KTS
226. She was the first woman to stand
D. HA 539KST. ANS D ..........election to parliament.

222. Danladi is to transfer some money into A. to


the account of his friend which reads
B. for
040000111906. Which one of the
following is a correct copy of the account C. through
number of his friend? A. 040000111960. D. by
B. 040000111906. ANS B

C. 04000111906. 227. They waited in a state of feverish anxiety


..........their mother to come home.
D. 04000011906. ANS B
A. till
223. Bible is to a priest as ........to a don
B. to
A. Qur’an
C. until

29
D. for ANS • bank
D • school
228. If John starts work at 8.845am and • hospital
finishes at 5.15pm. He has 90 minutes of
break. How many hours does he work in 5
ANS A
days?
232. The basic needs of man are
A.38
• Money, children and friends
B. 39 • Food, shelter and clothing

C. 35 • Education, money and happiness


• Power, authority and position
D. 40 ANS
C
ANS B

229. A restaurant bill is made up of the 233. Petroleum is to Port Harcourt as coal
following: #12.50 for starters, #28.55 for main is to
courses and #8.95 for deserts, plus a #17.50
• Enugu
service charge. How much is the bill?
• Kaduna
A.#56.50 • Kebbi
B. #57.50 • Ibadan

C. #57.00
ANS A
D. 59.50 ANS
234. Which of these is used for making
B
fabrics
230. Dapo was knock down by a car with
• Oil
plate number HA539KST. The police on duty
picked the car plate number in a reverse order • Timber
as.............. • Coal

A. HA 359KST • Cotton

B. KST 593HA
ANS D
C. HA 539KTS
235. The First Prime Minister of Nigeria
D. TSK935AH. ANS D was

231. Judge is to a Court as a teacher is to • Alhaji Nuhu Bamali


• Alhaji Tafawa Balewa
• Sir Akanu Ibaim

• class room
30
• Dr Nnamdi Azikwe • The Central Bank of Nigeria was
established in
ANS B • 1920
236. Substances that remove dirt by • 1908
absorbing the grease that fixes the dirt • 1959
are known as • 1950
• Dirt removers
• Dirt absorbers ANS C
• Grease absorbers • Headquarters of NYSC is at ________
• Cleaning agents • Abuja
• Aba
ANS C • Kano
237. Green is a • Lagos
• Primary colour
• Secondary colour ANS A
• Tertiary colour • The Nigeria Civil War started on --------
• Complimentary colour ------------
• 3rd October, 1970
ANS B • 6th July 1967
238. Blue is a • 16th July 1967
• Primary colour • 1st April 1976
• Secondary colour
• Tertiary colour ANS B
• Complimentary colour
• Taiwo Akinkunmi designed the
ANS A Nigerian Flag in
• Sir Richard’s constitution was • 1888
introduced in Nigeria in • 1977
• 1946 • 1976
• 1950 • 1958
• 1964
• 1864 ANS D

ANS A
31
• Nigeria has ---------------- geo-political • The International organization
zones responsible for taking care of the
• 5 injured is known as …………
• 6 • Red Cross Society
• 7
• 10 • UNESCO
• World Bank
ANS B • Man O’ War
• Alhaji Shehu Shagari became Nigerian
First Executive President ANS A
• 1st October 1960 • Which of the following is linked with
• 1st October 1961 CONFLUENCE
th
• 7 June 1985 • Niger State
st
• 1 October 1979 • Kogi State
• Benue State
ANS D • None of above

ANS B
• Nigeria started using N200 notes on • A patient suffering from Night
• 1st Feb, 2000 blindness requires
th
• 5 May, 2001 • Vitamin A
th
• 20 August, 2000 • Vitamin E
• 1st Nov. 2000 • Cacium only
• Iodine
ANS D
ANS A
• Queen Elizabeth visited Nigeria for • Zebra crossing is for ---------------
the second time on • Playground
rd
• 3 Dec. 2003 • Easy transport action of Zebra
• 2nd October. 1980 • Safety of Pedestrians
• 7th June 1985 • Zoological garden
th
• 14 June 1979
ANS C
ANS A
32
• The capital of Kastina State is ----------- • Chlorine
----
• Kastina ANS C
• Gusau • A crossed cheque does not allow cash
• Dutse payment
• Ilorin • Over the counter
• In savings account
ANS A • With thumb printing
• The capital of Cameroon is • None of the above
• Yauonde
• Cairo ANS A
• Lome
• Conakry • Which of the following is owned by
the Federal government
ANS A • Union Bank Nigeria Plc
• The first man to get to the space is • UBA Plc
• Yuri Gagarim • Central Bank of Nigeria
• Gay Lussac • Progress Bank Nigeria Plc
• Dalton
• Sir Isaac Newton ANS C
• Birmingham is in the
ANS A • U.S.A
• The capital of Benin Republic is --------- • U.K
----- • Ireland
• Kinshasa • Italy
• Algeria
• Niamey ANS B
• Port Novo • Which of these doctor treats pregnant
women
ANS D • Orthopedic doctor
• Goitre is caused by the deficiency of • Gynaecologist
• Iron • Neuro-Surgeon
• Calcium
• Iodine
33
• Opthamologist b) tribe
c) associates
• fellow
ANS B
• A number of bees living in the same ANS B
place 267. A place where milk is converted into
• swarm butter and cheese
• hive a) dairy
• team b) bakery
• gaggle c) distillery
d) refinery
ANS B ANS A
262. A number of people listening to a 268. A place for housing aeroplanes
concert a) tarmac
a) congregation b) garage
b) spectators c) hangar
c) audience d) airport
d )crowd ANS C
ANS 269. A place where government records
C are kept
263. A number of singers in a church a) library
a) troupe b) archive
b) choir c) government house
c) entertainers d) museum
d) artists ANS B
ANS 270. A place for storing grain
B a) depot
264. A number of sailors manning a ship b) store
a) crew c) bans
b) gang d) granary
c) staff ANS D
d) host 271. Abuja is to Nigeria as _______ is to
ANS Canada
A • Winnipeg
265. A number of directors of a company • Toronto
a) panel • Ottawa
b) jury • Montreal
c) bench
d) board ANS C
ANS
D 272. Which of the following is not a
266. A number of persons of the same primary reason why many people hold cash?
race and character • to undertake transactions
a) gang • to avoid credit
34
• to have an emergency reserve 126.A book of names and addresses
• to have a store of value • diary
ANS • dictionary
B • album
• directory
273. Which alternative is an example of an
intangible asset? ANS D
• a common stock 127.Which of the following does not
• a bond belong?
• a mutual fund share • encyclopaedia
• all of the above • catalogue
ANS • contraband
D • bibliography

122.A place where money is coined ANS C


• mint 128.A statement which is accepted as true
• factory without proof
• bank • axiom
• Central Bank • epitome
• caption
ANS A • excerpt
123.A place where fruit trees are grown ANS
• farm A
• courtyard 129.The history of the life of a person
• orchard • autograph
• backyard • autobiography
• biography
ANS C • comedy
124.An institution for the reformation of ANS
young offenders B
• prison 130.A soup made with meat, fish or
• orphanage vegetables
• hostel • baste
• reformatory • braise
• bisque
ANS D • broth
125.The art of conducting negotiations
between nations ANS D
• armistice 131.A stew of meat, especially mutton
• diplomacy and vegetables
• amnesty • haricot
• demobilise • goulash
• cutlet
ANS B
35
• simmers • Amylase (b) ptyalin (c)
Zymase (d) Resin
ANS A ANS B
132.A plane figure with ten sides and ten
angles 138.The liquid part of the blood is called --
• hexagon -----------------------
• octagon • Plasma (b) Platelets (c)
• decagon Red blood cells (d) White
• pentagon blood cells ANS A

ANS C 139.Which of the following is not a


133.Living on fish makes one disease of the blood?
• piscivorous
• Leukaemia (b) Sickle cell (c)
• carnivorous
Haemophilia (d) Anaemia
• graminivorous
ANS D
• omnivorous
140.-----------------is a part of dentition
ANS A used for cutting, biting and tearing
134.Goods found floating after a
shipwreck are called • Incisors (b) Canines (c)
• jetsam Premolars (d) Molars
• careen ANS B
• flotsam
141.ICT means……………………………
• argosy
• Information and Communication
ANS C Theology
135.A man is 30 years older than his
daughter. If the sum of their ages is 48 • Information and Communicate
years, what is the daughter’s age? Technology

• 18 years (b) 9 years (c) 8 • Information and Communication


years (d) 12 years Technology
ANS B
• Infolink and Communication
136. 8 divided by a number is 5 more than Technology
the value of 3 divided by the number, ANS C
what is the number.
142.Perspiration is to animals while -------
• 4 (b) 7 (c) 1 (d) 3 ------------------- is to plants

• Transpiration (b) Excretion (c)


ANS C
Transportation (d) Evaporation
137.Digestion of food starts from the ANS A
mouth with enzyme -----------------------in
143.Animals that feed on both plant and
the saliva.
flesh are called --------------------------------
36
• Herbivores (b) Carnivores (c) • 29.4 b) 14.7 c)
Omnivores (d) Perovores 13.9 d) 7.4
ANS C ANS B

144. Scurvy is a disease caused by lack of 151.Wale and Joke have ratio 6:4 in 80
which vitamins? units of Transcorp’s shares. How many
units of these shares belong to Wale?
• A b) B c) C d)E
• 48 b) 40 c) 32
ANS C d) 24
ANS A
145. Collection of stars is called --------------
------------- 152.The sum of and 2/3 and 1/3 is what?

• Stardom b) Galaxy c) • 1/3 b) 2/3 c) 1


Astronomy d) Meteorology d) 3
ANS B ANS C

146.The non-resident head of a University 153.A trader had 19 lanterns in her store,
is ------------------------- sold 13 and bought 6 more. How many
lanterns does she left in her store?
• Vice-chancellor b) Chancellor
c) Provost d) Registrar • 6 b) 9 c) 12
ANS B d) 4
ANS C
147.An area of land almost surrounded by
water is known as --------------------------- 154.----------------can be defined as the
movement of water molecules across a
a) Lake b) Lagoon c)
semi-permeable a) Plasmolysis b)
Island d) Peninsula ANS
Diffusion c) Osmosis d) Transpiration
D
ANS C
148.Solve 1/4 of 20/7
155.Which of the following factors could
• 3/7 b) 7/5 c) 5/7 not affect photosynthesis
d) 2/7
• Light intensity b) Water c)
ANS C
Temperature d) Pressure
149.The cost of 2 biros and 3 pencils is ANS D
N18, if a pencil cost N2.50k, how much is
156.Which of the following is not a class of
a biro?
food?
• N4.25k b) N3.20k c)
• Carbohydrates b) Water c)
N4.00 d) N5.25K
Vegetables d) Vitamins
ANS D
ANS C
150.Solve 21% of 70
157 In how many ways can a six applicant
for a job arrange themselves at around the
37
table when they are invited for an • Tomatoes b) Orange c)
interview? Pepper d) Pawpaw
ANS C
• 102 ways b) 6 ways c) 12 ways
d) 120 ways • The sum of interior angle of a triangle
ANS D is equal to -------------------

158. How many local governments do we • 900 b)1800 c)


have in Nigeria? 2700 d) 3600
ANS B
• 700 b) 720 c) 574
d) 774
ANS D • The first President of Nigeria is ---------
-----------------------
159.NCC means ---------------------------
• Obafemi Awolowo b) Olusegun
• Nitel Communication Commission Obasanjo c) Nnamdi Azikwe d)
• Nigerian Communication Centre Tafawa Balewa ANS B

• Nigeria Communication Centre • The Nigerian flag has how many


colour(s)
• Nigerian Communication
Commission • 3 b) 1
ANS D c) 4 d) 2 ANS
D
160.Which of the following is referred to
as the power house of the cells • The Nigeria flag was designed by ------
------------------
• Nucleus b) Mitochondria c)
Cytoplasm d) Ribosome • Akinkunmi Alabi b) Kola
ANS B Olawuyi c) Taiwo Akinkunmi
d) Tai Solarin ANS C
161.Starch could be found in all of these
except -------------------------- • Gavel is an instrument used by ---------
----------
• Yam b) Rice c) Wheat
d) Palm oil • Welder b) Surgeon
ANS D c) Judge d) Bricklayer
ANS C
162.The organism that cause MALARIA is -
---------------------- 64) Which of these diseases is Insect
borne?
• Mosquitoes b) Plasmodium c) Tse
Tse Fly d) Butterfly a) Sleeping sickness b) Whooping
ANS B cough c) Measles d)
Chicken pox ANS: A
163.Which of these is NOT a fruit?

38
65) The digestive tract is also referred to 74) Who presides over the House of
as………….. Canal. Representative in Nigeria?

a) Abdominal b) Alimentary a) Chief Whip b) President


c) Lymphatic d)Intestinal c) Presiding Officer d)
ANS B Speaker ANS D

66) The period after childhood and before 75) What prefix can be added to the word
adulthood is referred to as……………… “manage” from the list below?

a) Infancy b) Adolescence a) Non b) Un


c) Puberty d) Maturity c) Mis d) Dis
ANS: B
ANS C
68) Which of the following belongs to the
electronic media? 76) Animals store carbohydrate mainly as

a) Novel b) Road sign a) Fats b) Glycogen


c) Magazine d) Television c) Minerals d)
ANS: D Starch ANS B

69) On the Nigeria’s Coat of Arm, the black 77) A bicycle factory produces 600
shield stands for
bicycles in 5 working days. How
a) Fertile soil b) Strength
many bicycles will it produce in 8
c) Honour d) Dignity
ANS A working days?
70) Which of the following is NOT a factor a) 120 b) 640
of production?
c) 480 d)
a) Labour b) Land
c) Capital d) Factory 960 ANS
ANS: D
D
72) The amount of blood pumped per
minute in the body is called --------------------- 78) Which of the following gases is

a) Pulse rate b) Cardiac arrest required for rusting?


c) Cardiac output d) Stroke
volume ANS A a) Helium b) Hydrogen

73) Which of the following organization c) Oxygen


performs FIRST AID activities?
d) Argon
a) WHO b) UNESCO
c) NAFDAC d) REDCROSS ANS C
ANS: D

39
80) Reflected sound is called----------------- a) Hair b) Ear

------- c) Eye

a) Echo b) Noise d) Skin

c) Wave d) ANS A

Mirage ANS 90) The weather is cooler at Jos

A compare with Lagos because Jos-----

81) A mixture of sand and salt can be -------

separated based on the difference in a) Has less sunshine

their…… b) Has less rainfall

a) Magnetism b) Colour c) Has more rocks

c) Solubility d) d) Is on a higher ground

Size ANS ANS

C D

82) Which of the following trait is NOT 91) All these parts are involved in

genetically transmitted? breathing EXCPECT

a) Eye Colour b) Hair Style a) Diaphragm b) Mouth c)

c) Fatness d) Nose d) Ribs

Complexion ANS B ANS B

83) Steel is an alloy of ---------------------- 92) Meteorologists are scientist who

a) Iron and Carbon b) Iron and specialise in the study of ---------------

Silicon c) Calcium and Silicon d) ----

Iron and Tin ANS A a) Sun b) Hydrosphere c)

87) Which of the following is NOT a Atmosphere d) Weather

sense organ? ANS D

40
95) The ability of living things to respond • 1992 (b) 1996 (c)
1998 (d) 1991
to stimuli is described as----------------
ANS: D
--
• The highest court in Nigeria is the -
a) Locomotion b) Respiration ------------------

c) Growth d) (a) Federal High Court (b)


Supreme Court (c) Federal Court
Irritability ANS (d) Sharia Court
ANS:
D B
3. The rotation of the earth on its
96) Which of these forest products is axis causes?

used for the manufacture of paper? (a) Autumn and Winter (b)
Wet and dry seasons (c) Summer and
a) Wood Pulp b) Gum c)
water (d) Dawn and Twilight
Flower d) Fruit ANS:
D
ANS A
4. The earth is the --------?
98) Every participant -----------------
(a) Closest planet to the sun (b)
reported for the opening ceremony 2nd closest to the sun (c) 3rd closest to the
sun
a) Have b) Was (d) 4th closest to the sun
ANS:
c) Had C

d) Has 5. Naira is to Nigeria as ---------is to India


(a) Yen (b) France (c)
ANS D
Rupee (d) Dupee ANS: C
100) A die is thrown once, what is the
6. Collection of different human
probability that the number that organs working as a unit is referred to as

turns up is a multiple of two?
(a) Tissue (b) Cell (c)
a) 5/6 b) 2/3 c) 1/6 System (d) Skeleton ANS: C
d) 1/2 ANS D
7. A body of shallow sea water or
• Abuja officially replaced Lagos as brackish water separated from the
capital of Nigeria in -------------- sea by some form of bamer is
called -------

41
(a) Nile’s Valley (b) Lagoon (a) Kind (b) Modest (c)
(c) Greeks (d) Oasis ANS: Proud (d) Pompous
B ANS: B

8. HIV can be transmitted in the of 14. It is right to say, switch-------or put-


these ways except ------the light subject to the type of light.

(a) Homosexual intercourse (b) (a) of/off (b) off/off (c)


Heterosexual intercourse (c) out/off (d) off/out
Mosquitoes as a
ANS: D
Vector (d) Needle pick injury
with infected blood ANS:
15. A betrayal from someone
C
supposed to be a friend is a stab--------the
9. Her younger sister likes back
conservative hair style but her
(a) On (b) In (c)
cousin prefers---------hairstyle
At (d) From
(a) Modest (b) New (c) ANS: B
Modern (d) Magnanimous ANS:
16. The handset won’t work, the
C
battery has run--------
10. She adores eating junks but------ (a) Off (b)
eating fruits Out (c) Down (d) Over
ANS: C
(a) Loves (b) Supports (c)
Dislikes (d) Disliked 17. The thief was caught. He couldn’t
ANS: C get------with what he stole

11. The brevity of the first message (a) Away (b) Scot free (c)
contracts with the---------of the Free (d) Over
second message ANS: A

(a) Shortness (b) Integrity (c) 18. The University project---------owing


Verbosity (d) Moral to lack of funds
ANS: C
(a) Fell down (b) Fell over
12. Humility is a virtue while-------can (c) Fell off (d) Fell
be regarded as a vice through ANS: D

(a) Pride (b) Proud (c) 19. An iron rod feels cooler in
Curiosity (d) Position the hand than a wooden
ANS: A rod at the same
temperature. The reason is
13. Femi is very Presumptous whereas
that iron--------
his friend is --------
42
(a) is a better conductor of heat d. holding and is ANS:B
than wood
(b) has a higher density than
wood
( c) has a higher expansivity than 10. Political party stalwart is the same
wood as
(d) is heavier than wood of the
same size a. political supporters
ANS: A
b. political opponents
20. Which of the following is used
for controlling the amount of light entering c. political enemies
the eye?
d. politicians ANS:C
• Cornea
• Pupil 1.
• Iris Bisi is as tall as Uche. Adamu and
• Cilary muscles Hafeez are the same height. There is
ANS:B no difference in height between Uche
and Adamu. Therefore:
1. Calculate the area of a square floor • Bisi is taller than Adamu.
with dimension 13m by 13m in cm. • Uche is shorter than Hafeez.
• Bisi is as tall as Hafeez.
a. 169cm • Hafeez is shorter than Bisi.
ANS:
b. 169sq.cm C

c.16900sqcm 2.
If your daily newspaper costs N500
d.16900cm ANS:C during the weekend and N300 on other
days, how much do you spend weekly
8. Cheerfulness is the opposite of-------
on newspapers?
a. hostility • N2,500.00
• N3,000.00
b. war • N3,500.00
• N4,000.00
c. fighting ANS:
A
d. kindness ANS:A
3.
9. The news papers reported that the A printing machine produces at 30
meeting -------yesterday between the labour pages per minute. How many pages
leaders can the machine print in one hour?
and federal government -------- in a dead luck • 30
• 180
a. hold and was • 300
• 1800
b. held and was ANS:
D
c. came up and was

43
4. Reorganise the following sentences
How many days are there in a leap into the order in which they were
year? originally written-
• 364 days P: There are thought to be billions of
• 365 days them. Q: Perhaps the most common
• 366 days objects in the universe are stars. R: It
• 367 days includes everything visible, invisible,
ANS: known and not yet known. S: The
C universe is made up of everything that
exists.
5. • PQRS
If forty people donated N6 a month to • QSRP
a cooperative society for one year, • SRQP
how much would be collected? • QRPS
• N288 ANS:
• N2880 C
• N28,000
• None of these 9.
ANS: How many state Governors are there
B in Nigeria in the year 2011?
• 36
6. • 37
How many weeks make a leap year? • 38
• 50 • 39
• 51 ANS:
• 52 A
• 53
ANS: 10.
C 6 + 9 = Z. Therefore
• Z = 15
7. • Z = 16
Reorganise the following sentences • Z = 17
into the order in which they were • Z = 18
originally written- ANS:
P: This is pollution. Q: It can be small- A
scale or global and most plants and
animals suffer its effects. R: harmful by 11.
–products of industry and agriculture 25% of Z = 37.5 Therefore
enter the environment. • Z = 1.5
• RPQ • Z = 9.375
• PRQ • Z = 150
• PQR • Z = 937.5
• QRP ANS:
ANS: C
A
12.
8. Ten buses are to carry 20 passengers
each. Each passenger is travelling with
a bag weighing 15kg. What is the
44
combined weight of luggage to be • SUNDAY
moved by all the buses? • MONDAY
• 300kg • TUESDAY
• 600kg • WEDNESDAY
• 3000kg ANS:
• 6000kg A
ANS:
C 17.
Tola. Vincent, Adamu, Idrisu and
13. Emeka took part in a shot put
Which is the odd number out? 462 competition. Idrisu took more shots
683 385 198 than Adamu, Vincent took more than
• 198 Idrisu, Adamu took more than Tola,
• 385 and Emeka took fewer than Vincent.
• 462 No two competitors took the same
• 683 number of shots. Which one of the
ANS: following conclusions is, therefore
D proved to be correct?
• Idrisu took more shots than Tola
14. but fewer than Adamu.
ABCDEFGH • Idrisu took fewer shots than
What letter is two letters to the left of Tola and Vincent.
the letter immediately to the right of the • Idrisu took more shots than Tola
letter three letters to the right of the and Tola took fewer shots than
letter A? Adamu.
• B • Emeka took more shots than
• C Adamu. ANS:
• D C
• E
ANS: 18.
B If you have four-fifths of N100 and
spend N36.00, how much will you be
15. left with?
Only one group of five letters below • N9
can be re-arranged to spell out a five- • N44
letter word in the English language. • N64
Identify the word. • N424
• ANOIP ANS:
• TNIEC B
• HEOLC
• LEVUR 19.
ANS: If Friday is the fourth day of the month,
A what day is the 13th day of the month?
• WEDNESDAY
16. • THURSDAY
SUNDAY, MONDAY, WEDNESDAY, • SATURDAY
SATURDAY, WEDNESDAY, • SUNDAY
MONDAY, SUNDAY, ... What day is ANS:
next? D

45
• PQRS
20. • QRSP
If four people all said “hello” to each • PSRQ
other once, how many times would the • QPSR
word “hello” be spoken? ANS:
• 11 times D
• 12 times
• 13 times 23.
• 14 times Reorganise the following sentences
ANS: into the order in which they were
B originally written-
P: The other was left standing by alone
21. until they were ready for him. Q: I saw
Reorganise the following sentences a man strike one of them with a
into the order in which they were wooden axe. R: As I looked, I saw two
originally written- fellows pulled from the boat: they were
X: They give special practice at being brought to be killed.
improving your ability to calculate. Y: • QRP
Where you see QR beside an exercise • RQP
or a question, this stands for • QPR
Quantitative Reasoning. Z: You should • PQR
do and discuss these questions with ANS:
your teacher and classmates. B
• XYZ
• XZY 24.
• YXZ If the word PINT is written under the
• YZX word SAFE, the word THUD is written
ANS: under the word PINT and the word
D HOPE is written above the word
SAFE, what word can be read
22. diagonally?
Reorganise the following sentences • TEND
into the order in which they were • FAST
originally written- • HAND
P: ICT is generally taken to mean • SPIN
technologies that support ANS:
communication via computers. Q: C
Presently, the world is experiencing an
ever-increasing use of information and 25.
communication technology (ICT) in What is the day and date 30 days after
many areas of human endeavour. R: Wednesday 12 May?
Thus, the advent of ICT has brought • Thursday 29 May
about effective and efficient • Thursday 10 June
information generation, utilization and • Friday 30 May
dissemination, as well as storage and • Friday 11 June
retrieval. S: Such technologies include ANS:
the Internet, local area networking, D
electronic mail, and the world-wide-
web. 26.
46
In the series: AB, ABD, ABDG, 30.
ABDGK, ... What comes next? If the word SOUR is written under the
• ABDGKO word EVEN, the word PAST is written
• ABDGKP under the word SOUR and the word
• ABDGKQ CALM is written above the word
• ABDGKR EVEN, what word can be read
ANS: diagonally?
B • REAL
• LAST
27. • POEM
ZK9PXL428 rearranged as follows – • CORE
letters first in alphabetical order, ANS:
followed by the numbers in ascending C
numerical order, gives:
• KZ9LPX248
• 2489KLPXZ 31.
• KLPXZ2489 Consider the following statements.
• 248LPXKZ9 Statement One: 2 kg of bread is lighter
ANS: than 2 kg of stones.
C Statement Two: 5kg of cement is not
heavier than 5kg of sand.
Which of the options are correct?
28. • Statement One is true.
If the woord HAVE is written under the • Statement Two is true.
word FARM, the word COST is written • Statement One is false and
above the word FARM and the word Statement Two is false.
GATE is written under the word HAVE, • Statement One is true and
what word cannot be read diagonally? Statement Two is true. ANS:
• HAS B
• ORE
• RAG 32.
• CAVE What is the day and date 7 days after
ANS: Monday 28 August?
C • Sunday 3 September
• Monday 3 September
29. • Tuesday 3 September
If the word DONE is written under the • Wednesday 3 September
word BALL, the word COLD is written ANS:
above the word BALL and the word B
SING is written above the word COLD,
how many words can be read 33.
diagonally? If Thursday is the fourth day of the
• 2 month, what day is the 18th day of the
• 3 month?
• 4 • Thursday
• 5 • Friday
ANS: • Saturday
D

47
• Sunday
ANS: 38.
A Rearrange 2F39EAUYMK596 as
follows – first, the consonants in
34. alphabetical order, followed by the
What is the day and date 15 days after numbers in descending numerical
Friday 19 September? order, then the vowels in alphabetical
• Tuesday 29 September order.
• Wednesday 30 September • AEFKMUY996532
• Friday 2 October • FKMY996532AEU
• Saturday 4 October • FKMY96532AEU
ANS: • AEFKMUY96532
D ANS:
B
35.
In the sequence MONDAY, 39.
WEDNESDAY, SATURDAY, ABCDEFGH
MONDAY, THURSDAY, SATURDAY, What letter is three letters to the left of
... What day comes next? the letter immediately to the right of the
• SUNDAY letter four letters to the right of the
• MONDAY letter C?
• TUESDAY • B
• WEDNESDAY • C
ANS: • D
C • E
ANS:
36. In the sequence MONDAY, D
WEDNESDAY, SATURDAY,
THURSDAY, SATURDAY, ... What 40.
day comes next? ABCDEFGH
• TUESDAY What letter is four letters to the left of
• WEDNESDAY the letter immediately to the left of the
• THURSDAY letter three letters to the right of the
• SATURDAY letter E?
ANS: • B
A • C
• D
37. • E
Rearrange 582QUETF67as follows – ANS:
first, the consonants in alphabetical B
order, followed by the numbers in
descending numerical order, then the 41.
vowels in alphabetical order. January, March, June, October; what
• FQT87652EU comes next?
• QTFEU85276 • November
• FQTEU87652 • December
• 85276EUFQT • February
ANS:
A

48
• March • 95
ANS: ANS:
D C

42.
A, C, F, J, O, ... what letter comes
next? 47.
• S My watch was correct at noon, after
• T which it started to lose 15 minutes per
• U hour. It now shows 2.15 pm. What time
• V is it now?
ANS: • 2.30 pm
C • 2.45 pm
• 3.00 pm
43. • 3.15 pm
Which set of letters is the odd one out? ANS:
• IJKNO C
• LMNPQ
• EFGIJ 48.
• OPQST My watch is now showing the time as
ANS: 11.00 pm. What time was it 90 minutes
A ago?
• 9.00 pm
44. • 9.30 pm
Which set of numbers is the odd one • 10.00 pm
out? • 10.30 pm
• 9854 ANS:
• 7621 B
• 6521
• 8743 49.
ANS: How much time can be spent on each
B question if there are 60 questions to be
completed in one and a half hours?
45. • 2 minutes
Which set of letters is the odd one out? • 3 minutes
• ACEGI • 5 minutes
• EGIKM • 10 minutes
• UVWYZ ANS:
• OQSUW B
ANS:
C 50.
Two friends went to a neighbourhood
46. shop with N500. They bought a loaf of
The combined age of me and my two bread at N180 and two tins of milk for
children is 75. What will it be in five N80 each. How much did they have
years time? left?
• 80 • N200
• 85 • N220
• 90 • N240
49
• N260 • Hole
ANS: • Window
D • Door
• Roof
• You live in a bungalow made entirely ANS:
of red wood. What colour would the B
stairs be?
• If you drop a yellow hat in the Red
• Red Sea, what does it become?
• Green
• Yellow • Red
• Not applicable • Green
ANS: • Blue
D • Wet
• What word can be written forward,
backward or upside down and can still ANS: D
be read from left to right. • A turtle a a/an
• Amphibian
• Noon • Fish
• None • Mammal
• Moon • Reptile
• Boom ANS: D
ANS:
A • A heptagon is a ---------------------sided
• Samuel was out for a walk when it shape
started to rain. He did not have an • 6
umbrella and he was not wearing a • 7
hat. His clothes were soaked, yet not • 8
a single hair on his head got wet. • 9
How could this happen? ANS: B
• He used his hands to cover his
head • Fred bought a book that was
• He used his clothes to cover his discounted 75% off the list price of
head N20. How much did he pay for the
• He was bald book?.
• He braided his hair. • N15
ANS: • N10
C • N7.50
• What question can you never answer? • N5
• What is it like to be married? ANS: D
• What is it like to be hungry
• What is it like to be dead? • The windows of a building have 6
• What is it like to be happy? sides and 6 angles. What type of a
ANS: polygon are the windows?
C • Septagon
• ----------is an ancient invention still • Octagon
used in some parts of the world today • Hexagon
that allows people to see through • Pentagon
walls. ANS: C

50
• Of the 72 students on a track team, • Customers will likely become
34 are boys, what fractional past of interested in the product
the team are girls? • Customers will become more
knowledgeable about the product
• • Customers will be satisfied with
the price of the product
• • Customers will pay attention to
how the product is marketed
• ANS: A

• ANS: A
23. Express 398753 correct to three significant
• What is the reciprocal of figures

• A. 398000

• B. 398700

• C. 398800

D. 399000
• ANS: B Ans=D

• A kilometer is made up of ------ 24. A boy estimated his transport fare for a
meters. journey as N 190 instead of N200. Find the
• 100 meter
percentage error in his estimate
• 1000 meters
• 10 meters A. 5%
• 10,000 meters
ANS: B B. 47.5%

• Write 18% as a decimal C. 5.26%


• 0.18
• 0.018 D. 95%
• 0.0018 Ans=A
• 1.8
ANS: A 25. In a bag of oranges, the ratio of good ones
to bad ones is 5:4. If the number of bad
• A company has increased its monthly
oranges in the bag is 36, how many
profit from N12,500 to N17,800. By
oranges are there altogether?
what percentage did their profit
increase?.
A. 72
• 29.3%
• 36.7% B. 81
• 42.4%
• 53.9% C. 54
ANS: C
D. 45
• What is meant when it is stated that Ans=B
“a good marketing strategy will likely
draw customers in?”
51
26. A man is four times as old as his son. The 30. The way to stop frivolous publications is
difference between their ages is 36. Find the to-----the press
sum of their ages
A. bribe
A. 45years
B. gag
B. 48years
C. jail
C. 60years
D. shackle
D. 74years Ans=B
Ans=C
31. The instrument used to measure air
27. A student spends ¼ of his pocket money pressure is called
on books and 1/3 on a shirt. What fraction
remains? A. Rain gauge

A. 5/6 B. Thermometer

B.7/12 C. Wind vane

D. Hygrometer
C. 1/6
Ans=D
D. 5/12
Ans=D 32. The relationship traced to one’s father
side is called
28. The ------ in today’s issue of The Punch
News paper focused on inflation A. matrilineal

A. title B. patrilineal

B. editorial C. lineage

C. headline D.cousin
Ans=B
D. story
33. T he most wide spread traditional
Ans=C
occupation in Nigeria is
29. President Jonathan’s speech------ at 9pm
yesterday A. farming

A. had being broadcast B. blacksmithing

B. had been broadcast C. weaving

C. was broadcasted D.driving


Ans=A
D. was broadcast
Ans=D 34.The scientific explanation of man’s origin is
referred to as----theory

52
A. technological C. rain

B. ancient D. sunshine
Ans=D
C. experimental

D. evolution
Ans=D 39.The longest river in Nigeria is river

35.Which of these is NOT a symbol of the A. Benue


nation
B.Hadejia
A. coat of arms
C. Niger
B.the pledge
D. Osun Ans=C
C. Aso Rock

D.National flag
Ans=C

36.The money, equipment and building which 40. Nigeria is bounded in the south by
are being channelled for production of goods A.Togo
is known as
B. Atlantic ocean
A. capital
C. Pacific ocean
B. labour
D. Cameroon
C. land Ans=B
D. management
Ans=A
41. Which of these is NOT an Island in Africa?
37. Voyage refers to travelling by
A. Fiji
A. air
B. Madagascar
B. water
C. Cape Verde
C. rail
D.Zanzibar Ans=A
D. road
Ans=B 42. In which state is Lake Chad situated?

38. Which of the following is NOT an agent of A. Adamawa


erosion?
B. Borno
A. ice
C. Taraba
B. wind
D. Sokoto Ans=B
53
43. The black race is called (b) 19 and 17

A. Caucasian (c) 36

B. Red Indian (d) 36 each


ANS: A
C. Mongolian
2. As book is to_____________so also is
D. Negroid Ans=D
teaching is to________________
44. The following factors affect population
(a) School teach
EXCEPT
(b)Reading, lecturing
A. mortality rate
(c)Student, teacher
B. immigration

C. census
(d) Reading, teacher
ANS: D
D. fertility rate Ans=C
5. Subtract ninety nine from nine hundred
45. Which is NOT a root crop? and ninety and then, add it to eight
hundred and sixty five. What will be the
A. sorghum
final answer?
B. yam
(a) 1756
C. potato
(b) 1855
D. cassava Ans=A
(c) 1954

(d) 1180
ANS: A

50. Which of these United Nations Agencies is 6. Temporary teeth are replaced by
responsible for the promotion of culture? permanent teeth

A. ILO (a) anytime a tooth removes

B. WHO (b) twice only

C.UNESCO (c) after every other year


D. UNICEF Ans=C (d) only once
ANS: C
1. Ojo and Aina bought 25 oranges each
and gave Audu and Amina 6 and 8 oranges
each from both. How many oranges will
both of them have left?

(a) 18 each 7. A grandfather is ones__________


54
(a) father’s father only (a) 38

(b) mother’s father (b) 73

(c) both mother and fathers father (c) 56

(d) All fathers and mothers above 50years (d) 50


of age. ANS: C ANS: A

17. Suleiman is taller than Ajayi while


Usman is taller than Suleiman and Tope
who is taller than Suleiman . Who is the
shortest? Three children were playing counting their
(a) Usman fingers and toes. Talatu added her fingers
together 5times, while Binta added one-
(b) Suleiman foot toes 7times with three toes of the
other foot. Funmi then added both her toes
(c) Ajayi and fingers of the limb together.
(d) Tope 20. Funmi’s result is
ANS: B
(a) 20
Three children were playing counting their
fingers and toes. Talatu added her fingers (b) 10
together 5times, while Binta added one-
foot toes 7times with three toes of the (c) 40
other foot. Funmi then added both her toes (d) 30
and fingers of the limb together. ANS: A
18. What is the total of Talätu’s addition? Three children were playing counting their
(a) 60 fingers and toes. Talatu added her fingers
together 5times, while Binta added one-
(b) 50 foot toes 7times with three toes of the
other foot. Funmi then added both her toes
(c) 45 and fingers of the limb together.
(d) 25 21. If both Funmi and Talatu’s results are
ANS:B added together what will be the total?
Three children were playing counting their (a) 80
fingers and toes. Talatu added her fingers
together 5times, while Binta added one- (b) 58
foot toes 7times with three toes of the
other foot. Funmi then added both her toes (c) 70
and fingers of the limb together. (d) 45
19. Binta answer will be. ANS: C
55
25. The 2011 12th National Sport Festival 45. In a conference, one hundred people
took place at were seated at the opening ceremony.
Later, five hundred chairs were added.
(a)Uyo Three hundred and ninety nine people
(b) Asaba came to join others. How many chairs
were left unoccupied?
(c) Enugu
(a) one hundred and one
(d) Portharcourt
ANS: D (b) two hundred and one

26. The following are former Presidents of (c) ninety nine


Federal Republic of Nigeria except (d) one hundred and fifty
(a)Tafawa Balewa ANS: A

(b) Nnamdi Azikwe 48. All these are capitals of states except

(c) Shehu Shagari (a) Umuahia

(d) Olusegun Obasanjo (b) Asaba


ANS: A
(c) Birnin Kebbi
33.An early morning jogger jogged to (d) Okenne
cover a distance of 1.5 kilometres and ANS: D
followed the same route back to where he
started. The distance covered will be? 49. These are tools except

(a) 1.5km (a) tag

(b) 2.25km (b) spanner

(c) 3.0km (c) screwdriver

(d)3.10km (d) hammer


ANS: C ANS: A

34. All these are fruit except____

(a)Beans

(b) Banana

(c) Cashew

(d) Garden egg


ANS: A
• What name do we give to the shape
of a milk tin?

56
• A sphere B. A cylinder C. A 7. Which is the smallest ocean in the
cone D. A pyramid world?
ANS: B
A. Arctic Ocean B. Indian Ocean
• If we represent the word GRATEFUL C. Atlantic Ocean D. Pacific
with 97413586, what word is Ocean ANS: A
represented by 1347?
8. A Sport Utility Vehicle (SUV) runs at 100
• TEAR B. TALE C. km/hr top speed. It can carry a maximum of 8
GATE D. LATE persons, including the driver. If the speed of
ANS: A the SUV decreases in fixed proportion with
increase in number of persons, find the speed
• If we represent the word GRATEFUL when three persons are traveling in the Sport
with 97413586, what number is Utility Vehicle.
represented by FUEL?
A. 40 km/hr B. 120 km/hr
• 5831 B. 7836 C. C. 80 km/hr D. 100 km/hr
5356 D. 5836
ANS: D
ANS: D
9. If some green are blue and no blue is white,
• What number would fill the gap in the which of the following is correct?
set of numbers below:
A. No white is green
3 5 7 B. No green is white
6 10 14 C. No white is blue
12 20 ---- D. Some green are white
ANS: D
A. 30 B. 42 C. 28
D. 24 11. One cow tells the other that there are
ANS: C two cows in front of me. The other one also
shouts that he, too, has two behind him.
6. Which of the following statements should How many are they?
come first, second, and third in the normal
order of occurrence? A. 4 B. 3 C. 2
D. 5
1. I went to bed early yesterday ANS: B
evening.
12. Ojo, Emeka and Abdul have Toyota
2. I was very tired. Siennas. Abdul also has a Honda Civic. Otuka
has a Mercedes and a Jaguar. Ojo also has a
3. I worked hard throughout
Peugeot 206. Who has the fewest cars?
yesterday
A. Emeka B. Ojo C.
A. 1, 2, 3 B. 2, 1, 3 C. 3,
Abdul D. Otuka ANS:
2, 1 D. 3, 1, 2 ANS:
A
C

57
13. Eighty-Four is twelve times a particular 19. A horse rider rode into town on Friday,
number. What is eleven times that number? spent one night there, and left on Friday.
Which of the following explains the situation?
A. 7 B. 84 C.
132 D. 77 A. His horse was named Friday B. He arrived in
ANS: D the town after midnight on Friday C. He left
the town before noon on Friday D. His own
14. A window cleaner is cleaning the windows name was Friday
on the 25th floor of a skyscraper, when she
ANS: A
slips and falls. She is not wearing a safety
harness and nothing slows her fall yet she 21. What name do we give to the fear of
suffered no injuries. Which of the following death?
best explains the incident?
A. Emetophobia B. Necrophobia C.
A. She was inside a giant fork lift. B. She was Brontophobia D. Aerophobia
cleaning from an outside scaffold ANS: B

C. She used a long ladder D. She was 22. Which side of a cat contains the most
cleaning the inside of the windows hair?
ANS: D
A. The left–hand side B. The outside C.
16. Bayo is double the age of Ibok and one The right-hand side D. The inside
third as old as Sanusi, who will be 48 years old ANS: B
in 6 years. How old is Ibok?
23. My successor’s father is my father’s son,
A. 6 years old B. 12 years old C. 7 years old and I do not have any brothers or sons. Who
D. 14 years old is my successor?
ANS: C
A. Myself B. Daughter C. Niece
17. A cattle rearer has 15 cows, all but 8 die. D. Nephew
How many does he have left? ANS: B

A. Seven B. Zero C. 24. What can you hold in your right hand, but
Eight D. Fifteen ANS: not in your left?
C
A. Left hand, left elbow, and left
18. Fausat, Bolanle, Amaka, and Osese are forearm
polyglots. Fausat and Bolanle speak French,
B. Right hand, right elbow, and left
whereas the others speak Russian. Bolanle
forearm
and Osese speak Italian. Everyone except
Fausat speak German. Who of them only C. Left leg, left knee, and left foreleg
speaks Russian and German?
D. Right leg, right knee, and right
A. Fausat B. Bolanle C. Amaka foreleg ANS:
D. Osese ANS: A
C

58
25. How many times do the hands of a clock carbohydrates (b) adequate amounts of
overlap in 24 hours? vitamin and minerals (c) all the nutrients in
the correct proportion. (d)
A. 22 B. 24 C. 20 Proteins, Vitamins and Carbohydrate.
D. 48
ANS:
ANS: B C
26. Fausat, Bolanle, Amaka, and Osese are
30. The democratic way of selecting a leader
polyglots. Fausat and Bolanle speak French,
is through
whereas the others speak Russian. Bolanle
and Osese speak Italian. Everyone except (a) Birth (b) election (c) force (d)
Fausat speak German. Who of them speaks Imposition
more than one language but not Russian? ANS: B

A. Fausat B. Bolanle C. 33. Discrimination of people based on the


Amaka D. Osese ANS: colour of the skin is called
B
(a) antisocialism (b) race (c) separation
29. How many squares altogether are there (d) racism ANS:
on a normal chessboard? D

A. 204 B. 144 C. 264 53. Audu had no ................... he is an orphan


D. 196
ANS: A (a) brothers (b) friend (c) parents
(d) relations ANS:
30. David, Rufai and Okpoti need to be able to C
run 100m in under 12.5 seconds to qualify for
a regional championship. Rufai and Okpoti run 54. An accused person is presumed
raster than David. David’s best time for the ................. until proved guilty
100m is 13.2 seconds. Which of the following
(a) acquitted (b) bailed (c)
options is true?
persecuted (d) innocent
A. Only Rufai qualifies B. ANS: D
David does not qualify
1. The first female to drive a car in Nigeria is
C. Rufai and Okpoti both qualify D. • Mrs. Olufunmilayo Ransome Kuti
(b) Mrs. Bolanle Awe
David qualifies ANS: (c) Mrs. Mary Herbert
B (d) None of the above

21. The radiator in a car engine contains ANS: A


2. Who was the first prime minister of
(a) brake fluid (b) water (c) engine oil (d)
Nigeria?
fuel ANS: (a) Sultan of sokoto
B (b) Alh. Tafawa Balogun
(c) Alh. Tafawa Balewa
28. Balanced meal is best described as a meal (d) Alh. Tafa Balewa
that contains (a) a lot of proteins and
ANS: C
59
3. The largest bird in the world is …….. • Mr. Taiwo Akinkunmi
(a) Vulture (b) Mr. Taiwo Akinyemi
(b) Eagle (c) Mr. Taiye Akinkumi
(c) Ostrich (d) None of the above
(d) Swan
ANS: A
ANS: C 10. University of Ilorin is a
4. The full meaning of the acronym “GSM” is …………….generation University
(a) Global System Mobile • 1st
nd
(b) General System Mobile (b) 2
Communication (c) 3rd
(c) Global system of Mobile (d) 4th
communication
(d) General System Mobile ANS: B
11. Which of these is the first Nigerian
ANS: C University?
5. Nigeria as a country is not a member of • Obafemi Awolowo University
which of the following: (b) University of Ilorin
• U.N.O (c) University of Nsukka
(b) ECOWAS (d) Ahmadu Bello University
(c) AU
(d) None of the above ANS: C
12. What is the meaning of CBT?
ANS: D • Computer Basic Technology
6. Nigeria changed from Pounds to Naira in (b) Computer Basic Test
(a) 1972 (c) Computer Based Test
(b) 1971 (d) All of the above
(c) 1970
(d) 1975 ANS: C
13. Nigeria Federal Capital Territory shifted
ANS: A from Lagos to Abuja in what year?
7. Whose photograph appears in the N1000 (a) 1991
note
(a) Clement Isong and Alh Tafawa (b) 1989
Alewa
(b) Clement Isong and Alh Aliyu (c) 1990
Kingibe
(c) Clement Isong and Aliyu Maibornu (d) 1992
(d) Alh Maikingibe and Clement
Effiong ANS: A
ANS: C
8. The Green and White colours in Nigeria Flag 14. Nigeria celebrated her Silver Jubilee in the
symbolize year ……………
(a) Agriculture and Finance (a) 1985
(b) Peace and Prosperity
(c) Agriculture and Transparency (b) 2010
(d) Agriculture and peace
(c) 1995
ANS: D
9. Who designed the National flag?

60
(d) 2011 (d) guinea corn

ANS: A ANS: A
20. A man having 20 litres of petrol in his car,
15. People who carry out experiments to uses it for 600 km. How many kilometers will
investigate nature are called the same man travel with 19.25 litres.
(a) Scientist
(b) Experimentalist (a) 577.5km
(c) Experimentist
(d) Scientia (b) 576.2 km

ANS: A (c) 587.5km


16. Objects thrown up, always come down
due to (d) 586.2km
(a) kinetic force
(b) potential force ANS: A
(c) gravitational force
(d) Gravitational pull • -------------------------------decides what
should be included or left out in a
ANS: D newspaper.
17. Human beings have different types of
teeth and hence are referred to as • Publisher
(a) Honodent
(b) Homodent • Printer
(c) Heterodent
(d) Isodent • Editor

ANS: C • Journalist
18. ICT Means
ANS:
(a) Information and Communication C
Technology
• A person who writes for newspapers
(b) Information Communication or magazines is called a----------------
Technology
• Journalist
(c) International Communication
Technology • Printer

(d) Internal Community Technology • Editor

• Publisher
ANS: A

19. Which of the following is proteinous in ANS:


nature? A
(a) groundnut
(b) millet • Arrange these sentences in the
(c) beans correct order.

• The passengers got out.


61
ii. The bus stopped at the • Gold, Copper, Tin
corner.
• Milk, meat, fish
Iii. They all looked tired after the
• Coke, Fanta, Mirinda
trip.

• i, ii, iii ANS:


B
• i, iii, ii
• I went to the ----------------------- to
• ii, i, iii
read and borrow some books.
• ii, iii, i
• School

• Press
ANS:
C • Stationery

• I went to the --------------------- to • Library


watch a play.
ANS:
• Stage D

• Studio • The case was heard in the judge’s------


-?
• Theatre
• Court
• Chambers
• Arena
ANS:
D • Cell

• The lawn tennis match was played on • Witness box


a --------------
ANS:
• Court A

• Field • She took rolls of tissue off the ---------


in the supermarket.
• Stadium
• Shelves
• Track
• Stalls
ANS:
A • Desk

• Which of these can be described as • Checkout


minerals?
ANS:
• Calcium, Magnesium, Sulphur D
62
• He went to the ---------------- to buy • Opthamologist
some tomatoes to plant.
ANS:
• Granary
C
• Store
• Which of these is odd ?
• Nursery
• Jigawa
• Hatchery
• Kebbi

ANS: • Ekiti
C
• Zamfara
• A doctor who treats animals is a -------
---surgeon. ANS:
C
• Medical
• Pupils are advised to cross the road
• Veterinary
at---
• Vetinary
• Roundabout
• Vettinery
• T junction

ANS: • Children crossing


B
• Zebra crossing
• We have an orange -----------behind
our house.
ANS:D
• Orchard
• A system whereby a woman marries
• Farm
more than one man is --------------
• Garden
• Monogamy
• Bed
• Polygamy

ANS: • Polyandry
A
• Not in existence
• The specialist who examines the eye
for defects is an --------------------- ANS:
C
• Optician
• Which of the following is never true ?
• Eye surgeon
• Lizard lay eggs
• Optometrist

63
• Malaria is an airborne disease • Which of these is a communicable
disease
• Teachers are rich
• Bacteria
• Rice is a cereal
• Flu
ANS:
• Malaria
B
• Dialysis
• Which of the following is an airborne
disease.
ANS:
• cholera B

• Influenza • Which of the under listed belong to


the class Aves?
• Dysentery
• Monkey
• Guinea worm
• Tadpole
ANS:
• Eagle
B
• Rat
• Which of these item does not form
part of a first aid box.
ANS:
• Blade C

• Bandage • Animals that chew their cud are------

• Chloroquine • Carnivores

• Iodine • Herbivores

• Omnivores
ANS:
C • Ominivores
• All these animals live in water except
ANS:
• Frog B

• Lizard • How many faces have a cuboid?

• Snake • 4

• Turtle • 8

• 6
ANS:
B

64
• 12 • A farmer sold his cock at 20% loss. If
the cock was sold for ₦21:00 what
ANS: was the initial worth of the cock?
C
• ₦27:50
• A farmer sold 35 bags of rice at 25 %
• ₦25:20
profit. If he sold each bag for
₦7500:00. How much was his profit • ₦26:25
on of the lot.
• ₦20:20
• ₦1,875:00

• ₦65,625:00
ANS:
C
• ₦18,775:00
• In a class there are 50 pupils, if 15 of
• ₦26,250:00 them are girls, what percentage are
boys?
ANS:
• 70%
D
• 30%
• What is the product of 5Kg 350g and
9? • 85%
• 45kg 150g • 15%
• 14kg 350g
ANS:
• 48kg 150g A

• 14kg 150g • Write in words the sum of 31,004 and


117, 009.
ANS:
• Four hundred and twenty
C
seven thousand and thirteen.
• If eight and a half tonnes of sugar is
• One hundred and forty eight
sold in bags of 5kg. How many bags
thousand.
are there to sell ?
• Fourteen thousand and three
• 1.7 bags
• One hundred and forty eight
• 17 bags
thousand and thirteen
• 170bags ANS: D

• 1700 bags • Write 342 in Roman numerals.

• CCXLII
ANS:
D • CCCLXII
65
• CCCXLII • 15th April

• CCLXII • 21st April

• 26th April
ANS:
C • 16th April
• Write in Arabic numerals, CMVII
ANS:
• 907 D

• 1007 • President Ebele Jonathan was sworn


in as the acting president of the
• 1017 Federal republic of Nigeria on the
• 1107
• 5th May 2011

ANS: • 6th May 2010


A
• 7th May 2011
• Find the place value of 6 in 3421.016
• 7th May 2010
• 6 thousand
ANS:
• 6 hundred B
• 6 hundredths • In the 2011 elections held in Nigeria,
• 6 thousandths which two elections were held on the
same day.
ANS: • Gubernatorial and national
D assembly
• State the number of multiples of 4 • National assembly and
between 20 and 60 presidency
• 10 • State assembly and
• 6 presidency

• Gubernatorial and State


• 9
assembly
• 11 ANS:
D
ANS:
D

• The Nigerian 2011 presidential • Which of the following is the


election was held on the odd one out.

66
(a) Lawyers (b) Housefly

(b) Labourers (c) Butterfly

(c) Teachers (d) Bee

(d) Architects
ANS: C
ANS:
B • ................. Is the ability to do
some work
• A bowl of semolina that was left
over for three days was passing (a) Power
a.........odour.
(b) Energy
(a) Offensive
(c) Speed

(b) Aromatic (d) Effort

(c) Inviting ANS: B

• The child’s mother’s sister is


(d) Appetizing the ................ of that child
(a) Mate

(b) Aunt
ANS: A
(c) Uncle
• The square root of 256 (d) Sister
is..............

(a) 13 ANS: B

• The child’s mothers sister is the


(B) 14
................ of that child
(c) 16
(a) Mate
(d) 12
(b) Aunt

ANS: C (c) Uncle

• A caterpillar is an infant.............. (d) Sister


.
ANS: B
(a) Mosquito
67
• What happens when water and (c) Wood
oil of equal proportion are
poured together in a container (d) Paper

(a) Oil comes on top ANS: A


(b) Water comes on top
• Equal amount of red and blue
(c) The two changes colour colour mixed together will give.

(d) Nothing happens (a) Green


(b) Orange
ANS: A
(c) Red
• Okon invites Jude to his
birthday party Okon is the (d) Purple
.............. while Jude is.........
ANS: D
(a) Host, hostess
• Inter-Science is the combination
(b) Hostess, landlord
of.
(c) Host, guest
(a) Mathematics, Physics,
(d) Guest, landlady Biology
(b) Chemistry, Physics, Biology
ANS: C
(c) Geography, Physics, Health
• Time is determined by the use
Science
of
(d) Chemistry, Physical
(a) Colour
Education
(b) Rain ANS: B

• Paragraph is a group of
(c) Hour
.....................
(d) Clock • Words

ANS: D • Letters

• The core material for producing • Sentences


ceramic ware is.............
• Permutation
(a) Clay
ANS: C
(b) Plastic

68
• An official representing a (b) Wind clock
country abroad is a.................
(c) Wind Vane
• Diplomat
(d) Baro meter
• Visitors
ANS: C
• Foreigners
• The head of a local government
• Spies council is called...............

(a) Chairman
ANS: A (b) Councillor

• The acronym SARS (c) President


means...............................
(d) Senator
• Special Anti-Robbery Squad
• Special Autorobbery Squad ANS: A
(c) Special Anti-Robbers Squad
(d) Special Automated Squad • These are parts of the fish EXCEPT.

ANS: A (a) Leg


• “I challenge the public to put
(b) Fin
teeth into the law
means”.................. (c) Tail

• To council the law (d) Eyes

• To reject the law ANS: A


• To activate the law • Which of these is worn when going
• To review the law to bed.

(a) Dressing gown


ANS:
C (b) Apron

(c) Night dress

(18) The instrument for finding the (d) Pyjamas


direction of the wind is called...............
ANS: C
(a) Wind hook
69
• It is valued because of their (a) Yoke
fragrance and colour of their
(b) Baby
flowers.
(c) Albumen
(a) Rose
(d) Shell
(b) Vanilla

(c) Spoorid ANS: C

(d) Paper • Battering involves....................


one’s wife.
ANS: A
(a) Kissing
• The language ....................... is as
(b) Beating
follows; hearing and speaking.
(c) Caring
(a) Stages
(d) Carrying
(b) Voice

(c) Say ANS: B

(d) Mechanism • The framework of the body to


which modes are attached is
ANS: D called...............

• What ethic group in Nigeria is (a) Marrow


known for cattle rearing/farming?
(b) Head
(a) Yoruba
(c) Skeleton
(b) Igbo
(d) Back
(c) Fulani
ANS: C

• Fruits and vegetables are full


(d) Hausa of..........................

(a) Proteins
ANS: C

• What is the name given to the (b) Pork


white part if an egg? (c) Fat

70
(d) Vitamins (d) Town crier

ANS: D ANS: B

• Which of these international • .................... is/are usually


organisation is responsible for found along major roads to
the development of education advertise products.
and culture.
(a) Handbills
(a) UNICEF
(b) Newspaper
(b) ECOWAS
(c) Bill boards
(c) UNESCO
(d) Television
(d) Wai
ANS: C
ANS: C
• Which of these will be served as
• Television appeals to sences a snack?
of.................. and seeing.
(a) Fried rice
(a) Hearing
(b) Bean soup
(b) Booking
(c) Amala
(c) Knocking
(d) Egg roll
(d) Hissing
ANS: D
ANS: A
• ....................... is a factor to be
• ..................... is the fastest way considered in locating an
of getting information to industry.
millions of people at the same
(a) Reading market for the
time.
products
(a) Newspaper
(b) Available labour force
(b) Radio
(c) Availability of raw materials
(c) Television

71
(d) Presence of government to..................... in the nursery
official schools.
ANS: C
(a) Dance
• The art of doing what you are
asked to do is called................... (b) Eat

(a) Listening (c) Think

(b) Teachers (d) Teach

(c) Obedience ANS: D

(d) Complaining • The University of Ilorin was


founded in ........................
ANS: C
(a) 1975
• All are adhesive EXCEPT
(b) 1940
(a) Evostik
(c) 1967
(b) Araldite
(d) 1907
(c) Ponal glue
ANS: A
(d) Water
• Vocational studies involves the
ANS: A study of the following subjects
EXCEPT.
• Puppets are..................objects
that can be controlled to (a) History
perform various functions.
(b) Typing
(a) Natural
(c) Shorthand
(b) Oblong
(d) Introductory technology
(c) Inanimate
ANS: A
(d) Long
• The literal meaning of
ANS: C renaissance is..........................
• Puppets are used as (a) Revival
instructional materials
72
(b) Forgo (d) Civil Defence

(c) Remembrance
ANS: C
(d) Out dated • Good reading culture is done in
the.........................
ANS: A
(a) Darkroom
• The tallest tower under
construction in Abuja (b) Bathroom
is.........................
(c) Library
(a) Millennium
(d) Kitchen
(b) Decade
ANS: C
(c) London
• Nigeria does not have this
(d) Ivory cost military branch the United
States of America has.
ANS: A
(a) Navy
• A.....................is one of the tools
for bricklaying. (b) Air force

(a) Spanner (c) Infantry

(b) Nail (d) Marine

(c) Trowel
ANS: D
(d) Scriber • In which state is Madonna
University situated in Nigeria.
ANS: C
(a) Kwara
• The following maintains law and
order EXCEPT. (b) Ekiti

(a) Police (c) Anambra

(b) Air force (d) Bayelsa

(c) Engineer
ANS: C

73
• When did Nigeria join the World (b) West Africa Examination
Trade Organisation. Council

(a) 1995 (c) West Africa Examination


Councils
(b) 1904
(d) West Africa Examination
(c) 2004
Council
(d) 1996 ANS: A

ANS: A
• What fraction of N4.32 is N
• The following are stationery
EXCEPT. 3.87

(a) Stapler • 41/48


(b) Exercise book • 43/48

(c) Abacus Ans


B
(d) Office pin
• 47/48
ANS: C • 39/48
• The following are types of • Yinka and Segun have 120
camera EXCEPT.
oranges Yinka gets 16
(a) Kodak
oranges more than Segun.
(b) Samsung
How many oranges has
(c) Sony

(d) Supper Kodak Segun?

• 60
ANS: D
• 26
• The full meaning of WAEC
is..................... • 68
• 52
(a) West African Examinations
Council Ans
D
74
• If 480.00 borrowed for 3 What time did it get to

years simple interest Ibadan?

became N516, what was its • 8.42am

rate of interest? • 9.42am

• 5% Ans
• 2½%
B
Ans
• 9.32am
B
• 6% • 9.30am
• 4%
• By how much is 845.79
• The daily sales in a week at a
greater than 99.359?
petrol station are 100 litres,
(a) 746.431
825 litres, 707 litres, 830
Ans A
litres, 642 litres, 908 litres

and 112 likes. What is the (b) 74.6431

average daily sales? (c) 835.8541

• 848 litres
(d) 83.5841
• 589 3/7 litres
Ans
• A square lawn has an area of
B
729 sq.m. What is the
• 718 litres
• 617 litres distance around the lawn?
• A car traveling at 80 km per
• 96
hour leaves Lagos at 8.am

for Ibadan 136km away.


75
• 108 • 1035 seeds

Ans Ans

B D

• 120 • Find the greater number

• 84 that will divide 43, 91 and

• In a school, the ratio of girls 183 so as to leave the same


to boys is 5:6. How many are
remainder in each case.
boys if the total population
• 4
is 605?
• 365 Ans
• 275
A
• 330
• 7
Ans
• 9
C
• 13
• 560
• The total number of digits
• A class-one teacher gave 23
used in numbering the pages
counting seeds to each of of a book having 336 pages
is:
the 45 pupils in his class.
• 732
How many counting seeds
• 990
did the teacher give out? Ans
B
• 225 seeds
• 1098
• 935 seeds
• 1305
• 2070 seeds

76
• Which of the following pairs • 96

of fractions adds up to a • 166

number greater than 5? Ans

• 5/3, 3/4 B

• 7/3, 11/5 • 160

• 11/4, 8/3 • 66

Ans • What is the product of 0.101

C and 11.1?

• 13/5, 11/6 • 12.21

• What is the difference • 1.1211

between the biggest and the Ans

smallest fraction among 2/3, B

3/4 , 4/5 and 5/6? • 1.211

• 1/6 • 0.11211
Ans
• What is 7/8 of the profit of a
A
contractor in a year, if half of
• 1/12
• 1/20 the profits made is
• 1/30
N2880.72?
• What number whose ½ is
• N5040.26
multiplied by 1/3 of the same
• N5041.26
number will give a product
Ans
of 726?
B
77
• N2520.63 • What principal will gain

• N5041.36 N168.00 simple interest in 4

years at 7 ½ %

• When 7867 is divided by 14, • 280.00

what remains? • N480.00

• 11 • N560.00

• 13 Ans

Ans C

B • N720.00

• 3 • The average of 3 bags of

• 7 cocoa is 4,621g. Two of the

• Eggs now sell at N2.55 a bags are 4,196g, and 4.706g

dozen and oranges 2 for 10k. each. What is the weight of

What will 20 dozen eggs and the third bag?

100 oranges cost? • 3.961g

• N48.00 • 4.961g

• N60.50 Ans

• N56.00 B

Ans • 5.091g

C • 5.096g

• N44.00

78
• Mary was 21 years on 3rd as an old man. How old is
he?
October, 1969, when will
• 48 years
she be 29 years old?
• 52 years
• 1972 • 60 years

• 1977 Ans

Ans C

B • 78 years

• 1981 • If 8 men can reap 80


hectares in 24 days, then
• 1989
how many hectares can 36
• A man’s monthly salary is
men reap in 30 days?
N770.00 What will be his • 300
• 350
new salary after it has been
• 425
increased by 10%?
• 450
• N700.00 Ans
D
• N787.00
• A is twice as fast as B and B
• N847.00
is thrice as fast as C is. The
Ans
journey covered by C is 54
C
minutes will be covered by B
• N8875.00
in:
1
• Ojo lived /4 of his life as a
boy, 1/5 as a young man and
1
/3 as a trader and 13 years

79
• 18 min • HIV/AIDS can be contracted

Ans through

A • Sharing of clippers for

• 27 min barbing hair Ans A

• 38 min • Hand shake

• 42 min • Worshipping together

• The Senate of the Federal • Using the same plate to

Republic of Nigeria has ____ eat

members • Parliamentary Filibuster

• 37 strategy is a method of delaying

• 109 decision in the parliament

Ans B through ____

• 170 (a) adjournment

• 108
(b) long speech
• Kerosene is a product refined
Ans B
from
(c) physical combat
• Petrol

• Petroleum Jelly (d) absenteeism

• Crude oil
• Urine contains urea, water and
Ans
____
C
• hydrogen
• Plan oil
80
• soldium • The power to adjudicate in law

• H 20 is the sole responsibility of ____

• Salt in Nigeria

Ans D • Lawyers

• Ceasarian section is a process of • Governor

giving birth to a baby by a • Judges

woman through ____ Ans C

• Labour • Police

• Medication • One of the agents of

• Oral therapy socialization in a society is ____

• Operation • Culture

Ans D • Sports

• Nigeria’s representative in any • Family

of the common wealth Ans C

countries is called ___ • None of the above

• Diplomat • Kwara State is located in the

• High commissioner ____ of geo-political zone of

Nigeria.
Ans B
• South Western
• Ambassador
• North Western
• Consult general
• South-eastern

• North Central

Ans D
81
• The House of Representative is • North Central

also called Ans C

• The Upper house • North West


• The amalgamation of the
• The Inner temple
• The Lower house protectorate of the North and

Ans C South in Nigeria took place in


• The Assembly hall the year?
• _______ is the capital of Yobe • 1889
state • 1909
• Maiduguri • 1914
• Kaduna Ans C
• Damaturu • 1922
Ans C • What year was University of
• Yola Ilorin established
• The slogan “Boko Haram” • 1975
means Ans A
• No to Haram • 1976
• Haram is lawful • 1978
• No to western education • 1979
Ans C • Whose face is on a five hundred
• Islamic education is good naira note
• The Federal Capital Territory • Chief Obafemi Awolowo
(FCT) is in which of the • Alhaji Mai Bornu
geopolitical zone of Nigeria? • Dr. Nnamdi Azikiwe
• North East Ans C
• South East
• Alhaji Sir Ahmadu Bello
82
• The main river flowing through d. 3.41%
ANS C
Nigeria is
64. A surveyor measured a road as being
• Benue 95km with 5% underestimation. What is the
true length of the road?
• Niger
a. 90km
Ans B
b. 95km
• Kaduna
c. 100km
• Victoria
d. 105km
ANS C

61. A pencil 20cm long was recorded as 22cm 65. A man’s annual salary is N132,000. What
long. What is the percentage error? is his monthly salary?

a. 22% a. N11,000

b. 20% b. N12,000

c. 10% c. N13,000

d. 5% d. N22,000
ANS C ANS A

62. A student made a 5% error in the 66. A man earns N20,000 a month. What does
measurement of a 50cm long stick. What was he earn in a year?
the student’s measurement?
a. N200,000
a. 50cm
b. N240,000
b. 60cm
c. N220,000
c. 512.5cm
d. N280,000
d. 55cm ANS B
ANS C
67. If the exchange rate is US$ 1 = N150, how
63. A can of Cola was labeled as containing much is US$ 500 in naira?
330ml of fluid but was found to contain
a. N150,000
341ml. What was the percentage error?
b. N500,000
a.33.0%
c. N7,500
b. 34.1%
d. N75,000
c. 3.33%
ANS D
83
68. How much is N3,000 worth in US$ if the 72. A girl spent 1/2 of her money at the
exchange rate is US$ 1 = N120? market and 1/5th of the remaining amount at
the bakery. If she had N1,000 originally, how
a. US$ 65 much has she left?
b.US$ 55 a. N100
c. US$ 45 b. N200
d. US$ 35 c. N300
ANS B
d. N400
69. Which is the greatest of the following
ANS D
fractions?
73. If Ayuba bought 5 oranges for N7, how
a. 7/9 many oranges of the same type can he buy for
b. ¾ N49?

c. 5/8 a. 15 oranges

d. 2/3 b. 35 oranges
ANS A c. 25 oranges
70. After spending 2/3 of her money on food, d. 30 oranges
a woman was left with N250. How much ANS B
money did she have originally?
74. If a can of malt drink costs N65, how much
a. N250 is a dozen cans of malt drink ?
b. N500 a. N780
c. N750 b. N715
d. N1,000
c. N650
ANS C
d. N585
71. A man with N5,000 spent 1/5th at the ANS A
Chemist and ¼ at the Electric shop. How much
money did he have left? Bunmi can hoe a yam plot in 5hrs. and Dele
can hoe it in 4hrs:
a. N2,750
75. If the two men hoe, what fraction of the
b. N3,750 yam plot will be left to hoe after 2hrs?
c. N4,000
a. ¼
d. N1,000
b. 1/5
ANS A
c. 1/8

84
d. 1/10 c. 200,000 votes
ANS D
d. 250,000 votes
76. What fraction of the yam plot will they ANS B
hoe in 1hr?
80. A factory increased its annual production
a. ¼ of radios from 40,000 to 50,000. Calculate the
percentage increase in production.
b. 9/20
a. 25%
c. 1/9
b. 10%
d. 9/10
ANS B c. 15%

In an election the winner received 3/7 of the d. 20%


total vote: ANS A

77. If the total number of votes cast were 81. In April, Mrs. Ngosi bought a bag of corn
700000, what number of votes did the winner for N1,200. When she bought more in June,
receive? the price had gone up by 20%. How much did
she pay in June for a bag of corn?
a. 100,000 votes
a. N1,300
b. 200,000 votes
b. N1,400
c. 300,000 votes
c. N1,440
d. 400,000 votes
ANS C d. N1,340
ANS C
78. If the total votes cast were 1.4 million,
what total votes did the losers receive ? A trader made a loss of 10% on a motorcycle
he sold for N54,000:
a. 800,000 votes
82. How much should he have sold the
b. 600,000 votes motorcycle to avoid any loss?
c. 1 million votes a. N56,000
d. 1.2 million votes b. N58,000
ANS A
c. N60,000
79. If there were 3 candidates at the election
and one of the losers received 5/14 of the d. N62,000
votes, what number of votes did the other ANS C
loser receive ?
83. How much should he have sold the
a. 100,000 votes motorcycle to make a gain of 10%?

b. 150,000 votes a. N60,000


85
b. N62,000 • Memorandum of Understanding

c. N64,000 • Memorandum of Unity

d. N66,000 • Memorandum of Union


ANS D
• Moment of Unity
A train moves at a speed of 55km/hr:
Ans: a
84. How long will it take to cover 385km?
6. UNO stands for
a. 5hrs.

b. 7hrs. • United Nations Organisations

c. 9hrs. • Unity Nations Organisations


d. 11hrs.
• United Nation Organogram
ANS B
• United National Organisations
85. How far will it go in 3hrs?
Ans: a
a. 165km

b. 175km 7. The UNO was formed in the year

c. 185km
• 1945
d. 195km
• 1947
ANS A
• 1958

• 1972

Ans: a

4. ETF stands for

28. All of the following are correct


• Educational Trust Fund
procedures for putting out a fire in a pan on
• Education Trust Fund
the stove except:
• Education Total Fund

• Economics Trust Fund a. Leave the pan where it is.

Ans: b
b. Pour water into the pan.
5. What is the full meaning of MoU?
c. Slide a fitted lid onto the pan.
86
d. Turn off the burner controls d. A preposition

Ans: b Ans: a

29. Which of the following is a characteristic 32. To avoid infection after receiving a

of a virus? puncture wound to the hand, you should

always:
a. It can cause disease.

a. Go to the immunization center to receive a


b. It can reproduce by itself.
tetanus shot.
c. It is composed of large living cells.
b. Be treated with an antibiotic if the wound is
d. It lives in plant and animal cells.
painful.
Ans: a & d
c. Ensure that no foreign object has been left
30. Which of the following was the principal
in the wound.
keyboard instrument in 16th century Europe?
d. Wipe the wound with alcohol unless it is
a. Clavichord.
still bleeding. Ans: c

b. Harpsichord. 36.The longest river in the world is the

c. Organ. • Niger

• Nile
d. Pianoforte.
• Orange
Ans: b
• Mississippi

31. A word used to describe a noun is called: Ans:

B
a. An adjective.
37.The largest coffee growing country in the
b. A conjunction.
world is

c. A pronoun. • France

87
• Brazil of rubber in the world is

• Ethiopia
• China
• Italy
• Mali
Ans:
• Malaysia
B
• Japan
39.The country also known as "country of
Ans c
copper" is
44.The country which is the largest producer
• Brazil
of tin in the world is
• Cameroon
• Japan
• Zambia
• South Korea
• Belgium
• Finland
Ans: c
• China
40.The river Jordan flows out into the
Ans: d
• Red sea
45.The river which carries maximum quantity
• Dead sea
of water into the sea is the
• Live sea

• Mediterranean sea • Orange

Ans: B • Nile

42. The world's oldest known city is • Benue

• Amazon River
• Rome
Ans: d
• Jeddah
46.The country called the Land of Rising Sun is
• Damascus

• Paris • China

Ans: C • Belgium

43.The country which is the largest producer • Italy

88
• Japan • Italy

Ans: d • Germany

47.The country known as the Sugar Bowl of • England

the world is • U.S.A

Ans: c
• Mexico
51.World Environment Day is observed on
• Italy

• Poland • 5th June

• Cuba • 13th April

Ans: d • 1st November

48.The world's largest diamond producing • 13th October

country is Ans: a

World literacy day is celebrated on


• Australia

• Morocco • 8th August

• China • 23rd November

• South Africa • 31st December

Ans: d • 8th September

49.The national flower of Britain is Ans: d

54.The first British University to admit women


• Rose
for degree courses was
• Lily

• Narcissus • University of Dundee

• Hibiscus • University of Cambridge

Ans: a • Temple University

50.The first Industrial Revolution took place in • London University

Ans: d

89
55.The principal export of Jamaica is 59.Who invented the Light Bulb?

• Coffee • Humphry Davy

• Kola • James King

• Sugar • Alva Fisher

• Salt • Hubert Booth

Ans: c Ans: a

56.New York is popularly known as the city of 60.Which country hosted the Football World

Cup in 2006?
• Cloves

• Skyscrapers • Italy

• White Elephant • Germany

• Morning Calm • Belgium

Ans: b • France

57.What do you call a group of sheep? Ans: b

61. Speed of computer mouse is measured in


• Flock
which unit?
• Hockey

• Herd • Joule

• gang • Watt

Ans: a • GB

58.How many legs do butterflies have? • Mickey

Ans: d
• 6
62. Which University topped Forbes list of
• 4
'Billionaire Universities' in 2010?
• 7

• 8 • Harvard University

Ans: a • University of Readings

90
• Cambridge University • 1 thousand

• University of Nigeria, Nsukka Ans: a

Ans: a • Who developed the small pox

vaccination?

65. Which bird is the international symbol • Edward Jenner

of happiness? • Alexander Fleming

• Ostrich • Albert Einstein

• Owl • None of these


• Bluebird Ans: a
• Redbird
• Who is also known as the Lady with
Ans: c
the Lamp?
66. The city of Bonn is situated in
• Florence Nightingale
• Germany
• Sarojini Naidu
• Italy
• Rani Laxmibai
• France
• Bachendri Pal
• Canada Ans: a
Ans: a
• The American General who led the
• Approximately, how many people revolt against the British & declared
speak Chinese language? American independence was:

• billion • George Washington

• million • Bill Clinton

• 1 lakh • George Bush

91
• Abraham Lincoln • Badminton is the national sport at:

Ans: a • Malaysia

• Who was first to sail sound the strait, • Scotland


reached the Philippines and named the
• China
Pacific Ocean?
• Former soviet Union
• Ferdinand Magelion
Ans: a
• Jacques Carter
• On which date is Nobel Prize awarded?
• William Janszoom
• December 10
• Vasco da Gama
• January 10
Ans: a
• April 10
• Arena? is the special name for
• July 10
playground of:
Ans: a
• Cricket
1. What is the full meaning of
• Lawn Tennis NEPAD?
(a) Nigeria Economic
• Wrestling Partnership of Advance Development
(b) National Enterprise and
• Skating Promotional Advancement
(c) New Partnership of African
Ans: c Development
(d) None of the above
• The national sport of Canada is: ANS C
2. …………………………….. was
• Tennis and cricket the 1st Civilian Governor to be
impeached during the fourth
• Lacrosse Republic?
(a) Balarabe Musa
• Judo (b) Rasheed Ladoja
(c) D S P Alameseigha
• Rugby and Football ANS C
Ans: b
92
6. The Directorate of Food, Roads • Nigeria
and Rural Infrastructures was ANS
established during the regime of C
29. The Corporate body saddled with
(a) Muritala- Obasanjo
the responsibility of supervising,
Regime 1976-1979
controlling and administering the
(b) Abacha-Diya Regime
affairs of the registered companies
1993-1998
in Nigeria today is
(c) Obasanjo-Yar adua
(a) Companies Registry
Regime 1976-1979
(b) Securities and Exchange
• Babangida Regime
ANS Commission
D (c) Nigerian Deposit Insurance
Company
7. The National Health Insurance • Corporate Affairs
Scheme was introduced by the Commission
Federal Government in ANS D
th
36. The 12 Under 17 World FIFA
year………………………
Football competition was won by
(a) 2000
(a) Switzerland
(b) 1999
(b) Spain
(c) 2004
(c) Nigeria
• 2002
ANS • Colombia
B
ANS C
13. The Nomadic system of Education
2. A class of 27 students has a 33% pass. How
was introduced in Nigeria during
many of them passed?
the reign of
…………………………….. as a (a) 9
Minister of Education
(a) Dr. Sam Egwu (b) 10
(b) Professor Jibril Aminu (c) 12
(c) Professor Aborishade
Babalola (d) 21.
(d) Professor Aliyu Babatunde ANS A
Fafunwa ANS
3. A graduating class has 420 candidates. If 5%
D
of them are in honors roll and the
requirement for being in honors roll is to
9. The Headquarter of African Union
score the minimum of 75%, how many
is in
candidates scored below 75%?
(a) Libya
(b) South Africa (a) 400
(c) Ethiopia
(b)399

93
(c) 200 (a) larger than

(d) 199. (b) smaller than


ANS B
(c) equal to
4. A graduating class has 420 candidates. If 5%
(d) twice that of the federal representative.
of them are in honors roll and the
requirement for being in honors roll is to ANS A
score the minimum of 75%, how many 17 Which of the following statements about
candidates made the honors roll? language is incorrect?
(a)321 (a) language barrier often created problems
(b) 40 (b) language barrier often led to
(c) 30 discrimination

(d) 21 (c) language barrier often bred suspicion


ANS D (d) language barrier often created friendship
5. If the class average of a 100 student class is ANS D
50% and 25%of the students scored 70%. How 18.Computer logic is based on two digits
many scored below 75%? namely:
(a) 50 (a) 0,1
(b) 75 (b) 1,10
(c) 60 (c) 1,2
(d) 25 (d) 0,9
ANS B ANS A
6. The take-over progression to the 19.The numbers 0 and 1 in computer
presidency in case of death or step-aside of language is referred to as
the president is
(a) byte
(a) vice president, senate president, speaker
of the house of representative (b) base

(b) vice president, , speaker of the house, (c) bit


senate president
(d) term.
(c) senate president, speaker of the house, ANS C
vice president
20 Computer operations is based on
(d) speaker of the house, vice president,
(a) logic
senate president. ANS A
(b) arithmetic
9.The consistuency of the senator is
94
(c) Philosophy the road a woman was travelling to Lagos
from Ibadan in another bus. Suddenly a car
(d) English language. passed the bus the man was travelling in. How
ANS A does the man feel as he watches the woman’s
21 Internet connectivity is based on vehicle speed by in the opposite direction?

(a) radio communication (a) his bus is accelerating

(b) computer communication (b) it feels like the distance of separation


between the two vehicles is rapidly increasing
(c) television communication
(c) it feels as if the woman’s bus is much
(d) video communication. faster than his bus
ANS B
(d) it seems that the car was overtaking the
22 There was lightning and this was followed bus caused it. ANS C
by thunder. The boy who saw the lightning
flash was not afraid of lightning but got scared 28 A man was travelling from Lagos to Ibadan
when the thunder roared. Which of these is in a bus and on the opposite side of the road a
dangerous? woman was travelling to Lagos from Ibadan in
another bus. Suddenly a car passed the bus
(a) lightning the man was travelling in. The car overtaking
them appears slow even though it is
(b) thunder
overtaking them because
(c) both
(a) the car is not really fast
(d) none.
(b) the difference between the speed of the
ANS A car and that of the bus is small
26 A man was travelling from Lagos to Ibadan
(c) the car slowed down when he got to the
in a bus and on the opposite side of the road a
bus
woman was travelling to Lagos from Ibadan in
another bus. Suddenly a car passed the bus (d) the driver of the car was no longer paying
the man was travelling in. How does the man attention to his driving ANS B
feel with respect to the car ?
30 .A man was travelling from Lagos to Ibadan
(a) bus faster than the car in a bus and on the opposite side of the road a
woman was travelling to Lagos from Ibadan in
(b) bus moving backwards
another bus. Suddenly a car passed the bus
(c) the driver of the car is slowing down the man was travelling in. If the man in the
bus was to speak to the woman in the other
(d) the bus driver was no longer moving at his bus and also to the occupant of the car, which
speed before the car approached them ANS one was more likely to hear distinctly what he
B is saying?

27. A man was travelling from Lagos to • The woman


Ibadan in a bus and on the opposite side of
95
(b) The car occupant (d) It is what government
recommends ANS
(c)Both of them
C
(d)None of them
55. In a class of boys and girls,
ANS B
(a)the tallest girl must be the tallest in
49.A 5 sided figure is called a
class
(a)quadrilateral
(b)the tallest boy must be the tallest
(b)septagon in class

( c)pentagon ( c)the shortest girl could be the


shortest in the class
(d)octagon
ANS C (d)the shortest boy must be the
shortest in the class. ANS
51 The elements found in carbohydrate C
are:
1 Odometer is to mileage as compass is to
(a)hydrogen , nitrogen, oxygen A. speed B. hiking
C. needle D. direc
(b)hydrogen, Carbon, nitrogen

(c)hydrogen, oxygen, water Answer: Option E

(d)hydrogen, carbon, oxygen 2 Marathon is to race as hibernation is to


ANS D A. winter B. b
C. dream D. s
52 Another name for carbohydrate is
Answer: Option D
(a)sugar
3 Window is to pane as book is to
(b)protein
A. novel B. g
(c)mineral C. cover D. p

(d)vitamin Answer: Option C


ANS A
4 Cup is to coffee as bowl is to
53 Why is breast milk superior to canned milk A. dish B. s
for a new baby? C. spoon D. f

(a) It is warmer
Answer: Option D
(b) It is very concentrated
5 Yard is to inch as quart is to
(c) It is balanced in body building A. gallon B. o
requirements C. milk D. l

96
Answer: Option A A. Dagger B.
C. Knife D.
6 Choose the word which is different from the rest.
A. Chicken B. FrogAnswer: Option B

C. Swan D. 16 Choose the word which is different from the r


Crocodile
A. Deck B.
Answer: Option A C. Stern D.

8 Choose the word which is different from the rest. Answer: Option B
A. Kiwi B. Eagle
C. Emu D. 17 Choose the word which is different from the r
Ostrich

Answer: Option B

9 Choose the word which is different from the rest.


A. Curd B. Butter
C. Oil D. Cheese
A. Entrepreneur
Answer: Option C

10 Choose the word which is different from the rest.


A. Tea B. Cocoa
C. Rubber D. Chalk

Answer: Option D

11 Choose the word which is different from the rest. C. Investor


A. . Hangar B. Platform
C. Dock D. Park Option B
Answer:

18 Choose the word which is different from th


Answer: Option D

Choose the word which is different from the rest. A. Tricycle B.


12
A. Sparrow B. Duck
C. Parrot D. Chicken

Answer: Option B C. Trifle D.

13 Choose the word which is different from the rest. Answer: Option C
A. Tall B. Huge
C. Thin D. 19 Choose the word which is different from th
Sharp

Answer: Option D

15 Choose the word which is different from the rest. A. Chameleon B.


97
Choose the word that is a necessary
le
26 part of the underlined word.
desert Lo
A. cactus B. arid cu
C. Alligator C. D. oasis D. flat st

Answer: Option D Answer: Option B

20 Choose the word which is different from the the


Choose rest.word that is a necessary
27 part of the underlined word.Y
book ea
A. Calendar A. B. fiction B. pagesr
C. pictures D. Shelf M
o
nt
Answer: Option B
C. Day D. h
Choose the word that is a necessary
Answer: Option A 28 part of the underlined word.
language
21 Choose the word which is different from the rest.
A. Dictation B. slang
Gog
A. Spectacles B.C. writing D. gles
words
Micr
Answer: Option D oph
C. Binoculars D.Choose the word that is aone necessary
29 part of the underlined word.
Answer: Option D school
report
A. student B. card
22 C. the
Choose the word which is different from testrest. D. learning
Answer: Option A Trump
A. Harmonica B. et
C. Flute 4 D. Look at this series: 2,Violin
1, (1/2), (1/4), ... What number shoul
1
Answer: Option D A. (1/3) B. (1/8)
C. (2/8) D. (1/16)
23 Choose the word which is different from the rest.
A. Cheetah B. Answer: OptionLionE
C. Bear 4D. Look at this series:Tiger
7, 10, 8, 11, 9, 12, ... What number shou
2
Answer: Option C A. 7 B. 10
C. 12 D. 13
24 Choose the word which is different from the rest.
A. Sheep B. Answer: Option B Lamb
C. Goat D. Ram
4 Look at this series: 36, 34, 30, 28, 24, ... What number shou
3
Answer: Option C
A. 20 B. 22
C. 23 D. 26
98
Answer: Option B

4 Look at this series: 22, 21, 23, 22, 24, 23, ... What number should come next?
4 C. 530
A. 22 B. 24
C. 25 D. 26 Answer: Option A

Answer: Option C 52 The Pacific yew is an evergreen tree


that grows in the Pacific Northwest.
4 Theshould
Look at this series: 21, 9, 21, 11, 21, 13, 21, ... What number Pacificcome
yew next?
has a fleshy,
5 poisonous fruit. Recently, taxol, a
A. 14 B. substance
15 found in the bark of the
Pacific yew, was discovered to be a
C. 21 D. 23
promising new anticancer drug.
Taxol is poisonous when taken
Answer: Option B
A. by healthy people.
Taxol has cured people from
4 Look at this series: 58, 52, 46, 40, 34, ... What number B. should comediseases.
various next?
6 People should not eat the fruit
A. 26 B. C. 28
of the Pacific yew.
C. 30 D. 32
The Pacific yew was considered
worthless until taxol was
Answer: Option B D. discovered.

4 Look at this series: 1.5, 2.3, 3.1, 3.9, ... What number should come next?
7 Answer: Option C
A. 4.2 B. 4.4
C. 4.7 D. 5.1
53 Last summer, Mike spent two weeks
Answer: Option C at a summer camp. There, he went
hiking, swimming, and canoeing. This
4 summer,
Look at this series: 2, 4, 6, 8, 10, ... What number should comeMike
next?looks forward to
8 attending a two-week music camp,
A. 11 B. where he hopes 12 to sing, dance, and
C. 13 D. learn to play the guitar.
14
Mike's parents want him to
Answer: Option B A. learn to play the guitar.
Mike prefers music to outdoor
4 B. activities.
Look at this series: 201, 202, 204, 207, ... What number should come next?
9 Mike goes to some type of
C. camp every summer.
A. 205 B. 208
D. Mike likes to sing and dance.
C. 210 D. 211

Answer: Option D
Answer: Option D
5 Look at this series: 500, 509, 518, 527, ... What number should come next?
0
A. 536 B. 4
99
54 Erin is twelve years old. For three motel and they were relieved to find availability
years, she has been asking her parents on such short notice.
for a dog. Her parents have told her
that they believe a dog would not be
happy in an apartment, but they have
given her permission to have a bird.
Erin has not yet decided what kind of Maya and Julian take beach vacations every
bird she would like to have. A. year.
Erin's parents like birds better B. The motel is overpriced.
A. than they like dogs. C. It is usually necessary to book at least six
B. Erin does not like birds. months in advance at the motel
Erin and her parents live in an D. Maya and Julian decided to change their
C. apartment.
vacation plans because of the storm.
Erin and her parents would like
D. to move.

Answer: Option D
Answer: Option C

5 Seahorse populations have declined everywhere


5 Tim's commute never bothered him because
7 that seahorses are fished. During the past five
5 there were always seats available on the train
years, seahorse populations have decreased by
and he was able to spend his 40 minutes
50%. Last year, biologists met to discuss what
comfortably reading the newspaper or catching
might be done to reverse this trend.
up on paperwork. Ever since the train schedule
Seahorses are likely to become extinct within
changed, the train has been extremely
A. five years.
crowded, and by the time the doors open at his
One way to increase seahorse populations is
station, there isn't a seat to be found.
B. to ban the fishing of seahorses.
Tim would be better off taking the bus to
Biologists from all over the world are working
A. work.
C. to save the seahorses.
Tim's commute is less comfortable since
Seahorse fishermen have spoken out against
B. the train schedule changed.
D. the biologists.
Many commuters will complain about the
C. new train schedule.
Tim will likely look for a new job closer to
D. home. Answer: Option B

58 Ten new television shows appeared


Answe
during the month of September. Five
r:
of the shows were sitcoms, three were
Option
hour-long dramas, and two were
B
news-magazine shows. By January,
only seven of these new shows were
still on the air. Five of the shows that
5 When they heard news of the storm, Maya and
remained were sitcoms.
6 Julian decided to change their vacation plans.
Only one of the news-magazine
Instead of traveling to the island beach resort,
A. shows remained on the air.
they booked a room at a fancy new motel in the
Only one of the hour-long
mountains. Their plans were a bit more expensive,
B. dramas remained on the air.
but they'd heard wonderful things about the
100
At least one of the shows that 1. I am 10 years old my sister is 4, in how
was cancelled was an hour- many years shall I be twice as old as she will
C. long drama. be?
Television viewers prefer
D. sitcoms over hour-long dramas. A. 3 B. 4 C. 2
D. 5 ANS C

Answer: Option C 2. Bola had fewer sums right than ojo, ojo had
fewer sums right than Dele. Which had the
most right answers?

A. Bola B. Ojo and Bola C. Dele


59 On weekends, Mr. Baba likes to spend
D. Bola and Dele ANS C
many hours working in his vegetable
and flower gardens. Mrs. Baba spends 3. My holiday starts in six day’s time.
her free time reading and listening to
Yesterday was my birth day –Friday june
classical music. Both Mr. Baba and
Mrs. Baba like to cook. 23rd.On which day of the week will my holiday
Mr. Baba enjoys planting and start?
A. growing vegetables.
Mr. Baba does not like classical A. Fri.june 30th B. Wed.30th June
B. music. C. Tues.29th june D. Sat.26th
Mrs. Baba cooks the vegetables july. ANS A
C. that Mr. Baba grows.
Mrs. Baba enjoys reading 4. Yemi was not at school at Friday last. She
D. nineteenth century novels. was first absent three days before that .Today
is Monday 31st may.When was yemi first
absent? Give the day and date.
Answer: Option A
A. Tues.25th May B. Wed. 29 April
C. Mon. 1st June D. Mon. 27
60 Bade is older than her cousin Shade. April ANS A
Shade's brother Sule is older than
Bade. When Shade and Sule are 5. Mary and Jane are both clever. Jean and
visiting with Bade, all three like to play Jane are tall. Mary and Jean are dark. My
a game of Monopoly. Shade wins
friend is tall and clever. Who is she?
more often than Bade does.
When he plays Monopoly with A. Mary B. Jane C. Jean and
Shade and Bade, Sule often
Jane D. Mary and Jane ANS B
A. loses.
Of the three, Bade is the 6. Audu is taller than Laide but shorter than
B. oldest.
Deji. Deji is of the same height as Taiwo.Who
Bade hates to lose at
C. Monopoly. is the shortest?
Of the three,Shade is the
D. youngest. A. Audu B. Laide C. Deji
D. Taiwo ANS B

7. Bisi is older than Ore ,Dele is older than


Answer: Option D
Bisi.Who is the youngest of all?

101
A. None B.Bisi C. Ore 12. Gloves are to hands as shoes are to:
D.Dele. ANS C
A. Boxers B. Hands
C. feet D. Legs ANS
C

8. Abayo, Tope, Femi and Seyi are friends. Choose the correct word to complete the
Only Abayo likes going to the theatre.Only analogies.
Tope and Seyi like the cinema. Only Femi and
Seyi like dancing. Which boy likes neither 13. Up is to down as before is to:
dancing nor cinema? A. later B. after C. now
A.Tope B.Seyi C.Abayo D. next ANS
B
D.Femi. ANS C

9. I do not like grapes or bananas but I have Choose the correct word to complete the
bought some fruits to eat. Do you think it is analogies.
grapes, bananas, both or neither? 14. Sew is to needle as dig is to:
A. Grapes B. Bananas A. garden B. plant C.
C. Both D. Neither ANS poke D. spade ANS
D D
10. There is a boat with a ladder attached to it Choose the correct word to complete the
and the ladder is eight metres tall if the water analogies.
rises four metres how much of the ladder will
be on to of the water? 15. School is to pupils, as army is to:

A. 12 metres B. 48 metres A. teachers B. chiefs


C. 8 metres D. 10 metres C.soldiers D. barracks
ANS C ANS D

11.I thought the time was ten minutes to two Choose the correct word to complete the
,but I was looking at the clock in a mirror. analogies.
What time was it really?
16. Feather is to bird as fin is to:
A. ten minutes to two B. ten
A. finish B. fish C.
minutes after two C.ten to ten
crab D. finies ANS
D. Ten to three.
B
ANS B
Choose the correct word to complete the
analogies.
ANALOGIES AND ASSOCIATIONS.
17.Tall is to short as broad is to:
Choose the correct word to complete the
A. wide B. long C. straight
analogies.
D. narrow ANS
D
102
Choose the correct word to complete the A. rug B.lino C. floor
analogies. D. room ANS
C
18.Orange is to peel as nut is to :
Choose from the alternatives, the word
A. kernel B. rind C. shell which is missing from the pairs
D. fruit ANS
A 24. Bees and ----Pigs and sty:

Choose the correct word to complete the A. hive B. kernel C. Hole


analogies. D. Sty ANS
A
19. Hour is to time as kilometre is to:
Choose from the alternatives, the word
A. Metre B. Day C. Speed
which is missing from the pairs
D. Distance ANS
D 25. Chop is to steak as --- is to mutton and :

Choose the correct word to complete the A. cutlet B. ham C. pork


analogies. D. veal ANS
A
20. Water and ship, air and:
Choose from the alternatives, the word
A. Breathing B. Flying C. Aeroplane D. Airfield
which is missing from theANS
pairs C
Choose the correct word to complete the
26. Hold and hand, Kneel and:
analogies.
A. Ankle B. Foot C. Heel
21. Poem is to a poet, as --- is to a novelist: D. Knee ANS
A. writer B. author C. story D
D. novel ANS
Choose from the alternatives, the word
D which is missing from the pairs

27. Bed and mattress, chair and:


Choose from the alternatives, the word A. Table B. Pillow C. Cushion
which is missing from the pairs D. Stool ANS
22. Merchant and selling ,artist and: C

A. picture B. painting C. paints Choose from the alternatives, the word


D. brushes ANS which is missing from the pairs
B 28. Knife is to cutting as pen is to:
Choose from the alternatives, the word A. pencil B. paper C. fork
which is missing from the pairs D. writing ANS
23. Paper and wall, carpet and: D

103
Choose from the alternatives, the word Choose a word which has the same or nearly
which is missing from the pairs the same meaning as the first word.

29. Trumpet is to blow as gong is to: 34. Permanent:

A. bellow B. strike C. pluck A. temporary B. old C. lasting D.


D. pull ANS complete ANS
B C

Choose a word which has the same or nearly


the same meaning as the first word.
SYNONYMS
35.Boss:
Choose a word which has the same or nearly
the same meaning as the first word. A. Governor B. Servant C. Mistress
D. master ANS
30 Feeble: D
A. Strong B. weak C. Tired Choose a word which has the same or nearly
D. Little ANS the same meaning as the first word.
B
36. Dear:
Choose a word which has the same or nearly
the same meaning as the first word. A. precious B. cheap C. nice
D. useless ANS
31. Astonish: A
A. asunder B. attack C. surprise Choose a word which has the same or nearly
D. Sensible ANS the same meaning as the first word.
C
37. Elegance:
Choose a word which has the same or nearly
the same meaning as the first word. A. grace B. display C. safety
D. ugly ANS
32. miserable: A
A. Mischief B. wretched C. mistake Choose a word which has the same or nearly
D. wicked ANS the same meaning as the first word.
B
38.Thankfulness:
Choose a word which has the same or nearly
the same meaning as the first word. A. greatness B. gratitude C. meanness
D. goodness ANS
33. stubborn: B
A. obstinate B. sturdy C. Fearful Choose a word which has the same or nearly
D. strong ANS the same meaning as the first word.
A
39. Study:

104
A. Learn B. Enjoy C. Teach 45. He is as industrious as :
D. Remember ANS
A A. an ant B. elephant C. worker
D. messenger ANS
Choose a word which has the same or nearly A
the same meaning as the first word.
Complete the following similes:
40.Prohibit:
46. As merciless as a:
A. advertise B. allow C. forbid
D. destroy ANS A. grave B. grasshopper C.
C Catastrophe D. Catapult
ANS A
Choose a word which has the same or nearly
the same meaning as the first word. Complete the following similes:

41. Excitement: 47.As brittle as:

A. Enthusiasm B. Zeal C. Fuss A. pure water B. glass C. bintu


D. Success ANS D.wood ANS
B
A

SIMILES Complete the following similes:

Complete the following similes: 48. As beautiful as :

A. rainbow B. brass C. peacock


42.Her gown is as green as:
D. lamb ANS
A. grace B. grass C. flag A
D. butter ANS
B

Complete the following similes: ANTONYMS

43. Olu is as hungry as a: Choose a word opposite in meaning to the


first word
A. orphan B. wolf C. termites
D. Thug ANS 49.Above :
B A. over B. below C. high D.low
Complete the following similes: ANS B

44. He is as mischievous as: Choose a word opposite in meaning to the


first word
A. satan B. monkey C. gazelle
D. tortoise ANS 50.liberty :
B A.freedom B.captivity C.stadium D.guilty
Complete the following similes: ANS B

105
Choose a word opposite in meaning to the 57.A promotional discount of 15% is offered
first word on a new coat which previously cost #18,000.
What is the discounted price of the coat?
51.Dissent:
A.#15,300 B. #14,000 C. #15,500 D.
A. dissent B. undissent C. Assent #16,000 ANS A
D.indissent ANS C
58. Seun ‘s friend was knock down by a car,
Choose a word opposite in meaning to the
he tried to copy the car plate number which
first word reads
52.Sharp : HA539KST. Which one of the following is
A. clever B.blunt C. Keen D.even a correct copy of the car plate number?
ANS B A. HA 359KST B. HA 593KST C. HA
Choose a word opposite in meaning to the 539KTS D. HA 539KST. ANS
first word D

53. Conductor : 59. Danladi is to transfer some money into


the account of his friend which reads
A. driver B. conductress C.
Passenger D. Wire ANS C 040000111906. Which one of the
following is a correct copy of the account
Choose a word opposite in meaning to the number of his friend?
first word
A. 040000111960. B. 040000111906. C.
54. Spendthrift : 04000111906. D. 04000011906. ANS
B
A. miser B. loan C. Foolish D.
happy ANS A 60. Bible is to a priest as ........to a don

Choose a word opposite in meaning to the A.qur’an B. books C. music D. drum


first word ANS
B
55. Profane :
70. Football is to a footballer as ........to a
A. beautiful B. sacred C. Profuse
cyclist
D. Stiff. ANS B
A.rider B. motorbike C. bicycle D.
motorcycle ANS
56. A team of eight lumberjacks cut an C
average of 15,000 cubic feet of timber in a
71. Ade is to arrange 5S55ES in a reverse
week. How many cu;bic feet will four
orders . Which one of the following is the
lumberjacks cut in four weeks?
correct
A.30,000 B. 25,000 C. 32,000 D. 16,000 .
arrangement?
ANS A

106
A. 5S55SE B. SE55S5 C. ESE55S5 D. A. HA 359KST B. KST 593HA C. HA
ES555S ANS 539KTS D. TSK935AH. ANS
B D

72. She was the first woman to stand 78..Judge is to a Court as a teacher is to
..........election to parliament.

A. to B. for C. through D. by • class room

ANS • bank
B • school
• hospital
73. They waited in a state of feverish anxiety
..........their mother to come home.
ANS A
A. till B. to C. until D. for
79.The basic needs of man are
ANS
• Money, children and friends
D
• Food, shelter and clothing
74. If John starts work at 8.845 am and
• Education, money and happiness
finishes at 5.15pm. He has 90 minutes of
• Power, authority and position
break. How many hours

does he work in 5 days? ANS B

A.38 B. 39 C. 35 D. 40 80.Petroleum is to Port Harcourt as coal is to


ANS • Enugu
C • Kaduna
• Kebbi
75. A restaurant bill is made up of the • Ibadan
following: #12.50 for starters, #28.55 for main
courses and #8.95 ANS A

for deserts, plus a #17.50 service charge. 81.Which of these is used for making fabrics
How much is the bill? • Oil
• Timber
A.#56.50 B. #57.50 C. #57.00 D.
59.50 ANS B • Coal
• Cotton
77. Dapo was knock down by a car with plate
number HA539KST. The police on duty picked
ANS D
the
82.The First Prime Minister of Nigeria was
car plate number in a reverse order
• Alhaji Nuhu Bamali
as..............
• Alhaji Tafawa Balewa
• Sir Akanu Ibaim

107
• Dr Nnamdi Azikwe • 1959
• 1950
ANS B
83.Substances that remove dirt by absorbing ANS C
the grease that fixes the dirt are known as 88.Headquarters of NYSC is at ________
• Dirt removers • Abuja
• Dirt absorbers • Aba
• Grease absorbers • Kano
• Cleaning agents • Lagos

ANS C ANS A
• Green is a 89.The Nigeria Civil War started on --------------
• Primary colour ------
• Secondary colour • 3rd October, 1970
• Tertiary colour • 6th July 1967
th
• Complimentary colour • 16 July 1967
st
• 1 April 1976
ANS B
85. Blue is a ANS B
• Primary colour
• Secondary colour
• Tertiary colour 90.Taiwo Akinkunmi designed the Nigerian
• Complimentary colour Flag in
• 1888
ANS A • 1977
• Sir Richard’s constitution was introduced • 1976
in Nigeria in • 1958
• 1946
• 1950 ANS D
• 1964
• 1864 91.Nigeria has ---------------- geo-political zones
• 5
ANS A • 6
87.The Central Bank of Nigeria was • 7
established in • 10
• 1920
• 1908 ANS B

108
92.Alhaji Shehu Shagari became Nigerian First 96.Which of the following is linked with
Executive President CONFLUENCE
st
• 1 October 1960 • Niger State
st
• 1 October 1961 • Kogi State
th
• 7 June 1985 • Benue State
st
• 1 October 1979 • None of above

ANS D ANS B
97.A patient suffering from Night blindness
requires
• Vitamin A
93.Nigeria started using N200 notes on
st • Vitamin E
• 1 Feb, 2000
th • Cacium only
• 5 May, 2001
th • Iodine
• 20 August, 2000
• 1st Nov. 2000
ANS A
98.Zebra crossing is for ---------------
ANS D
• Playground
• Easy transport action of Zebra
94.Queen Elizabeth visited Nigeria for the
• Safety of Pedestrians
second time on
rd • Zoological garden
• 3 Dec. 2003
• 2nd October. 1980
ANS C
• 7th June 1985
99.The capital of Kastina State is ---------------
• 14th June 1979
• Kastina
• Gusau
ANS A
• Dutse
95.The International organization responsible
• Ilorin
for taking care of the injured is known as
…………
ANS A
• Red Cross Society
100.The capital of Cameroon is
• Yauonde
• UNESCO
• Cairo
• World Bank
• Lome
• Man O’ War
• Conakry

ANS A
ANS A

109
101.The first man to get to the space is • Central Bank of Nigeria
• Yuri Gagarim • Progress Bank Nigeria Plc
• Gay Lussac
• Dalton ANS C
• Sir Isaac Newton 106.Birmingham is in the
• U.S.A
ANS A • U.K
102.The capital of Benin Republic is -------------- • Ireland
• Kinshasa • Italy
• Algeria
• Niamey ANS B
• Port Novo 107.Which of these doctor treats pregnant women
• Orthopedic doctor
ANS D • Gynaecologist
103.Goitre is caused by the deficiency of • Neuro-Surgeon
• Iron • Opthamologist
• Calcium
• Iodine ANS B
• Chlorine
108.A number of bees living in the same place
ANS C • swarm
• hive
104.A crossed cheque does not allow cash
• team
payment • gaggle
• Over the counter
• In savings account ANS B
109.A number of people listening to a concert
• With thumb printing
a) congregation
• None of the above b) spectators
c) audience
ANS A d )crowd
ANS
C
110.A number of singers in a church
a) troupe
b) choir
c) entertainers
105.Which of the following is owned by the
d) artists
Federal government ANS
• Union Bank Nigeria Plc B
• UBA Plc
112.A number of sailors manning a ship
a) crew
110
b) gang d) granary
c) staff ANS D
d) host 119. Abuja is to Nigeria as _______ is to
ANS Canada
A a)Winnipeg
b)Toronto
c) Ottawa
d)Montreal
113. A number of directors of a company ANS C
a) panel
b) jury 120.Which of the following is not a primary
c) bench reason why many people hold cash?
d) board a) to undertake transactions
ANS b) to avoid credit
D c) to have an emergency reserve
114. A number of persons of the same race d) to have a store of value
and character ANS B
a) gang
b) tribe 121.Which alternative is an example of an
c) associates intangible asset?
• fellow a)a common stock
b)a bond
ANS B c)a mutual fund share
115. A place where milk is converted into d)all of the above
butter and cheese ANS D
a) dairy 122.A place where money is coined
b) bakery a)mint
c) distillery b)factory
d) refinery c)bank
ANS A d)Central Bank
116. A place for housing aeroplanes ANS
a) tarmac A
b) garage 123.A place where fruit trees are grown
c) hangar a) farm
d) airport b) courtyard
ANS C c) orchard
117. A place where government records are d)backyard
kept ANS C
a) library 124.An institution for the reformation of
b) archive young offenders
c) government house a) prison
d) museum b) orphanage
ANS B c) hostel
118. A place for storing grain d)reformatory
a) depot ANS
b) store D
c) bans
111
125.The art of conducting negotiations b)goulash
between nations c)cutlet
a)armistice d)simmers
b)diplomacy ANS A
c)amnesty 132.A plane figure with ten sides and ten
d)demobilise angles
ANS a)hexagon
B b)octagon
126.A book of names and addresses c)decagon
a)diary d)pentagon
b)dictionary ANS C
c)album 133.Living on fish makes one
d)directory a)piscivorous
ANS D b)carnivorous
127.Which of the following does not c)graminivorous
belong? d)omnivorous
a)encyclopaedia ANS
b)catalogue A
c)contraband 134.Goods found floating after a
d)bibliography shipwreck are called
ANS a)jetsam
C b)careen
128.A statement which is accepted as true c)flotsam
without proof d)argosy
a)axiom ANS
b)epitome C
c)caption 135.A man is 30 years older than his
d)excerpt daughter. If the sum of their ages is 48
ANS A years, what is the daughter’s age?
129.The history of the life of a person
a)autograph a)18 years (b) 9 years (c) 8
b)autobiography years (d) 12 years ANS
c)biography B
d)comedy
ANS B 136. 8 divided by a number is 5 more
than the value of 3 divided by the
130.A soup made with meat, fish or number, what is the number.
vegetables
a)baste a)4 (b) 7 (c) 1 (d) 3
b)braise
c)bisque ANS C
d)broth
ANS 137.Digestion of food starts from the
D mouth with enzyme ----------------------
131.A stew of meat, especially mutton and -in the saliva.
vegetables
a)haricot
112
a)Amylase (b) ptyalin (c) 143.Animals that feed on both plant
Zymase (d) Resin and flesh are called -----------------------
ANS B ---------

138.The liquid part of the blood is a)Herbivores (b) Carnivores (c)


called ------------------------- Omnivores (d) Perovores
ANS C
a)Plasma (b) Platelets (c)
Red blood cells (d) White 144.Scurvy is a disease caused by lack
blood cells ANS A of which vitamins?

139.Which of the following is not a a)A b) B c) C d)E


disease of the blood?
ANS C
a)Leukaemia (b) Sickle cell (c)
Haemophilia (d) Anaemia 145.Collection of stars is called --------
ANS D -------------------

140.-----------------is a part of dentition a)Stardom b) Galaxy c)


used for cutting, biting and tearing Astronomy d) Meteorology
ANS B
a)Incisors (b) Canines (c)
Premolars (d) Molars 146.The non-resident head of a
ANS B University is -------------------------

141.ICT a)Vice-chancellor b)
means…………………………… Chancellor c) Provost d)
Registrar ANS B
a)Information and Communication
Theology
147.An area of land almost surrounded
b)Information and Communicate by water is known as -------------------------
Technology --
c)Information and Communication a) Lake b) Lagoon c)
Technology Island d) Peninsula ANS
d)Infolink and Communication D
Technology 148.Solve 1/4 of 20/7
ANS C
a)3/7 b) 7/5 c)
142.Perspiration is to animals while -- 5/7 d) 2/7
------------------------ is to plants ANS C
a)Transpiration (b) Excretion (c) 149.The cost of 2 biros and 3 pencils is
Transportation (d) Evaporation N18, if a pencil cost N2.50k, how
ANS A much is a biro?
113
a)N4.25k b) N3.20k c) a)Light intensity b) Water c)
N4.00 d) N5.25K Temperature d) Pressure
ANS D ANS D

150.Solve 21% of 70 156.Which of the following is not a


class of food?
a)29.4 b) 14.7 c) 13.9 d)
7.4 ANS a)Carbohydrates b) Water c)
B Vegetables d) Vitamins
ANS C
151.Wale and Joke have ratio 6:4 in 80
units of Transcorp’s shares. How many 157 In how many ways can a six
units of these shares belong to Wale? applicant for a job arrange themselves at
around the table when they are invited
a)48 b) 40 c) 32 for an interview?
d) 24
ANS A a)102 ways b) 6 ways c) 12
ways d) 120 ways
152.The sum of and 2/3 and 1/3 is
ANS D
what?
158. How many local governments do
a)1/3 b) 2/3 c) 1 we have in Nigeria?
d) 3
ANS C a)700 b) 720 c) 574
d) 774
153.A trader had 19 lanterns in her ANS D
store, sold 13 and bought 6 more. How
many lanterns does she left in her store? 159.NCC means ---------------------------

a)6 b) 9 c) 12 a)Nitel Communication


d) 4 Commission
ANS C
b)Nigerian Communication Centre
154.----------------can be defined as the
movement of water molecules across a c)Nigeria Communication Centre
semi-permeable anterns does she has d)Nigerian Communication
left in her store? Commission
a)Plasmolysis b) Diffusion c) ANS D
Osmosis d) Transpiration 160.Which of the following is referred
ANS C to as the power house of the cells
155.Which of the following factors a)Nucleus b) Mitochondria c)
could not affect photosynthesis Cytoplasm d) Ribosome
ANS B

114
161.Starch could be found in all of • Gavel is an instrument used by -----
these except -------------------------- --------------

a)Yam b) Rice c) Wheat d) a)Welder b) Surgeon


Palm oil c) Judge d) Bricklayer
ANS D ANS C

162.The organism that cause 64) Which of these diseases is Insect


MALARIA is ----------------------- borne?

a)Mosquitoes b) Plasmodium c) a) Sleeping sickness b)


Tse Tse Fly d) Butterfly Whooping cough c) Measles
ANS B d) Chicken pox ANS A

163.Which of these is NOT a fruit? 65) The digestive tract is also referred
to as………….. Canal.
a)Tomatoes b) Orange c)
Pepper d) Pawpaw a) Abdominal b)
ANS C Alimentary c) Lymphatic
d) Intestinal ANS B
• The sum of interior angle of a
triangle is equal to ------------------- 66) The period after childhood and
before adulthood is referred to
a)900 b)1800 c) as………………
2700 d) 3600
ANS B a) Infancy b)
Adolescence c) Puberty
• The first President of Nigeria is ---- d) Maturity ANS B
----------------------------
68) Which of the following belongs to
a)Obafemi Awolowo b) Olusegun
the electronic media?
Obasanjo c) Nnamdi Azikwe d)
Tafawa Balewa ANS B a) Novel b) Road
sign c) Magazine
• The Nigerian flag has how many d) Television ANS D
colour(s)
69) On the Nigeria’s Coat of Arm, the
a)3 b) 1 black shield stands for
c) 4 d) 2 ANS
D a) Fertile soil b)
Strength c) Honour
• The Nigeria flag was designed by - d) Dignity ANS
----------------------- A
a)Akinkunmi Alabi b) Kola 70) Which of the following is NOT a
Olawuyi c) Taiwo Akinkunmi factor of production?
d) Tai Solarin ANS
C
115
a) Labour b) Land many bicycles will it produce in 8
c) Capital d) Factory
ANS D working days?

72) The amount of blood pumped per a) 120 b) 640


minute in the body is called -----------------
c) 480
----

a) Pulse rate b) Cardiac d) 960


arrest c) Cardiac output
ANS D
d) Stroke volume
ANS A 78) Which of the following gases is
73) Which of the following required for rusting?
organization performs FIRST AID
activities? a) Helium b) Hydrogen

a) WHO b) UNESCO c) Oxygen


c) NAFDAC d) REDCROSS
ANS D d) Argon

74) Who presides over the House of ANS C


Representative in Nigeria?
80) Reflected sound is called-----------
a) Chief Whip b) President
c) Presiding Officer d) -------------
Speaker ANS D
a) Echo b) Noise
75) What prefix can be added to the
word “manage” from the list below? c) Wave

a) Non b) Un d) Mirage
c) Mis d) Dis
ANS A
ANS C
81) A mixture of sand and salt can be
76) Animals store carbohydrate
mainly as separated based on the difference

a) Fats b) Glycogen in their……


c) Minerals d)
a) Magnetism b) Colour
Starch ANS B

77) A bicycle factory produces 600 c) Solubility

bicycles in 5 working days. How d) Size

ANS C
116
82) Which of the following trait is

NOT genetically transmitted? ANS D

a) Eye Colour b) Hair Style 91) All these parts are involved in

c) Fatness d) breathing EXCPECT

Complexion ANS B a) Diaphragm b) Mouth

c) Nose d) Ribs

83) Steel is an alloy of ------------------ ANS

---- B

a) Iron and Carbon b) Iron and 92) Meteorologists are scientist who

Silicon c) Calcium and Silicon specialise in the study of ----------

d) Iron and Tin ANS ---------

A a) Sun b)

87) Which of the following is NOT a Hydrosphere c) Atmosphere

sense organ? d) Weather

a) Hair b) Ear ANS D

c) Eye 95) The ability of living things to

d) Skin respond to stimuli is described as-

ANS A -----------------

90) The weather is cooler at Jos a) Locomotion b)

compare with Lagos because Jos- Respiration c) Growth

----------- d) Irritability

a) Has less sunshine ANS D

b) Has less rainfall 96) Which of these forest products is

c) Has more rocks used for the manufacture of

d) Is on a higher ground paper?


117
a) Wood Pulp b) Gum
3. The gas predominantly responsible
c) Flower for global warming is

d) Fruit (a) Carbon dioxide


(b) Carbon monoxide
ANS A (c) Nitrous oxide
(d) Nitrogen peroxide
98) Every participant ----------------- ANS A

reported for the opening 4. Which of the following uses non-


conventional source of energy?
ceremony
(a) Kerosene lamp
a) Have b) Was (b) Wax candle
(c) Solar lantern
(d) Touch
c) Had
ANS C
d) Has
5. Electric current is measured by
ANS D
(a) Voltmeter
(b) Anemometer
100) A die is thrown once, what is the (c) Commentator
(d) Ammeter
probability that the number that ANS D
turns up is a multiple of two? 6. The hardest substance available on
earth is
a) 5/6 b) 2/3
c) 1/6 d) 1/2 (a) Platinum
ANS D (b) Diamond
(c) Quartz
(d) Gold
ANS A
1. Some green are blue, no blue is white.
7. 2008 Olympic games was held in
(a) Some green are white
(b) No white is green (a) Ghana
(c) No green is white (b) Greece
(d) None of the above (c) Nigeria
ANS A (d) France
ANS A
2. I have trouble
8. The Olympic games are normally held
(a) to remember my password at an interval of
(b) to remembering my password
(c) remember my password (a) 2years
(d) remembering my password (b) 3years
ANS D (c) 4years

118
(d) 5years (d) Internal National Electoral
ANS C Commission ANS A

9. For one to be eligible to vote in 14. The solar eclipse occurs when
Nigeria, one must be (a) the sun comes in between the
moon and the earth
(a) 18years (b) the earth comes in between
(b) 19years or above the sun and the moon
(c) Minimum of 18years (c) the moon comes in between
(d) Between 18years and 45years the sun and the earth
ANS C (d) None of the above
ANS C
10. All electioneering campaigns during
the time of elections are stopped 15. The removal of top soil by water or
wind is called
(a) 48 hrs before the appointed
time of election results (a) Soil wash
(b) 48 hrs before the actual poll (b) Soil erosion
(c) 24 hrs before the actual poll (c) Soil creep
(d) 36 hrs before the actual poll (d) Silting of soil
ANS C ANS B

11. Sellers market denotes a situation


where 16. Who invented the telephone?

(a) Commodities are available at (a) Thomas Alva Edison


competitive rates (b) Galileo
(b) Demand exceeds supply (c) Alexander Graham Bell
(c) Supply exceeds demand (d) Thomas Graham Bell
(d) Supply and demand are equal ANS C
ANS B
17. Albert Einsten was a famous
12. Development means economic
growth plus (a) Physician
(b) Chemist
(a) Inflation (c) Physicist
(b) Deflation (d) Biologist
(c) Price stability ANS C
(d) Social change
ANS D 18. In banking, ATM stands for

13. INEC stands for (a) Automated Tallying Machine


(b) Automatic Teller Machine
(a) Independent National (c) Automated Totaling Machine
Electoral Commission (d) Automated Transaction of
(b) Independent Nigerian Money ANS B
Elections Commission
(c) Independent National Electric 19. Which of the following stands for
Commission organization related to space in Nigeria?

119
(a) NARSDA
(b) NAFDAC 25. Persons with which blood group are
(c) NYSC called universal donors?
(d) ARMTI ANS
A (a) AB
(b) A
20. Goitre is caused by the deficiency of (c) O
(d) B
(a) Vitamin D ANS C
(b) Iron
(c) Vitamin A 26. Thermometer is related to degree on
(d) Iodine ANS the same way as clock is to
D
(a) Wall
21. Which of these is not a mosquito (b) Tower
borne disease? (c) Hour
(d) Cock
(a) Dengue fever ANS C
(b) Malaria
(c) Filariasis 27. The headquarters of United Nations is
(d) Goitre ANS located at
D
(a) Geneva
22. Oranges are rich in (b) New York
(c) Rome
(a) Carbohydrates (d) Washington
(b) Fats ANS B
(c) Proteins
(d) Vitamins ANS 28. To work on mobile cell phone, which
D of the following is required?

23. AIDS stands for (a) Favourable handset


(b) Sim card
(a) Acquired Immune Disease (c) Service Provider Network
Syndrome (d) All of the above ANS
(b) Acquired Immunity Deficiency C
Syndrome
(c) Acquired Immune Deficiency 29. Which of the following keys of
Syndrome Personal Computer is not available in the
(d) Acquired Infection Deficiency keyboard of traditional typewriter?
Syndrome ANS C
(a) Tab
24. Stem cuttings are commonly used for (b) Space bar
re-growing (c) Enter
(d) Backspace ANS
(a) Cotton C
(b) Banana
(c) Rice 30. Which of the following is not related
(d) Sugar cane to the functioning of internet?
ANS C
120
(a) www
(b) http (a) shall have been married for
(c) e-mail twenty-five years
(d) All of the above ANS (b) will have been married for
D twenty-five years
(c) will be married for twenty-
31. Mrs James has lived ……………. five five years
year (d) will have married for twenty-
five years ANS D
(a) while
(b) for 37. The first part of the test was on
(c) since biology, the second on physics, and the third
(d) during ANS B on literature
32. The fact …………… money orders can
usually be easily cashed has made them (a) on physics; and the third on
popular form of payment literature
(b) on physics: and the third on
(a) that literature
(b) is that (c) on physics and the third on
(c) which is literature
(d) is ANS (d) on physics, and the third on
A literature ANS D

33. We …………………. that concert 38. Online shopping can be convenient


and time saving with appropriate precautions,
(a) were disappointing in it is safe as well
(b) were disappointing
(c) were disappointed of (a) should be convenient and
(d) were disappointed by time saving
ANS D (b) could be convenient and time
saving
34. When did Bayo …………. college? (c) can be convenient and time
saving
(a) graduate from (d) can be convenient and time-
(b) graduate saving ANS D
(c) graduating from
(d) graduating 39. After it was repaired, it ran perfect
ANS A again
(a) would run perfectly
35. You look pale, you had better (b) could or run perfect
…………….. a doctor (c) ran perfectly
(d) ran perfect
(a) consult with ANS C
(b) consult to
(c) consult 40. My parent often ……………. after
(d) consult by dinner
ANS C
(a) walking
36. In August , my parents will be married (b) go a walk
for twenty-five years (c) take a work

121
(d) go for a work ANS 46. During colonial days a school room
D looked rather empty

41. Every child has to ………………. and (a) colonial days, a school room
respect other cultures looked
(b) colonial Days, a school room
(a) be acquainted with looked
(b) get acquainted with (c) colonial Days; a school room
(c) acquainted with looked
(d) getting acquainted with (d) colonialdays, a school room
ANS A looked ANS A

42. Such news is always ……………… to be 47. noun . Restraint; framework; raised
true edge of a side walk

(a) so good (a) prediction


(b) very good (b) curb
(c) too good (c) equanimity
(d) good ANS (d) lien ANS
C B

43. You should buy a computer ………….. 48. adjective . Frightened; quivering
you would be able to send e-mails
(a) congenial
(a) then (b) succulent
(b) well (c) tremulous
(c) therefore (d) craven
(d) so ANS ANS C
A
49. adjective . unfeeling; insensitive; hard
44. If the books have been catalogued
last week, why haven’t they been placed on (a) callous
the shelf? (b) labyrinthine
(c) effete
(a) have on the shelf (d) radical
(b) would have been catalogued ANS A
(c) had been catalogued
(d) were catalogued 50. ………………… getting the highest result
ANS D in the class, John still had problems with the
teacher
45. Linda decides they had better scram
before the killers find them (a) Nonetheless
(b) Despite
(a) should hurry and scram (c) Even though
(b) had better leave (d) In spite of
(c) could hurry and leave ANS D
(d) had better scram
ANS B 51. A gifted scientist, Newton ………….
Some of the most fundamental law in the
history of science

122
(a) Clumsy
(a) discovered (b) Obstacle
(b) keep discovering (c) Quit
(c) who discovered (d) Protest
(d) the discoverer of ANS D
ANS A
58. Sentence is to Paragraph as Word is
52. …………….. the variable drops by a unit to
of 1, the rank drops by x amount
(a) Letter
(a) What (b) Sentence
(b) Whenever (c) Paragraph
(c) How (d) Phase
(d) Why ANS ANS A
B
59. Which is the odd one out?
53. Ocean is to Pond as Deep is to
(a) Seminal
(a) Shallow (b) Germane
(b) Well (C) Propagative
(c) Sea (d) Derivative
(d) Lake ANS ANS C
B
60. Never is to seldom as Always is to
54. What does Frozen mean?
(a) Occasional
(a) Glued (b) Usual
(b) Liquid (c) Often
(c) Solid (d) Every time
(d) Water ANS ANS C
C
61. Bolt is to Nut as Hook is to
55. What means the same as Portion?
(a) Screw
(a) Whole (b) Eye
(b) Part (c) Door
(c) Chip (d) Grass ANS
(d) None ANS A
B
62. What is an Edifice?
56. Wise is to Foolish as Vein is to
(a) Building
(a) Modest (b) Illness
(b) Pretty (c) Boatyard
(c) Conceit (d) Executive ANS
(d) Proud ANS A
D
63. Irascible is the opposite of
57. Acquiesce in the opposite of
(a) Striking

123
(b) Flamboyant
(c) Irritable 69. Which of the following should appear
(d) Phlegmatic ANS first alphabetical?
D
(a) Able
64. Which is the odd one out? (b) Abel
(c) Abbel
(a) Marriage (d) Ably ANS
(b) Annul C
(c) Divorce
(d) Separation 70. Sue and Jennifer are fair, Brian and
ANS B Robyn are dark, Sue and Robyn are tall, who is
fair and tall?
65. Stint is the opposite of
(a) Sue
(a) Surfeit (b) Jennifer
(b) Prolix (c) Brian
(c) Meticulous (d) Robyn ANS
(d) Splint A
ANS C
66. Direct is to Tortuous as Dilatory is to 71. Sue and Jennifer are fair, Brian and
Robyn are dark, Sue and Robyn are tall, who is
(a) Expeditious tall and dark?
(b) Straight
(c) Curved (a) Sue
(d) Circumlocutory (b) Jennifer
ANS A (c) Brian
(d) Robyn ANS D
67. God should take control of the heart
of the organizers of this Post-UME screening
exercise, they should not make the 72. Which of the following numbers is
test …………….. than UME divisible by 3: (i) 541326 (ii) 5967013?

(a) more tough (a) (ii) only


(b) more tougher (b) (i) only
(c) much tougher (c) (i) & (ii)
(d) more much tougher (d) None
ANS C ANS B

68. The teacher took me for one of those 73. A man owns 2/3 of the market
students who could spell such words as research business and sells ¾ of his shares for
N75,000. What is the value of business
(a) ‘miscelanous and ?
maintainance’
(b) ‘miscelaneous and (a) N150,000
meintainance’ (b) N13,000
(c) ‘miscellaneous and (c) N240,000
maintenance’ (d) N34,000
(d) ‘miscellanous and ANS A
maintainance’ ANS C
124
74. Which of the following numbers (d) 16 ANS
should be added to 11158 to make it exactly C
divisible by 77?
80. The radius of a right circular cone is
(a) 9 30cm and its height is 4cm, the curved surface
(b) 8 of the cone will be
(c) 7
(d) 5 (a) 12sq.cm
ANS C (b) 15sq.cm
(c) 18sq.cm
75. On selling 3 articles at the cost of 4 (d) 21sq.cm
articles, there will be profit of ANS B

(a) 25% 81. The fractions 7/11; 16/20; 21/22


(b) 100/3% when arranged in descending order is
(c) 75/2%
(d) 40% (a) 7/11; 16/20; 21/22
ANS B (b) 21/22; 7/11; 16/20
(c) 21/22; 16/20; 7/11
76. By selling an article for N40, there is a (d) 7/11; 21/22; 16/20
loss of 40%, by selling it for N80, there would ANS C
be
82. If 10% of a umber is subtracted from
(a) Gain of 20% it, result is 1800. The number is
(b) Loss of 10%
(c) Loss of 20% (a) 1900
(d) Gain of 10% (b) 2000
ANS A (c) 2100
(d) 2140
77. A number consists of 2 digits whose ANS B
sum is 8, if 8 is subtracted from the number,
the digits interchanges their place. The 83. The number proceeding 9909 which is
number is a perfect square is

(a) 44 (a) 9908


(b) 35 (b) 9900
(c) 62 (c) 9899
(d) 33 (d) 9801 ANS
ANS D D

84. Find the root value of 36.1/102.4?


79. The sides of a triangle are in the ratio
of 3:5:7 and its perimeter is 30cm. The length (a) 61/34
of the greatest side of the (b) 19/31
triangle in cm is (c) 19/32
(d) 19/33
(a) 6 ANS C
(b) 10
(c) 14 85. A shop keeper sold a T.V. set for
N17,940 with a discount of 8% and earned a

125
profit of 19.6%, what would have been 90. A radio when sold at a certain price
the percentage of profit earned if no discount gives a gain of 20%, what will be the gain
was offered? percent if sold for 3ce the price?

(a) 24.8% (a) 280


(b) 25% (b) 270
(c) 26.4% (c) 290
(d) None of the above (d) 260
ANS D ANS D

86. If (2x-y) = 4, then (6x-3y) = ? 91. If the value of x lies between 0 and 1,
which of the following is the largest?
(a) 15
(b) 12 (a) x
(c) 18 (b) x2
(d) 10 (c) -x
ANS B (d) 1/x
ANS D
87. A clock is set at 8am. The clock gains
10min in 24hrs. What will be the true time 92. Thirty men take 20 days to complete
when the clock indicates 1pm on the a job working for 9hrs a day, how many hr(s) a
following day? day should 40 men to work to complete
the job?
(a) 48 minutes past 12
(b) 38 minutes past 12 (a) 8
(c) 28 minutes past 12 (b) 7.5
(d) 25 minutes past 12 (c) 7
ANS A (d) 9
ANS B
88. A person has 4 coins each of different
denomination, what is the number of 93. In a single throw of a dice, what is the
different sums of money the person probability of getting a number greater than
can form (using one or more coins at a time)? 4?

(a) 16 (a) 1/2


(b) 15 (b) 2/3
(c) 12 (c) 1/4
(d) 11 (d) 1/3
ANS B ANS D

89. Which number is the odd one out of 94. 1, 3, 5 ,7 , 9, ?


9678, 4572, 5261, 3527, 7768?
(a) 8
(a) 9678 (b) 11
(b) 4572 (c) 12
(c) 5261 (d) 13
(d) 3527 ANS B
ANS D
95. 2, 2, 4, 6, 10 ,?

126
(a) 14 decades. Which of the following is
(b) 10 true?
(c) 20
(d) 16 • Southern Sudan is now an
ANS D Independent Nation

96. 0.1, 0.3, 0.5, 0.7, 0.9, ? • Southern Sudan is not yet an
Independent nation
(a) 1.1
(b) 0.11 • Southern Sudan has been
(c) 11 struggling for Independence for
(d) 0.011 over 20 years
ANS A
• Southern Sudan and
97. 0, 3, 8, 15, 24, ?
Independence are fighting side by
(a) 36 side. ANS
(b) 30 C
(c) 35
(d) 31
ANS C
• During the 2008 POSTJAMB
98. 4, 5, 7, 11, 19, ? registration exercise which lasted 15
days, the number of candidates on
(a) 33 the first day was 2000 but doubled
(b) 27 daily in the next 4 days. Thereafter, it
(c) 37 was 250 per day till the end. How
(d) 35
many candidates registered for the
ANS D
exercise?
99. 2, 5, 11, 23, 47, ?
• 10 500
(a) 80 • 12 500
(b) 95
(c) 92 • 15 500
(d) 101
ANS B • 20 500
ANS B
100. 50, 40, 100, 90, 150, ?

(a) 200
(b) 180 • What is the missing figure in : twenty
(c) 140 and 6, fifty and 5, four and 4, and
(d) 300
sixteen and …..?
ANS C
• 5

• Southern Sudan fought for • 6


Independence for more than two
• 7

127
• 8 • Jide did better than Jude in
ANS C Physics and Chemistry but both
had same score in Biology.

• Jane was better than the boys in


• Tope had 30 kittens. After a bout of Biology and Chemistry but had
coccidiosis all but 13 died. How many same score as Jide in Chemistry.
kittens are left?
• All three had A grades in the three
• 30 subjects.
• 43 • 1 and 4
• 17 • 2 and 3
• 13 • 2 and 4
ANS D
• 3 and 4
ANS D
• Divide 500 by half and then add 50.
What do you have?
• All crabs like water. Most crabs like
• 550 fresh water. I saw three crabs with
• 550.5 Agnes. Which of the following is true?

• 1050 • Crabs live in water

• 1050.5 • The three crabs with Agnes are


ANS C from fresh water

• Some crabs cannot survive in sea


water
• Jide, Jude and Jane were the best
three students in Physics, Chemistry • Agnes has three crabs
and Biology. ANS C

The best student had an average of


72%. • What is the missing number in :
Which two pieces of information can twenty and three, fifty and two and a
be correct as to who the best half, four and two, and thirty nine
candidate was? and………?

• Jude scored less than 72% in only • Four


Biology • Five

• Four and a half

128
• Five and a half • 100 ANS
ANS B B

• Most Lecturers are corrupt. Joe is a • Nigerian Policemen are generally


Lecturer. Which of the following is far corrupt.
from being correct?
Most Policemen working in cities are
• Joe may be in the category of corrupt.
corrupt lecturers
Gbenga’s brother is a policeman.
• Joe, being a lecturer, is probably
corrupt Which of the following is true?

• Although Joe is a lecturer, he may • Gbenga’s brother is corrupt.


not be corrupt • Gbenga’s lives in the city.
• Joe is a good lecturer because he • Gbenga’s brother is probably
is not corrupt
corrupt and lives in the city.
ANS D
• Gbenga’s is a Nigerian Policeman.
ANS
• Dare earns twice as much as Ojo. If D
Ojo earns N250 per hour as a
Supervisor, how much will Dare earn
if each of them works only five hours • If Ope’s score of 300 in JAMB is one
a day and 20 days in a month? and a half times better than that of
Shuaib, what will the score of Olu be
• N500
if his is twice that of Shuaib?
• N25 000
• 150
• N50 000
• 200
• N75 000 ANS • 300
C
• 400
ANS D
• What is the missing number in:
twenty and ten, fifty and……, sixteen
and eight, and ninety and forty five? • Climate Change is caused by which of
the following?
• 50
• Increased global precipitation
• 25
• Increased global temperature
• 20
129
• Increased global desertification

• Increased global pollution • If in the English alphabet, A = 1, B = 2,


ANS C = 3…… and Z = 26,
B
what will be sum of all the vowels?

A. 5
• MTN charges 25K per second for calls
B. 15
and GLO 18K, if Ade called for 75
seconds on MTN and Lizzie 75 C. 51
seconds on GLO, how much less
would Lizzie have paid? D. 7
ANS C
• N8.75

• N3.50
• If in the English alphabet, A = 1, B = 2,
• N5.20 C = 3…… and Z = 26,
• N5.25 what will be sum of all the
ANS D consonants?

• 351
• Which of these colours will absorb • 325
more heat?
• 300
• Black
• 24 ANS
• Blue C
• Red

• White • If in the English alphabet, A = 1, B = 2,


ANS A C = 3…… and Z = 26,

what will be sum of all the alphabets?


• Which of the following countries has • 26
the largest land mass?
• 300
• Niger
• 325
• Nigeria
• 351 ANS
• Ghana
D
• South Africa
ANS A

130
• If in the English alphabet, A = 1, B = 2, • What is the missing figure, twenty
C = 3…… and Z = 26, and 100, four and 20, ten and …….,
and thirty and 150?
what will be the difference sum of the
consonants and that of the vowels? • 50

• 351 • 25

• 300 • 75

• 249 • 100 ANS


A
• 16 ANS
C • If in the English alphabet, A = 1, B = 2,
C = 3…… and Z = 26,

what will be the product of the sum of


• If in the English alphabet, A = 1, B = 2,
CAT and DOG?
C = 3…… and Z = 26,
• 3120
what will be sum of the alphabets L
and M? • 4157

• 25 • 624

• 26 • 600
ANS C
• 156

• 236 ANS
A • If in the English alphabet, A = 1, B = 2,
C = 3…… and Z = 26,

what will be difference in the sum of


• If in the English alphabet, A = 1, B = 2, DOG and CAT?
C = 3…… and Z = 26,
• 26
what will be product of the alphabets
E and Y? • 24

• 20 • 2

• 30 • 1 ANS
C
• 125

• 150 ANS
C • If in the English alphabet, A = 1, B = 2,
C = 3…… and Z = 26,

131
what will be sum of the alphabets in discount of 5%, how much did the
the word MATRICULATION? buyer pay?

• 154 • N19 905

• 144 • N19 800

• 134 • N19 600

• 124 ANS • N19 400 ANS


B C

• If in the English alphabet, A = 1, B = 2, • Bayo’s average score in Mathematics,


C = 3…… and Z = 26, Physics, chemistry and Biology was
67%. However when his score in
what will be sum of the alphabets in History was added the average rose
the word POSTJAMB? to 71%. What was his score in
• 96 History?

• 86 • 67%

• 106 • 71%

• 76 ANS • 77%
A • 87%
ANS D

• If 25 bricklayers will spend 5 days to


lay 25 000 blocks, how many days will • What is an ARCHIPELAGO?
40 bricklayers take to lay 32 000
blocks? • An expanse of land surrounded by
water
• 3
• An expanse of water surrounded
• 4 land
• 5 • An expanse of water with many
scattered islands
• 8 ANS
B • An expanse of land with many
scattered ponds
ANS C
• Bunmi sold a suit in her mother’s
shop with a price tag of N20 000 at a

132
• Which is the world’s largest • Orange and Yellow
archipelago? ANS B

• British Isles

• Canadian Arctic Archipelago • Which of these numbers is the odd


one out?
• Japanese Archipelago
• 4561
• Malay Archipelago
ANS • 3724
D
• 9142

• 6434
• Where is British Isles located? ANS D

• Atlantic Ocean

• Arctic Ocean • The cost of 20 pieces of tomato each


weighing 150g is N240, what will be
• Indian Ocean the cost of 30 pieces if each weighs
• Pacific ocean 120g?
ANS • N240
A
• N300

• N288
• Where is the RING OF FIRE located?
• N388
• Baltic Sea ANS C
• Dead Sea

• Indian Ocean • A well digger was trying to rescue a


• Pacific Ocean dog from a well 30 feet deep using a
ANS D bucket. Each time he pulls up the rope
3 feet, it slides back 2 feet. How many
times will he have to pull up the rope
to bring the dog to the surface?
• The two colours seen at the upper
part of the rainbow are • 90

• Indigo and Violet • 60

• Red and Orange • 30

• Green and Blue • 6


ANS C
133
• Bacteria are useful so should be
handled carefully.
• Timi scored 90% in History
examination. His friend Ade scored • Bacteria are dangerous so must
two thirds of Timi’s score while the be handled with care.
cousin Gunte scored 70% of Timi’s. ANS B
What is their average score?

• 61%
• A thermometer reading 32 C was kept
• 71% in boiling water for five minutes.
What would be the temperature
• 75.5%
reading?
• 66%
• 32 C
ANS B
• 100 C

• 160 C
• In the mid-term test, Yaq scored 72%
in Mathematics, his brother scored • 144 C
2/3 of his mark and their niece twice ANS B
the score of Yaq’s brother. What was
the score of the niece?

• 72% • Chief Awolowo, Dr. Azikwe, Chief


Enahoro and Mal Aminu Kano are
• 100% examples of our past………..

• 96% • Heros

• 76% ANS • Heroes


C
• Heroins

• Heroines ANS
• Some bacteria are beneficial but most B
are dangerous if carelessly handled. A
lot cause untold losses and hardships
but some can be used for mankind. • A tanker discharged 33 000 litres of
Which of the following is correct? kerosene into the underground tanks
• Bacteria should be carefully of AP Petrol Station. After selling half
handled because they are of it, NNPC brought a pump to
dangerous. withdraw the balance. If the pump
siphons at the rate of 250 litres per
• Even if carefully handled bacteria minute, how long will it take to empty
can still be dangerous. the underground tank if only 2500
litres is to be left?
134
• 36 Minutes • If Bolt runs 100 meters in 10 seconds,
what will be his speed per hour?
• 46 Minutes
• 10 Km per hour
• 56 Minutes
• 3.6 Km per hour
• 66 Minutes ANS
C • 1 Km per hour

• 10.6 Km per hour ANS


B
• If the average of Clem in three
subjects is 67.5% and his scores in
Mathematics and Physics were 54 and
87 % respectively, what was his score • If in registering for WAEC, only 1500
candidates registered on the first day
in the third subject?
but twice the number registered
• 61.5% subsequently till the sixth day, how
many candidates would have
• 56.5% registered?
• 65.5% • 10 000
• 55.5% ANS • 15 0000
A
• 15 500

• 16 500 ANS
• During the 2010 PostJamb
C
registration exercise which lasted 15
days the number of candidates
registering doubled every day in the
last five days but was constant in the • Driving on Nigerian roads predispose
first 10 days. If the number of one to accidents. Akeem is a careless
candidates at the beginning was 740, driver but has never had an accident.
what was the number after the A bus was involved in an accident on a
Nigerian road. Which of the following
registration?
is true?
• 326 000
• Akeem must have been involved
• 236 000 in the accident.

• 218 000 • The bus could not have been


Akeem’s.
• 118 000 ANS
B • Akeem can never be involved in
accident.

135
• Akeem is an experienced driver.
ANS B (a) to remember my password
(b) to remembering my password
(c) remember my password
(d) remembering my password
• Quartz clocks never stop ticking. My ANS: D
daddy’s clock stopped suddenly.
Which of the following is true? 3. The gas predominantly responsible
for global warming is
• All other clocks are not reliable.
(a) Carbon dioxide
• My daddy’s clock is not quartz. (b) Carbon monoxide
(c) Nitrous oxide
(d) Nitrogen peroxide
• My daddy does not like quartz ANS: A
clocks
4. Which of the following uses non-
• Quarz clocks keep correct time. conventional source of energy?
ANS B
(a) Kerosene lamp
(b) Wax candle
(c) Solar lantern
• Most Nigerian Politicians are either (d) Torch
corrupt or arrogant. ANS: C

Bisi has been a Politician for 30 years. 5. Electric current is measured by


Therefore
(a) Voltmeter
• Bisi is corrupt. (b) Anemometer
(c) Current
• Bisi is arrogant (d) Ammeter
ANS: D
• Bisi has always been both corrupt
and arrogant 6. The hardest substance available on
earth is
• Bisi is over three decades old.
ANS (a) Platinum
(b) Diamond
D
(c) Quartz
(d) Gold
ANS: A
1. Some green are blue, no blue is white.
7. 2008 Olympic games was held in
(a) Some green are white
(b) No white is green (a) Ghana
(c) No green is white (b) Greece
(d) None of the above (c) Nigeria
ANS: A (d) France
ANS: A
2. I have trouble

136
8. The Olympic games are normally held (a) Independent National
at an interval of Electoral Commission
(b) Independent Nigerian
(a) 2years Elections Commission
(b) 3years (c) Independent National Electric
(c) 4years Commission
(d) 5years (d) Internal National Electoral
ANS: C Commission ANS: A

9. For one to be eligible to vote in 14. The solar eclipse occurs when
Nigeria, one must be (a) the sun comes in between the
moon and the earth
(a) 18years (b) the earth comes in between
(b) 19years or above the sun and the moon
(c) Minimum of 18years (c) the moon comes in between
(d) Between 18years and 45years the sun and the earth
ANS: C (d) None of the above
ANS: C
10. All electioneering campaigns during
the time of elections are stopped 15. The removal of top soil by water or
wind is called
(a) 48 hrs before the appointed
time of election results (a) Soil wash
(b) 48 hrs before the actual poll (b) Soil erosion
(c) 24 hrs before the actual poll (c) Soil creep
(d) 36 hrs before the actual poll (d) Silting of soil
ANS: C ANS: B

11. Sellers market denotes a situation 16. Who invented the telephone?
where
(a) Thomas Alva Edison
(a) Commodities are available at (b) Galileo
competitive rates (c) Alexander Graham Bell
(b) Demand exceeds supply (d) Thomas Graham Bell
(c) Supply exceeds demand ANS: C
(d) Supply and demand are equal
ANS: B 17. Albert Einsten was a famous

12. Development means economic (a) Physician


growth plus (b) Chemist
(c) Physicist
(a) Inflation (d) Biologist
(b) Deflation ANS: C
(c) Price stability
(d) Social change 18. In banking, ATM stands for
ANS: D
(a) Automated Tallying Machine
13. INEC stands for (b) Automatic Teller Machine
(c) Automated Totaling Machine

137
(d) Automated Transaction of 24. Stem cuttings are commonly used for
Money ANS: B re-growing

19. Which of the following stands for (a) Cotton


organization related to space in Nigeria? (b) Banana
(c) Rice
(a) NARSDA (d) Sugar cane
(b) NAFDAC ANS:
(c) NYSC D
(d) ARMTI
ANS: A 25. Persons with which blood group are
called universal donors?
20. Goitre is caused by the deficiency of
(a) AB
(a) Vitamin D (b) A
(b) Iron (c) O
(c) Vitamin A (d) B
(d) Iodine ANS:
ANS: D C

21. Which of these is not a mosquito 26. Thermometer is related to Degree on


borne disease? the same way as Clock is to

(a) Dengue fever (a) Wall


(b) Malaria (b) Tower
(c) Filariasis (c) Hour
(d) Insomnia (d) Cock
ANS: D ANS:
C
22. Oranges are rich in
27. The headquarters of United Nations is
(a) Carbohydrates located at
(b) Fats
(c) Proteins (a) Geneva
(d) Vitamins (b) New York
ANS: D (c) Rome
(d) Washington
23. AIDS stands for ANS:
B
(a) Acquired Immune Disease
Syndrome 28. To work on mobile cell phone, which
(b) Acquired Immunity Deficiency of the following is required?
Syndrome
(c) Acquired Immune Deficiency (a) Favourable handset
Syndrome (b) Sim card
(d) Acquired Infection Deficiency (c) Service Provider Network
Syndrome ANS: (d) All of the above
C ANS: D

138
29. Which of the following keys of 74. Which of the following numbers
Personal Computer is not available in the should be added to 11158 to make it exactly
keyboard of traditional typewriter? divisible by 77?

(a) Tab (a) 9


(b) Space bar (b) 8
(c) Enter (c) 7
(d) Backspace (d) 5
ANS: ANS: C
C
76. By selling an article for N40, there is a
30. Which of the following is not related loss of 40%, by selling it for N80, there would
to the functioning of internet? be

(a) www (a) Gain of 20%


(b) http (b) Loss of 10%
(c) e-mail (c) Loss of 20%
(d) none of the above (d) Gain of 10%
ANS: ANS: A
D
82. If 10% of a umber is subtracted from
it, result is 1800. The number is
70. Sue and Jennifer are fair, Brian and
Robyn are dark, Sue and Robyn are tall, who is (a) 1900
fair and tall? (b) 2000
(c) 2100
(a) Sue (d) 2140
(b) Jennifer ANS: B
(c) Brian
(d) Robyn 85. A shop keeper sold a T.V. set for
ANS: A N17,940 with a discount of 8% and earned a
72. Which of the following numbers is profit of 19.6%, what would have been
divisible by 3: (i) 541326 (ii) 5967013? the percentage of profit earned if no discount
was offered?
(a) (ii) only
(b) (i) only (a) 24.8%
(c) (i) & (ii) (b) 25%
(d) None (c) 26.4%
ANS: B (d) None of the above
ANS: D
73. A man owns 2/3 of the market
research business and sells ¾ of his shares for 87. A clock is set at 8am. The clock gains
N75,000. What is the value of business 10min in 24hrs. What will be the true time
? when the clock indicates 1pm on the
following day?
(a) N150,000
(b) N13,000 (a) 48 minutes past 12
(c) N240,000 (b) 38 minutes past 12
(d) N34,000 (c) 28 minutes past 12
ANS: A

139
(d) 25 minutes past 12 (d) 1/3
ANS: A ANS: D

89. Which number is the odd one out of 94. 1, 3, 5 ,7 , 9, ?


9678, 4572, 5261, 3527, 7768?
(a) 8
(a) 9678 (b) 11
(b) 4572 (c) 12
(c) 5261 (d) 13
(d) 3527 ANS: B
ANS: D
95. 2, 2, 4, 6, 10 ,?
90. A radio when sold at a certain price
gives a gain of 20%, what will be the gain (a) 14
percent if sold for 3ce the price? (b) 10
(c) 20
(a) 280 (d) 16
(b) 270 ANS: D
(c) 290
(d) 260 96. 0.1, 0.3, 0.5, 0.7, 0.9, ?
ANS: D
(a) 1.1
91. If the value of x lies between 0 and 1, (b) 0.11
which of the following is the largest? (c) 11
(d) 0.011
(a) x ANS: A
(b) x2
(c) -x 97. 0, 3, 8, 15, 24, ?
(d) 1/x
ANS: D (a) 36
(b) 30
92. Thirty men take 20 days to complete (c) 35
a job working for 9hrs a day, how many hr(s) a (d) 31
day should 40 men to work to complete ANS: C
the job?
98. 4, 5, 7, 11, 19, ?
(a) 8
(b) 7.5 (a) 33
(c) 7 (b) 27
(d) 9 (c) 37
ANS: B (d) 35
ANS: D
93. In a single throw of a dice, what is the
probability of getting a number greater than 99. 2, 5, 11, 23, 47, ?
4?
(a) 80
(a) 1/2 (b) 95
(b) 2/3 (c) 92
(c) 1/4 (d) 101
ANS: B

140
(a) 1/8, 3/7, 5/9 and 1/3 (b) 3/7, 1/8,
100. 50, 40, 100, 90, 150, ? 5/9 and 1/3 ( c) 1/8, 3/7, 5/9 and 1/34

(a) 200 (d) 1/8, 1/3, 3/7 and 5/9


(b) 180 ANS: D
(c) 140
(d) 300 6. Arrange the following in descending order
ANS: C
2/3, 4/7, 5/9 and 1/4

(a) 2/3, 4/7, 5/9 and 1/4 (b) 1/4, 5/9,


• Add all the even numbers from 1 to 8. 4/7 and 2/3 ( c) 1/4, 4/7, 5/9 and 2/3
• 19 (d) 1/4, 2/3, 4/7 and 5/9
ANS: A
• 23
7. What is Roman numeral of 50?
• 20
(a) L (b) X (c) C (d) D
• 16
ANS: A
ANS C
8. What is Roman numeral of 19?
• Add all the odd numbers between 2
and 10. (a) XX (b) IX ( c)XIX (d) D
ANS: C
• 22
9. What is Roman numeral of 60?
• 25
(a) XL (b)LX ( c) CX (d) XD
• 24
ANS: B
• 19
10. What is Roman numeral of 1000?
ANS C
(a) D (b) L (c) C (d) M
• Add all the even numbers from 15 to
ANS: D
20
11. What is Arabic figure of Roman figure IC?
(a) 55 (b) 34 ( c)54 (d) 64
ANS: C (a) 49 (b) 99 (c) 101 (d) 89
ANS: B
4. Which of the following fractions is the
highest? 12. What is Arabic figure of Roman figure ID?
(a) 3/4 (b) 7/9 ( c) 2/3 (d) 12/15 (a) 499 (b) 299 (c) 401 (d) 489
ANS: D ANS: A
5. Arrange the following in ascending 13. What is Arabic figure of Roman figure VC?
order
(a) 99 (b) 95 (c) 105 (d) 500
1/8, 3/7, 5/9 and 1/3 ANS: B

141
15. Subtract 6289 from 4389 24. A plane leaves town B at 09.50 hours and
arrives town C at 19.40hours. If the time at C
(a) 1900 (b) 1800 (c)-1900 (d) -1800 is one hour ahead of time B how long does
ANS: C the flight take?

(a) 8hrs 45mins (b) 8hrs 50mins (c) 7hrs


18. How many seconds do have in a day? 55mins (d) 7hrs 45mins ANS: B

(a) 3600sec (b) 86.4x 102 sec (c) 86.4x 103 25 Find the HCF of 36 and 60
sec (d) 864sec ANS: C (a) 3 (b) 6 (c) 12 (d) 36
19. How many seconds make one week? ANS:
C
(a) 86.4x 104secs (b) 604.8x 103secs (c)
60.48103secs (d) 6864secs ANS:
B 33. Reduce to its lowest term.

20. How many hours and minutes are there


from 7.45am to 3.30p.m.?
(a) (b) (c) (d)
(a) 6hrs. 45mins. (b) 8hrs 30mins (c) 7hrs ANS:
45mins (d) 7hrs 30mins ANS: C C

21. How long does a train take to cover the


journey if it starts at 6.10a.m. and ends at 51. Express in a decimal form.
3.55p.m. the same day?
(a).075 (b) .75 (c) 7.50 (d) .0075
(a)10hrs 35mins (b) 9hrs. 45mins (c) 9hrs ANS:
30mins (d) 10hrs 45mins ANS: B
B
60. A teacher earns #15,000 a month. He
22. A clock gains 20 seconds every hour. It spends #9000 and saves the remaining
shows the correct time at 6 a.m. What time amount. What is the ratio of his savings to the
will it show at 3p.m.? amount he spends?

(a) 3.03p.m. (b) 3.06p.m. (c) 2.57p.m. (d) (a)3:2 (b)2:3 (c) 3:5(d) 5:3
3.13p.m. ANS: ANS:
A B

65. A square has

23. Find the number of days from 12th June (a) 2 of its sides are equal (b) all it sides
noon to 12th September noon. equal (c) non of its sides equal (d)3 of its
sides equal
(a) 90days (b) 92days (c) 61days (d) 62days
ANS:
B ANS:
B

142
72. If 6 men can dig a well in 3 days, how 119. Which of the following fraction is the
many men will dig it in 2 days at the same smallest?
rate?

(a)9men (b) 12men (c) 16men (d) 15men (a) (b) (c) (d)
ANS:
A ANS:

81. A trader bought a good for #20 and sold C


it for #20. What was the gain percent?
120. Which of the following fraction is the
(a)100% (b)10% (c)1% (d) 0%
highest?
ANS: D

83. Some goods were sold for #11,000 at a (a) (b) (c) (d)
profit of 10% of the cost price. What did the
goods cost? ANS:

(a) #10,000 (b) #11,100 (c) #1,200 (d) A

#13,200 ANS: 122. Divide 15015 by 15.

A (a)11 (b) 15 (c) 1001 (d) 105

ANS:
94. How many 40K can we get in #16,000?
C
(a) 400 (b)300 (c)200 (d)100
ANS:
A
2. A special place where animals are
110. Express two Million, two hundred slaughtered or killed particularly for
thousand and 50 in figures. commercial
(a)2200050 (b) 2020050 (c) 2205000 purposes is called a/an -----------------------
------------------------------------------------------
(d)2020050 ANS: A
(a) shrine (b)killing center
111. If 6 men can do a piece of work in 2
days, how long will it 2 men? (c)backyard (d)abattoir ANS:

(a) 4days (b) 6days (c) 5days D

(d)12days 4. Which of the following is not an ethnic


group in Nigeria?
ANS: B
(a) Kanuri (b) Tiv (c)Edo (d)Ewe
ANS:

143
5. A leader who does not listen to people’s 22. Nigeria’s major source of income is----------
opinion before taking decisions is-------- ----------------------------------------

(a)monarch (b)hardworking (c)democratic (a) income tax (b) school fees (c)cocoa
(d)autocratic ANS: D
(d)petroleum ANS: D
7. Which of these does not belong to the
digestive system? 26. Which of these is needed for building
healthy teeth?
(a) mouth (b)stomach (c)lung (d)intestine
ANS: C (a) sweets (b)starch (c)calcium (d)vitamin

11. ------------------------ is not a rainbow colour. ANS: C

(a) red (b)white (c)orange (d)yellow 27. Which part of pineapple do we generally
plant?
ANS: B
(a)the leaf (b)the tube (c)the sucker
12. Which of these conditions is necessary for
germination of plant? (d)the root ANS: C

(a) water (b)fertile soil (c)air (d)all of the 28. The chief raw material for making soap is--
-------------------------------------
above ANS: D
(a) cocoa (b) onion (c)palm oil (d)coffee
14. What do we call the air we breathe in? ANS: C

(a) oxygen (b) carbon dioxide(c)smoke 32. Money paid on goods brought into a
country in known as…………………….
(d)hydrogen ANS: A
(a)airport tax (b)excise duty (c)sales tax
16. The following insects are useful for the
(d)import tax ANS: B
farmer except------------------
33. Which of the following diseases require
(a) cricket (b) butterfly (c)bee (d)dragon fly the patient to be isolated?
ANS: D (a) tape worm (b)malaria (c)small pox
(d)dysentery ANS: C
17. Which of these is not a method of
purifying water? 35. In which of the following towns can we
find the Cocoa House?
(a) boiling (b) addition alum (c)sieving
(a) Kano (b)Ife (c)Onitsha (d)Ibadan
(d)addition of salt ANS: D
ANS: D
19. All these are water borne diseases except 36. -------------------- is the process by which the
(a) guinea worm (b) diarrhoea (c)Cholera food is broken down in the body.

(d) malaria ANS: D

144
(a) eating system (b)constipation (a) Goods and services produced in a the
(c)digestion (d)respiration ANS: country
C
(b) Manufacture outside the country
41. For a paper kite to fly high, it needs
…………………………………… (c) Goods produced for sale in the country

(a) fire (b)fuel (c)air (d)water (d) Goods produced for sale outside the
ANS: C country ANS: D

42. Compatriots in the first stanza of the 69. The sun is the major source of ----------------
Nigeria National Anthem refers to………. ---- and ----------------------------

(a)soldiers (b)Nigerians (c)companies (a)power and heat (b)light and heat


(d)foreigners ANS: B (c)strength and heat

43. Braille is used by --------------------------------- (d)light and darkness


-------------------------------------- ANS: B

(a)illiterate people (b)blind people (c)short 73. A female dog is called a/an ………….
sighted people (d)old people ANS: B (a)kitten (b)bitch (c)ewe (d) damp
44. What is the similarity between a tree and ANS: B
a fly? 74. The Armed Forces of Nigeria comprise of
(a)they are friends (b)both are living ……………………..
things(c)they like each other (a)the Army, Custom, police
(d)both can talk (b)Navy, Immigration, Air-force
ANS: B
(c)the police, Air-force, Navy (d)Army, Air-
45. Which side of the chest is the heart force, Navy ANS: D
normarlly located?
75. A man who mends shoes is known as
(a)left (b)right (c)center (d)part …………………….
ANS:
(a)leather maker (b)shoe shiner (c)cobbler
46. Ikeja is to Lagos as ----------- is to Akwa- (d)artist ANS: C
Ibom
78. Complete the pair appropriately.
(a)Calabar (b)Mecca (c)Uyo (d)Ibandan
ANS: C Car and accident, airplane and ----------------
-------
47. Which of these is active in the night?
(a)stop (b) crash (c) hangar (d)wings
(a)bird (b)squirrel (c)bat (d)lizard ANS: B
ANS: C
85. The first set of teeth which later replaced
64. Export goods are…………….. by permanent teeth is……………..

145
(a)milk teeth (b)abnormal teeth (c)good (a)spider (b)termite (c)goat (d)moth
teeth (d)incisors teeth ANS: A ANS: C

88.The ----------------- is referred to as the 116. From which part of Nigeria can we get
pumping station of an animal. groundnut in large amounts?

(a)brain (b)lung (c)heart (d)kidney (a)Southern (b)Northern (c)Western


ANS: C (d)Eastern ANS: B

89. The bone of the brain is covered by the---- 117. Which of these is responsible for strong
-------------------- bone in the body?

(a)skull (b)scull (c)tibia (d)scapular (a)sulphur (b)chlorine (c)calcium and


ANS: A phosphorus(d)potassium ANS: C

90. Digestion of food starts from the ------------ 120. Whose picture is shown on 100 naira
----------------- note?

(a)mouth (b)anus (c)stomach (d)gullet (a)Tafawa Balewa (b)Obafemi Awolowo


ANS: A (c)Nnadi Azikwe

91. The missionary that stopped the killing of (d)Herbert Macauly


twins in Nigeria is named……………….. ANS: B

(a)Mungo Pack (b)Mary Slessor 124. The first woman to drive a car in Nigeria
is --------------------------------
(c)Funmilayo Ransome Kuti(d)Queen
Elizabeth ANS: B (a)Mrs. Dorah Akuyili (b) Mrs Funmilayo
Ransome Kuti
92. The shape of the earth is -----------------------
------------------- (c) Mrs. Elam (d) Mrs. Juliana Coker
ANS: B
(a)round (b)cubic (c)spherical (d)flat
ANS: C 125. The Nigeria female football Team is
called ---------------------
112. What is the name given to the green
colouring matter of a leaf? (a)Flying eagles (b)the falcons (c)golden
eagles
(a)stigma (b)chlorophyll (c)segment
(d)colouring ANS: B (d)Oladimeji queen
ANS: B
114. Which of these diseases is caused by a
mosquito? 127. Whose picture is shown on the five naira
note?
(a)tuberculosis (b)diabetics (c)cholera
(d)dysentary ANS: C (a) Sir Tafawa balewa (b) Chief Anthony
Enahoro
115. Which of the following is not an insect?
(c) Sir, Arther Richards (d)Sir Ahmadu
Bello ANS: C
146
128. The male part of a flower is called---------- Select the lettered pair that best expresses a
---------------------- relationship similar to that expressed in the
original pair
(a)pistil (b)corolla (c)stigma (d)stamen • PRUNE: HEDGE
ANS: A • Reap: crop
• Shave: mustache
144. Cement is made from………………… • Trim: hair
• Shuck: corn
(a)columbite (b)iron-ore (c)limestone
(d)marble ANS: C ANS: C
Select the lettered pair that best expresses a
146. The line that divided the word into two is relationship similar to that expressed in the
the …………………………. original pair
• KIDNEY: ORGAN
(a)Equator (b)longitude (c)latitude • Rib: bone
(d)pole ANS: A • Aorta: blood
• Stomach: abdomen
Select the lettered pair that best expresses a • Kneecap: skeleton
relationship similar to that expressed in the
ANS: A
original pair.
Select the lettered pair that best expresses a
• RECORDING: SOUND relationship similar to that expressed in the
• Concert: sono original pair
• Script: scene • GAVEL: JUDGE
• Photograph: light • Whistle: referee
• Film: negative • Insionia: soldier
• Lecturer: podium
ANS: C • Uniform: detective
Select the lettered pair that best expresses a
relationship similar to that expressed in the ANS: A
original pair Select the lettered pair that best expresses a
• COAGULANT: BLEEDING relationship similar to that expressed in the
• Stimulant: relaxation original pair
• Antibiotic: infection • UNIFORM: SECURITY AGENT
• Enzyme: digestion • gentle: heart
• Narcotic: dependency • SIM Card: mobile phone
• Pressing Iron: desk
ANS: B • Diluted: gas
Select the lettered pair that best expresses a
relationship similar to that expressed in the ANS: B
original pair Select the lettered pair that best expresses a
• PASTOR: CHURCH relationship similar to that expressed in the
• Book: bedroom original pair
• Kitchen: television • ARBOREAL: TREES
• Teacher: market • Sidereal: stars
• Imam: Mosque • Herbaceous: plants
• Terrestrial: plains
ANS: D • Sleep: listen

ANS: A

147
Select the lettered pair that best expresses a Select the lettered pair that best expresses a
relationship similar to that expressed in the relationship similar to that expressed in the
original pair original pair
• FREEZER: FREEZE • ANTIBODIES: PROTECT
• Theatre: stadium • Facts: saturate
• Water: whether • Circulate: fluid
• Black: red • Degenerate: cells
• Warmer: warm • Food: nourish

ANS: D ANS: D
Select the lettered pair that best expresses a
relationship similar to that expressed in the Select the lettered pair that best expresses a
original pair relationship similar to that expressed in the
• SYMPHONY: SCORE original pair
• Play: Script • LICENSE: DRIVER
• Lecturer: manifesto • handcuffs: officer
• Story: collection • Certification: Teacher
• Song: refrain • Book: librarian
• Mechanic: tool
ANS: A
Select the lettered pair that best expresses a ANS: B
relationship similar to that expressed in the Select the lettered pair that best expresses a
original pair relationship similar to that expressed in the
• HAIR: TOUPEE original pair
• Thumb: hand • ADDICTION: REHABILITATE
• Eye: eyebrow • Imperfection: belittle
• Teeth: denture • Error: disabuse
• Wound: bandage • Discern: discriminate
• Reputation: discredit
ANS: C
Select the lettered pair that best expresses a ANS: B
relationship similar to that expressed in the Select the lettered pair that best expresses a
original pair relationship similar to that expressed in the
• PENTICOSTAL: GENERAL original pair
• Choir: mosque • DEHYDRATE: WATER
• Priest: catholic • Detoxify: poison
• Quaranic School: Imam • Destabilise: deviance
• Classroom: food • Dissolve: liquid
• Determine: certainty
ANS: B
Select the lettered pair that best expresses a ANS: A
relationship similar to that expressed in the Select the lettered pair that best expresses a
original pair relationship similar to that expressed in the
• PRODIGALITY: GENEROSITY original pair
• humility: honour • GUST: WIND
• Compassion: contempt • Surf: sea
• Courage: rashness • Rapids: river
• Spirit: mettle • Cloudburst: rainfall

ANS: C

148
• Mist: fog • Compute: estimate

ANS: C ANS: C
Select the lettered pair that best expresses a Select the lettered pair that best expresses a
relationship similar to that expressed in the relationship similar to that expressed in the
original pair original pair
• CORFON: CROWD • GOUGING: PRICE
• Level: river • Usury: interest
• Castle: moat • Monopoly: production
• Corona: sun • Embezzlement: saving
• Flower: petal • Fraud: property

ANS: A ANS: A
Select the lettered pair that best expresses a Select the lettered pair that best expresses a
relationship similar to that expressed in the relationship similar to that expressed in the
original pair original pair
• SUGAR: SACCHARIN • MOVIE: SCREEN
• Photograph: painting • Play: stage
• Butter: margarine • Frame: portrait
• Signal: whistle • Performance: score
• Music: tape • Shadow: object

ANS: B ANS: A
Select the lettered pair that best expresses a Select the lettered pair that best expresses a
relationship similar to that expressed in the relationship similar to that expressed in the
original pair original pair
• NECK: MUFFLER 26. CONDUCTOR: ORCHESTRA
• Suspenders: trouser
• Collar: blouse a. Philosopher: inspiration
• Sandal: foot
• Hand: mitten b. Actor: Scene

ANS: D c. Surveyor: landscape


Select the lettered pair that best expresses a
d. Pilot: Ship
relationship similar to that expressed in the
original pair
• MEDIUM: ORGANISM ANS: D
• Bacteria: germ
• Soil: plants Chose the lettered word or phrase that is
• Growth: Sample most nearly opposite in meaning to
• Liquid: nutrient the words in capital letters

ANS: B 27. ALLIEN


Select the lettered pair that best expresses a
relationship similar to that expressed in the a. Pleasing
original pair
• MULTIPLY: DIVIDE b. Disgusting
• Computer: estimate
• Enumerate: court c. Indigenous
• Enter: leaves

149
d. Irate • Prospect
ANS: C • Attack
ANS: B
Chose the lettered word or phrase that is Chose the lettered word or phrase that is
most nearly opposite in meaning to most nearly opposite in meaning to the
the words in capital letters words in capital letters

28. EXEMPLARY • MARRIED


• Couple
a. Foreign • Divorced
• Single
b. Imitate • Searching
ANS: C
c. Additional Chose the lettered word or phrase that is
most nearly opposite in meaning to the
d. Outstanding
words in capital letters
ANS: D
• FELL
Chose the lettered word or phrase that is
• Cruel
most nearly opposite in meaning to • Official
the words in capital letters • Risen
• Downcast
29. ABUNDANT ANS: C
Chose the lettered word or phrase that is
a. Surplus
most nearly opposite in meaning to the
b. Few words in capital letters

c. Brilliance • EDUCATED
• Literate
d. Birth • Teacher
ANS: B • Medical Practitioner
• Illiterate
Chose the lettered word or phrase that is ANS: D
most nearly opposite in meaning to Chose the lettered word or phrase that is
the words in capital letters most nearly opposite in meaning to the
words in capital letters
• GGRAVATE
• Ameliorate • BOY
• Scrape • Female
• Improve • Girl
• Harden • Male
ANS: A • Feminine
Chose the lettered word or phrase that is ANS: B
most nearly opposite in meaning to the Chose the lettered word or phrase that is
words in capital letters most nearly opposite in meaning to the
words in capital letters
• PROBLEM
• Headache • QUIESCENT
• Solution • Gyrate

150
• Exuberant • Uninteresting
• Cursory • Elasticity
• Turbulent • Endeavour
Chose the lettered word or phrase that is • Dormant
most nearly opposite in meaning to the ANS: D
words in capital letters Chose the lettered word or phrase that is
most nearly opposite in meaning to the
• INDIFFERENCE words in capital letters
• Affordable
• Ignorant • RIGID
• Zeal • Account
• Applaud • Flexible
ANS: C • Oracle
Chose the lettered word or phrase that is • Irritable
most nearly opposite in meaning to the ANS: B
words in capital letters Chose the lettered word or phrase that is
most nearly opposite in meaning to the
• HERETIC words in capital letters
• Stoic
• Cleric • ARROGANT
• Orthodox • Humble
• Verbal • Intolerable
ANS: C • Covetous
Chose the lettered word or phrase that is • Concomitant
most nearly opposite in meaning to the ANS: A
words in capital letters

• JUDICIOUS
• Imprudent Chose the lettered word or phrase that is
• Obeissance most nearly opposite in meaning to the
• Wasteful words in capital letters
• Volition
ANS: A • EXTINGUISH
Chose the lettered word or phrase that is • Ignite
most nearly opposite in meaning to the • Flourish
• Anonymous
words in capital letters
• Anatomy
• INSIGNIFICANT ANS: A
• Radical Chose the lettered word or phrase that is
• Unnoticed most nearly opposite in meaning to the
• Salient words in capital letters
• Awesome
ANS:C • DEMAN
Chose the lettered word or phrase that is • Elasticity
most nearly opposite in meaning to the • Market price
• Monopoly
words in capital letters
• Supply
• ACTIVE ANS: D

151
Chose the lettered word or phrase that is • GARRULOUS
most nearly opposite in meaning to the • Quiet
words in capital letters • Stranger
• Wordy
• SIMILAR • Frozen
• Alike ANS: C
• Different Select the word or phrase closest in meaning
• Confuse to the given word.
• Organise
ANS: B • UNIQUE
Chose the lettered word or phrase that is • Unequaled
most nearly opposite in meaning to the • Special
• Inferior
words in capital letters
• Outdated
• DISMISS ANS: A
• Disorganise Select the word or phrase closest in meaning
• Carriage to the given word.
• Absorb
• Calore • POVERTY
ANS: C • Wealth
Chose the lettered word or phrase that is • Penury
• Abundant
most nearly opposite in meaning to the
• Inadequate
words in capital letters ANS: B
Select the word or phrase closest in meaning
• LIQUID
• Absorbent to the given word.
• Dissolve
• INCULATE
• Solid
• Dentist
• Evaporate
• Teach
ANS: C
• Corroborate
Select the word or phrase closest in meaning
• Acquire
to the given word. ANS: B
Select the word or phrase closest in meaning
• CELIBATE
• Married to the given word.
• Widowed
• FAVOURITISM
• Divorced
• Nepotism
• Single
• Hooliganism
ANS: D
• Capitalism
Select the word or phrase closest in meaning
• Socialism
to the given word. ANS: A
Select the word or phrase closest in meaning
• HOMOGENEOUS
• Similar to the given word.
• Scrambled
• SUCCULENT
• Different
• Notorious
• Selective
• Juicy
ANS: A
• Sour
Select the word or phrase closest in meaning
to the given word.
152
• Angry • REGIME
ANS: B • Office
Select the word or phrase closest in meaning • Uniform
to the given word. • Carnivore
• Rule
• PROPER ANS: D
• Carelessness Select the word or phrase closest in meaning
• Inefficient to the given word.
• Decorous
• Catastrophe • HOMILY
ANS: C • Sermon
Select the word or phrase closest in meaning • Suburb
to the given word. • Pension
• Cereal
• COWARD ANS: A
• Jaundice Select the word or phrase closest in meaning
• Melancholic to the given word.
• Fearful
• Brave • ERUDITE
ANS: D • Slave
Select the word or phrase closest in meaning • Scholarship
to the given word. • Worm
• Schooling
• THWARTING ANS: B
• Frustration Select the word or phrase closest in meaning
• Experience to the given word.
• Deviant
• Prosperity • VILLAGE
ANS: A • Skyscapper
Select the word or phrase closest in meaning • Operation
to the given word. • Hamlet
• Catholism
• INDEFINITE ANS: C
• Vague Select the word or phrase closest in meaning
• Verbal to the given word.
• Infinite
• Strange • CANTATA
ANS: C • Music
Select the word or phrase closest in meaning • Lecture
to the given word. • Administration
• Choral work
• HARDSHIP ANS: D
• Agency Select the word or phrase closest in meaning
• Adversity to the given word.
• Lavish
• Opponent • LUKEWARM
ANS: B • Ensnared
Select the word or phrase closest in meaning • Tepid
to the given word. • Equity

153
• Embarrassed • Animal
ANS: B • Rock
Select the word or phrase closest in meaning • Origin
to the given word. • Causes
ANS: D
• ACUMEN Fill in the gaps with the appropriate word or
• Kindness phrase.
• Brilliance • …………………. is a place where
• Audacity weapons are stored or made
• Keenness • Arsenal
ANS: D • Manchester Unit
Select the word or phrase closest in meaning • Redanoel
to the given word. • Super Eagle
ANS: C
• PEDAGOGUE Fill in the gaps with the appropriate word or
• Teacher phrase.
• Pupil • A physician trained in female affair is
• Scholar a …………………
• Peddler • Psychiatrist
ANS: A • Gyneacologist
Select the word or phrase closest in meaning • Psychologist
to the given word. • Optician
ANS: B
• EXPERIMENT Fill in the gaps with the appropriate word or
• Odd phrase.
• Newness • An individual trained in sporting with
• Practical bicycle is called a …………
• Accept • Motocyclist
ANS: C • Tricyclist
Fill in the gaps with the appropriate word • Bicyclist
or phrase. • Cyclist
ANS: D
• A person who buys and sell goods and Fill in the gaps with the appropriate word or
services is called a ………………………. phrase.
• Church goer • X-rays are taken care off in the
• Victim department of…….
• Trader • Radiology
• Traditionalist • Geology
ANS: C • Cartography
Fill in the gaps with the appropriate word or • Geography
phrase. ANS: A
• Cars are packed at……………… Fill in the gaps with the appropriate word or
• Car pack phrase.
• Garage • In Nigeria, GRA is an abbreviation of
• Bookshop …………..
• Bedroom • Government Rejected Area
ANS: B • Guest Room Area
Fill in the gaps with the appropriate word or • Government Reserved Area
phrase. • All of the above
• Aetiology is the study of ……… ANS: C

154
Fill in the gaps with the appropriate word or • 20 years
phrase. • 35 years
• Private Institutions are owned by ANS: A
• Proprietors Fill in the gaps with the appropriate word or
• Principals phrase.
• Vice Chancellors • Fashion designer is a refined name
• Academicians for…..
ANS: A • Sailor
Fill in the gaps with the appropriate word or • Tailor
phrase. • Councillor
• ………… is a compulsory subject in • Architecture
UTME ANS: B
• Geography Fill in the gaps with the appropriate word or
• Mathematics phrase.
• English • Carpentry tools are kept in……….
• Biology • Cupboard
ANS: C • Locker
Fill in the gaps with the appropriate word or • Tool box
phrase. • Shelf
• ………….. is an example of a ANS: C
carnivorous animal Fill in the gaps with the appropriate word or
• Goat phrase.
• Millipede • Shoes are made mostly from….
• Elephant • Leather
• Lion • Rubber
ANS: D • Upholstery
Fill in the gaps with the appropriate word or • Skin
phrase. ANS: A
• …………….. is a name of any person, Fill in the gaps with the appropriate word or
animal, place or thing phrase.
• An Adverb • Beans is a pea, while rice is a ..
• A Noun • Fruit
• A Pronoun • Perishable
• Preposition • Grain
ANS: B • Food
Fill in the gaps with the appropriate word or ANS: C
phrase. Fill in the gaps with the appropriate word or
• A counselor is to counsel clients while phrase.
teacher is to teach………. • Students are kept in the dormitories
• Students while animal are kept in ………..
• Lecturers • Hostels
• Agriculturists • Auditorium
• Learners • Zoo
ANS: A • Aquarium
Fill in the gaps with the appropriate word or ANS: C
phrase. Fill in the gaps with the appropriate word or
• To be a bonafide voter in Nigeria, the phrase.
attainable age is… • A female lion is called a …
• 18 years • Pigeon
• 19 years • Lioness

155
• Lilies • Ecology
• Centipede • Psychology
ANS: B • Sociology
Fill in the gaps with the appropriate word or • Sexology
phrase. ANS: A
• A Vice Chancellor is an Administrator Fill in the gaps with the appropriate word or
in charge of … phrase.
• School of Medicine • Cartographers are experts in…….
• University • Landscaping
• School of Preliminary Studies • Mapprage
• Polytechnic • Road construction
ANS: B • Map drawing
Fill in the gaps with the appropriate word or ANS: D
phrase. Fill in the gaps with the appropriate word or
• …………… is an example of a phrase.
communicable disease • Lunatics are those who are………..
• Fibroid disordered
• Electrolysis • Sociologically
• Malaria • Mentally
• Cough • Empirically
ANS: D • Geographically
Fill in the gaps with the appropriate word or ANS: B
phrase. Fill in the gaps with the appropriate word or
• Plantain is a good source of phrase.
• Water • College of Education is headed by a
• Vegetable ………….
• Iron • Administrator
• Carbohydrate • Chancellor
ANS: C • Provost
Fill in the gaps with the appropriate word or • Council Member
phrase. ANS: C
• Perishable foods can be preserved Which of the following words can be formed
through …. from the words in capital.
• Grilling
• Sun drying • ECONOMICS
• Roasting • Economical
• Harvesting • Come
ANS: B • Cooks
Fill in the gaps with the appropriate word or • Song
phrase. ANS: B
• Orange is a good source of Which of the following words can be formed
• Vitamin A from the words in capital.
• Vitamin B
• Vitamin C • DICTATION
• Vitamin D • Icon
ANS: C • Tate
Fill in the gaps with the appropriate word or • Ione
phrase. • Data
• The study of relationship of living ANS: A
thing to their environment is…………..

156
Which of the following words can be formed • Shadow b. Light c. Image
from the words in capital. d. darkness
ANS C
• ENVANGELISM
• manager • An insect that attacks and destroys
• Glid
crops is called ____________
• Slim
• Ever
ANS: C
Which of the following words can be formed
• Soldier b. Parasite c. Pest
from the words in capital. d. pesticide
ANS C
• MADELEINE
• The thermometer is used to
• Deed measure ________________
• Date
• Elder • Distance b. Water c.
• Line Temperature d. Pressure
ANS: D ANS C
Which of the following words can be formed
from the words in capital. • Chocolate is made from
________________
• FINGERPRINTING
• Finger • Coffee b. Cocoa c.
• Printings
• Fern
Sugarcane d. Sugar
• Granite ANS B
ANS: C
• Which of these is not a sense organ

• Nose b. Ear c. Jaw d.


• Share 60% of #20,000 equally Tongue
between three people. How much ANS C
does each person get?
• How many 250g packets of sweets
• #12,000 b. #10,000 c. can be made from a 2Kg bag of
#8,000 d. #4,000 sweet?
ANS D
• 12 b. 32.5 c. 6 d. 8
• The period in which hen lies on its
egg until it hatches is known as ANS D
________
• Captain is to ship as ___________
• Hibernation b. Resting c. is to aeroplane.
Incubation d. Breeding
• Planner b. Manager c. Pilot
ANS C
d. Oceanic
• A reflection in a mirror is called ANS C
___________
157
• The full meaning of AIDS is • One shirt can dry in the sun for 30
____________________ min. How long will it take for ten
shirts to dry if they were put in the
• All Is Disease Starting b. sun at the same time?
Acquired Immune Disease
Symptoms c. Acquired • 2 ½ h b. 5h. c. 3h d.
Immuno Deficiency Syndrome ½h
d. Acquired Immuno ANS D
Deficiency Symptoms
• Plants use __________ to make
food. A
N
• Glucose b. starch
S c. Oxygen
d. Carbon dioxide
ANS DC

• The official language in a country


• The young of a goat is is called____________
called_____________ • Ethnicity b. English c.
• Son b. Kid c. calf d. Baby Swahili d. Lingua Franca
ANS D
ANS B • Which of these is the odd one
• The young of a cow is • Plate b. Spoon c. Pot d.
called____________ Fork
• Ewe b. Kitten c. Calf d. ANS C
Kid • A university is often referred to as
ANS C _______________
• The full meaning of GSM is • Primary Institution b.
_____________________ Secondary Institution c.
• Got Sent Messages b. Global Tertiary Institution
Service Mobile c. General d. Quaternary Institution
System manager d. Global
Satellite Mobile ANS C
ANS
D • If 2/5 of the total number of pupils
in a class is 40. What is the total
• Express 0.9 as a percentage. number of pupils in the class?
• 9% b. 30% c. 50% d. • 20 b. 80 c. 60 d. 100
90%
ANS D ANS D
158
• Add the lowest and the highest of • ₦19.60K b. ₦19.00K c.
these numbers and find the ₦190.00K d. ₦196.00
average. ANS D

20, 48, 0, 100 and 10 • Complete the sentence.


_____________ Ade and Tope are
• 50 b. 100 c. 55 d. 18 students.

ANS A • Neither b. Both c. All d.


None
• Five years ago, Ade was 8yrs old. ANS B
How old will Ade be in ten years
time? • Complete the following sentence.
________ Lola nor Laide got the
• 18 b. 13 c. 23 d. 28 question right.

ANS C • Both b. Neither c. All d.


Either
• Ayoola divided 50 naira among his
ANS B
three friends in the ratio of 1:2:2.
How much will each take? • If APPETIZER correspond to
122361738, then PRIZE will
• 10:20:20 b. 50:100:100 c. correspond to _________
5:25:25 d. 5:20:20 e.
5:20:25 ANS A • 12345 b. 28173 c. 23617
d. 28317 ANS
• Shade is 8 years older than Lola B
who is 4 years younger than Tunde.
If Tunde is 7years old, how old is • Change to improper fraction.
Shade?
• 49/7 b. 50/7 c. 51/7 d.
• 10 b. 11 c. 12 d. 13 e. 52/7
Cannot be determined ANS D
ANS B
• If y . What is ?
• The telephone is a means of
_____________ • 9 b. 19 c. 21 d. 18

• Singing b. Communication ANS B


c. Photography d. Reading
ANS B • Solve 286/6 to two decimal places.

• A trader bought seven pots at • 47.667 b. 47.67 c. 48 66 d.


₦100.00 each and sold each at 48.77 e. 28.46
₦72.00. How much money has he ANS B
lost all together? • What is the place value of 5 in
25904?
159
• 5 b. 50 c. 500 d. 5000 • Your father’s sister is
your______________ .
ANS D
• Grandmother b. Uncle c. Aunt
• Gaseous water is called? d. Sister
ANS C
• Ice b. Rock c. Vapour
d. Cold • Complete the sentence. As soon as
ANS C he entered the room he
___________ off his shirt.
• Pupil is to school as ____________
is to hospital. • Take b. Takes c. Took d.
taken
• Sickness b. Doctor c. Nurse ANS C
d. Patient
ANS D • Ngozi is the ______________ of
the three sisters.
• __________ indicates that the
speed at which a car is moving. • Taller b. Tallest c. Most tall
d. More taller ANS
• Fuel gauge b. speed gauge c. B
speedometer d. moving gauge.
ANS C • A trader brought a radio for #36
and sold it for #45. Express the
• A veterinarian takes care of profit as a percentage of the cost.
___________ .
• 25% b. 30% c. 35% d. 40%
• Vegetables b. Animals c. Food
d. Water ANS ANS A
B
• A man who earns #480 a month
• A person who studied the weather has to pay 25K as tax on every
condition of a place and predicts naira. How much does he pay as
possible changes in the weather is tax?
called a_______________ .
• #480 b. #120 c. #100 d. #75

• Weather man b. Geologist c. rain ANS B


man d. Meteorologist ANS
D • There are 600 pupils in a school. If
33 % are girls, how many boys are
• Which of these is your relation in there in the school?
the extended family?
• 198 b. 300 c. 402 d. 450
• Mother b. father c. uncle d.
brother ANS ANS C
C
160
• A woman bought some oranges • 234 is to 342 as 987 is to ________
and 30% of them were bad. If there .
were 60 bad oranges, how many
oranges did she buy? • 789 b. 798 c. 879 d. 897

• 18 b. 42 c. 100 d. 200 ANS C

ANS D • Amaka scored 60% in a test that


had 50 questions. How many
• Express 0.085 as a percentage. questions did she get correct?

• 85% b. 850% c. 8.5% d. 0.85% • 25 b. 30 c. 35 d. 40

ANS C ANS B

• A man’s step is 80cm long. How • In what ratio will 56 oranges be


many steps should he take to walk shared between Femi and Dayo
a distance of 2km? such that Femi gets 16 and Dayo
gets 40?
• 2050 b. 1250 c. 25000 d. 2500
• 1:2 b. 1:5 c. 2:5 d. 3:5
ANS D
ANS C

• A farmer has 120 birds. If he has


• Ali and Adam shared #840 in the increase of 20% how many birds
ratio of 3:4. How much did Ali has he?
get?
a. 124 b. 144 c. 164
• #420 b. #360 c. #480 d. #300 d. 200 ANS
B
ANS B
• The cost of electricity is 15K per
• By how much is 75% more than unit. How many units did a man
½? consume if he paid #42.55K
• 15% b. 20% c. 25% d. 35% e. • 283.67 b. 2.837 c. 482.67 d.
40% ANS 48.27
C
ANS A
• What is the product of 108 and 112 • A lecture lasts for 142 minutes. If it
is divided by 24? begins at 9.45am., at what time
• 504 b. 555 c. 220 d. 55 does it end?

• 12.30pm b. 12.07pm c.
ANS A 2.15pm d. 1.45pm
ANS B
161
• Add together the odd numbers • Boils b. Melts c. Solidifies
between 20 and 32. d. Evaporates
ANS C

• 168 b. 156 c. 128 d. 160 • Which of these is not a fuel?

ANS B • Petrol b. Diesel c. Coal d.


Wood
• I have two boxes of sweets with 56 ANS D
in each of them. To how many
children can I give 4 sweets each • Dog, cat, goat and _____________
and have 16 left. are examples of mammals.

• 5 b. 10 c. 20 d. 24 • Snake b. Lizard c. chicken


d. Whale ANS
ANS D D

• Ayoola is the ___________ • The International Airport in Lagos


is called ____________________ .
brilliant of the three boys.

• More b. Most c. Very d. both • JF Kenedy Airport b. Nnamdi


e. first ANS Azikiwe International Airport
B c. Murtala Mohammed
International Airport d.
• Three numbers are multiplied Nigeria Airport Authority
together to get 270. If 5 and 6 are ANS C
two of them, what is the third
number? • Which of these does not belong to
the group?
• 7 b. 8 c. 9 d. 4
• Lamb b. kid c. kitten d.
ANS C goat
ANS D
• Animals have camouflage so that
they cannot easily be seen. Which • Animals that live inside water are
of these animals have camouflage? referred to as ______________
animals.
• Zebra b. Peacock c.
• Living b. Oceanic c.
Chameleon d. Hyena
ANS C Aquatic d. Fishy
ANS C
• When wet clothes are spread in the
sun, they become dry because the • Bread is ________ in the oven
water _______________ . • Made b. Cooked c. Dough
d. Baked
ANS D
162
• Bello’s recent success got into his • Which of these diseases is
head. This means it made him contagious?
____________ .
• Head ache b. malaria c.
• Humble b. More careful c. Cough d. Stomach ache
Proud d. Happy ANS C
ANS C
• Which of the following is a
• Nigeria is surrounded by member of the nuclear family?
_____________ speaking
countries. • Grandmother b. Aunt c.
Brother d. Nephew
• English b. French c. ANS C
German d. Swahili
ANS B • The main implement used in
mechanized farming is
• The plate _________________ by ___________.
Ayo.
• Cutlass b. Bull dozer c.
• Was breaking b. was tractor d. Hoe 0
broken c. is broken d. break ANS C
ANS B
• A place where people keep their
• Lizard, crocodile and snake are money and Valuables is called
examples of _______________ . ______.

• Mammals b. Herbivores c. • Save b. Security house c.


Amphibians d. Reptiles bank d. Treasury
ANS D ANS C

• Malaria is a disease transmitted by • Which of the following is not a


_____________. voluntary organization?

• Water b. Insect c. Air d. • Boys scout b. Girls Guild


food c. Red Cross d. Road safety
ANS B ANS D

• For proper ventilation, a house • Butter and soap can be made from
must have _____________ . ___________.

• Toilets and bathrooms b. • Cocoa b. Cotton c. Palm oil


Furniture and carpet c. d. Crude oil
Windows and doors ANS C
d. Kitchen and store
• Which of the instrument is used to
ANS C measure rainfall.

163
• Rain meter b. rain gauge c.
rain level d. water rate ANS B
ANS B
• Tuned and Ade shared a certain
• A ruler is 30cm long. How many sum of money. Tuned got #48
times can it be used to measure a which was three times Ade’s share.
distance of 4.2m? What was the amount shared?

• 10 b. 14 c. 140 d. 12 • #62 b. #64 c. #144


ANS d. #96
C ANS B

• The excretory system in the body • Divide 22 by half and deduct 4.


does the work of ___________ . What do you have?
• 20 b. 30 c. 40 d. 7
• Carrying blood b. Removing
food waste c. Digesting ANS C
food d. Breathing • Which of these does not belong?

• Teacher b. Doctor c.
ANS B student d. lawyer
ANS C
• Electricity does not flow through
rubber because it is • The rainbow has __________
____________. colours.

• An insulator b. A conductor • 5 b. 6 c. 7 d.8


c. A condenser d. A circuit
ANS A ANS C

• If your father has two wives, the • Which of the professional travels
daughter of the other wife is your into space.
______.
• Pilots b. Divers c.
• Sister b. Half sister c. Astronaut d. Space wagon
Cousin d. Step sister ANS C
ANS B
• The period used by the fetus in the
• Great misfortune or disaster like womb is called __________
thunderstorm which are not caused period.
by man is called ___________.
• Hibernation b. Gestation
• Death problem b. Natural c. Acclimatization d.
disaster c. Industrial disaster Growing ANS B
d. artificial disaster
• Which of these is a good source of
vitamin c.
164
• Orange b. Beans c. Meat
d. Butter ANS ANS B
A
• Common salt is chemically called
• Which of these is an example of _____________.
body building food.
• Table Salt b. Sodium
• Yam b. Egg c. Coke chloride c. food salt d.
d. Butter Sodium Carbonate ANS B
ANS B
• The bony framework of an animal
• The process of making beer is is its ____________.
called___________.
• Shape b. Skeleton c. Image
• Brewing b. Browning d. Archeologist
c. Bearing d. Boring ANS B
ANS A
• Which of these is not a type of
• Which of these does not belong to religion.
the group?
• Buddhism b. Christianity
• Dog b. Goat c. Rat d. c. Cannibalism d. Islam
Hen ANS C
ANS D
• Which of these describe the way of
• Which of these diseases is caused life of a group of people.
by tse - tse fly.
• Believe b. religion c.
• Cholera b. Dysentery c. Culture d. Food
Sleeping sickness d. Malaria ANS C
ANS C
• You can catch a glimpse of the
• To be able to sing the anthem I queen means __________.
must know the __________.
• You can follow the queen
• Vocals b. Choir c. Lyrics b. You can get a brief look at
d. Title ANS the queen
C
• The glimpse belong to the
• Iron can be prevented from rusting queen d. Queen can be seen
by __________. always ANS B

• Adding water b. Painting • The examination takes place


Exposure to air c. Adding quarterly means the examination
water and air takes place

• Once in four months b.


Once in three months c. Only
165
in four places d. Only in three • A pedestrian travel on
places ___________.

ANS B • Water b. Bicycle c. Foot


d. Train
• A distant object can appear closer ANS C
if a__________ is used.
• Which of these options can be used
• Microscope b. Telescope to control plant growth around the
c. Stethoscope d. house.
Microphone ANS B
• Genocide b. herbicide c.
• Which of these options is the act of Insecticide d. Kerosene
killing oneself deliberately. ANS B

• Poisoning b. Suicide c. • I have 25 oranges. I gave 10 to Ade


Homicide e. Herbicide and 5 to Bola. What fraction do I
ANS B have remaining?

• A female sheep is called • 3/5 b. 2/5 c. 4/5 d.


1/5 ANS
• Kid b. Fingerling c. Ewe B
d. Ram
ANS C • Increase #3400 by 13%.

• The patient was taken to the • #442 b. #3842 c.


_________ by the doctor for the #3413 d. #3952
surgery. ANS B

• Labour room Laboratory c. • How many times is 0.04 contained


Theatre d. Ward in 44
ANS C
• 1100 b. 1.76 c. 11 d.
• How many months of the year have 110
less than 31 days? ANS A

• 1 b. 3 c. 5 d. 7
• Which of these seasons of the year
ANS C is not present in Nigeria?

• The sun or any star and the planets • Harmattan b. Dry c.


which move round it is referred to Raining d. Winter
as ____. ANS D

• Sunny system b. Solar • Scientists perform Experiments in


System c. Solar Plexus d. the ________.
Solar power ANS B
166
• Library b. Lavatory c. • #25 b. #5.00 c. #50.00
Laboratory d. Lobby d. #53.00
ANS C ANS C

• We keep food in the • Shade went out with some money.


____________ to prevent spoilage. She spent 5/10 of it and had #2.40
remaining. How much did she have
• Shelve b. cupboard c. before going out?
Fridge d. Store
ANS C • #4.80 b. #2.60 c. #6.00
d. #15.60
• Which of these is a domestic ANS A
animal.
• Add together the nine smallest
• Cat b. snake c. Lion d. numbers from 1 to 19.
Giraffe ANS
A • 30 b. 55 c. 42 d. 45

ANS D
• The marked price of a book is • How many seedlings must be
#180.00. What will be the cash planted to obtain 60 trees if only
price if a discount of 6% is 20% of the seedlings survive?
allowed?
• 150 b. 125 c. 300 d. 250
• #210.80 b. #30.60 c. ANS
#169.20 d. #190.80 C
ANS C
129.At what time of the day is the sun
• Reduce 216/324 to its lowest
at the zenith?
terms.
A 8am
• 1/9 b. 2/3 c. ¾ d. 4/5 B 10am
C 4pm
ANS B D 12 Noon
E 2pm
• A ship sails 400km in 18hours.
How many days will it take to
ANS D
cover 1600km at the same rate?
130.Supply of electricity in Nigeria is
• 3 days b. 4 days c. 6 days the responsibility of
d. 8 days A National Electric Power Authority
ANS A B Power Holding Company of
Nigeria
• By selling an article for #47.00, a
C. National Atomic Power Agency
man lost 6%. How much did he
D National Energy Commission
pay for it?
167
E National Emergency Management 135.Which of the following
Agency ANS components of the blood is responsible
B for the transport of
131.Which of these is not a bird? oxygen?
A Kite A White blood cells
B Eagle B Plasma
C Vulture C Red blood cells
D Bat D Water
E Pigeon E Serum
ANS
ANS D C
136.Which of these enzymes is
responsible for the digestion of fat?
A Lipase
132.Which of the following is an B Pepsin
amphibian? C Amylase
A Whale D Cellulase
B Toad E Xylanase
C Lizard ANS
D Pigeon A
E Tilapia 137.How many sides in a pentagon?
ANS A 8
B B 6
133.Which of the following is a C 7
reptile? D 5
A Whale E 9
B Toad ANS
C Lizard D
D Pigeon
E Tilapia 138.In which geo-political zone of
ANS Nigeria is Plateau state located?
C A North east
134.Which of the following is a B North west
mammal? C North central
A Whale D South south
B Toad E East
C Lizard ANS C
D Pigeon 139. A cardiologist deals with ailments
E Tilapia related to the
ANS A Eye
A B Ear
168
C Lung E Mite
D Heart ANS C
ANS D 145. A disease that destroys the body’s
immune system is
A Malaria
140. The process of breathing is referred B Sleeping sickness
to as C HIV Aids
A Reproduction D Diabetes
B Respiration E Flu
C Oxidation ANS C
D Perspiration 146. A disease that is associated with high
E Digestion blood sugar
ANS B A Malaria
141. Insects have how many legs? B Sleeping sickness
A8 C HIV Aids
B2 D Diabetes
C6 E High Blood pressure
D 10 ANS D
E 12 147. Which of the following rivers is not
ANS C found in Nigeria?
142.Which of the following is not an A River Kaduna
insect? B Cross River
A Dragon fly C River Niger
B Spider D River Nile
C Termite E River Benue
D Housefly ANS D
E Cockroach 148. Which of the following is correct?
ANS B A Acomodation
143. Antibiotic is used to kill B Accomodation
A Virus C Accommodation
B Yeast D Acommodation
C Fungi E Acomoddation
D Bacteria ANS C
E Protozoa 149.What is your percentage profit margin
ANS D if you bought an item for N600 and sold it
144. The vector for the transmission of for N750?
sleeping sickness is A 600
A Housefly B 750
B Female Anopheles mosquito C 150
C Tsetse fly D 25
D Male Anopheles mosquito
169
E 100 D7
ANS D E 30
150. 50% of a kilometre is ANS A
A 500mm 155. What is the speed of a car that
B 500m travelled 840km in 7 hours?
C 500cm A 7km/hr
D 500ft B 120km/hr
E 500yd C 940km/hr
ANS B D 110km/hr
151. If a typist can type at the rate of 50 E 130km/hr
words/minute, How long will it take to ANS B
type a 7500 word document? 156. What type of vegetation are you
A 3Hrs likely to find at the northernmost parts of
B 4Hrs Nigeria?
C 21/2Hrs A Guinea savannah
D 1Hr B Sudan savannah
ANS C C Mangrove forest
152. Which of these vitamins is obtained D Rain forest
from sunlight? E Sahel savannah
A Vitamin C ANS E
B Vitamin A 157. Which of the following is not a tree
C Vitamin D crop?
D Vitamin K A Yam
E Vitamin E B Cashew
ANS C C Cocoa
153. If N1200 is divided between boys A, D Coffee
B and C at a ratio of 5:2:1 respectively, ANS A
what is the share of boy B? 158. Which of these animals does not have
A N600 horns?
B N750 A Goat
C N300 B Cattle
D N150 C Buffalo
E N400 D Horse
ANS C E Ram
154. Dotun, Ayo, Stephen and Olu were ANS D
given 10, 18, 24 and 8 oranges 159.Which country boarders Nigeria in the
respectively. What percentage of the total west?
oranges was given to Stephen? A Ghana
A 40 B Republic of Benin
B 18 C Togo
C 25 D Cameroun
170
E Chad (a) forty – four (b) twenty (c)
ANS B fifty (d) fifty – four
160.A tripod has how many supports?
A2
B4
C3
D5
E 6
ANS C
161. Tyres are made from
165. Some oranges were shared between
A Pulp
two children in ratio three to five respectively.
B Steel
If the first child’s share was 120, how many
C Cotton
oranges were shared between them?
D Rubber
E Sizal (a) three hundred and twenty (b) two
ANS D hundred (c) four hundred and twenty (d)
two hundred and twenty

162.The position of planet earth from the


sun in the solar system is
A 3rd
B 2nd
C 1st
166. Increase six hundred by twenty five
D 4th
percent
E 5th
ANS A (a) one hundred and fifty (b) seven
163. The entity called Nigeria today was hundred and fifty (c) one thousand
created in (d) six thousand
A 1960
B 1963
C 1956
D 1914
E 1956
ANS D
164. I owe a sum of two hundred and
twenty naira and was able to pay one
167. A debt of four hundred and fifty naira
hundred and ten naira. What percentage have
is to be paid in monthly installments of sixty
I not paid?
naira after the first payment of ninety naira,

171
how many months is required to complete
the payment?

(a) nine (b) five (c) twenty


four (d) six

A
n
170. Ins some countries drug trafficking is
punished by the offender being
=
(a) hanged (b) hung (c) shaken
(d)Dshook

168. In Anwar ul Islam girls School Lagos


there are five hundred pupils, if there are one
thousand five hundred pupils in St. Anthony
Secondary School Ilorin. What is the ratio of
the pupils in the Ilorin school to that of the
Lagos school?

(a) one to two (b) three to one


171. In a class of thirty two boys, sixteen
(c) three to two (d) five to
passed an examination and four were absent
four
while the rest students failed. What
percentage of the Awhole class failed?
n
(a) 37.5 (b) 50 (c) 42.86
s
(d) 57.14

169. What is a fringe benefit?

(a) a benefit derived from a job in addition to


remuneration

(b) the edge a brilliant person has 172. Ada has a certain number of oranges,
Tolu promised Ada five times the oranges he
(c) an advantage that looks small but which is has and Ada’s Mother promised to add
in-fact big thirteen more oranges for Ada. The total
oranges of Ada is forty eight. How many
(d) a benefit that will make u smile
oranges has Ada originally.

172
(a) sixty – five (b)eight
(c)thirty – five (d) seven

A
n
s

=
175. Modinah, Kafayah and Abdullahi
shared a number
D of apples. Modinah had two
- third while Abdullahi had one – fifth. What
fraction of the apples did Kafayah get?

(a) 2/15 (b) 3/15 (c)20


(d) 1/3

173. How long will a man take to cover a


distance of seven kilometers by trekking four
kilometers per hour?

(a) One hour thirty – five minutes

(b) one hour forty – five minutes

(c) Les than one hour


176. A quantity of food would last a family
(d) more than two hours of five for twenty – one days. How long would
the same quantity last a family of seven if all
of them eat at theAsame rate?
n
(a) twelvesdays (b) 2o days
(c) 15days (d) 29.4days
=

174. A plane takes off from Ilorin at


10.30am. It flies through Lagos to Accra in
sixty – five minutes. The plane makes a stop
of 40 minutes at Accra airport and then
proceeds back to Ilorin in seventy – minutes.
177. If 125 men can do a piece of work in
At what time does the plane land again back
120 days, how many men would be sufficient
at Ilorin airport?
to do the same work in 100days?
(a) 1.25pm (b) 12.55pm
(a) 180 men (b) 150 men
(c) 1.35pm (d) 12.05pm
(c) 96 men (d) 115 men

173
working at the same A rate, how long will the
n
work take them?
s
(a) 2 days (b) 16 days
(c)=4 days (d) 12 days

178. When Caleb was 15years old his


brother Clement was 18years, if Caleb was
born in year 1900, in what year was Clement
born?

(a) 1903 (b) 1915


(c) 1897 (d) 1867
181. Mr. Gorila bought
A a Bens Jeep last month
and agreedn to pay in two installments, In June
he paid three
s hundred and thirty naira, which
amount to two – third of the cost price. He is
to pay the rest at the end of August. How
=
much is he intending to pay as the balance?

(a) N110 C (b) N495


179. Patience, Abdul Rahman and Tola (c) N165 (d) N220
received three hundred and ninety - six naira
to share in the ratio of five to four and three
respectively, how much did patience receive?

(a) N132 (b) N198


(c) 99 (d) 165

A
n
182. A trader pays a debt in 6 weeks
s
installments as follows: the first week he paid
N4; the second twice this and the third, three
=
times the first and so on. How much was the
owed debt?
D
(a) N84 (b) N54
(c) N86
(d) N56
180. A contractor employed sixty men to
face a certain work and get it completed in
fifteen days. If the number of labourers is to
be increased to two hundred and twenty five,

174
(a) 750 students (b) 6,750 students
(c)=675 students (d) 7, 000
students
A

183. Two sums of money are in ratio two


to ten, if the smallest is fifteen kobo, what is
the largest?

(a) one naira fifty kobo (b) thirty five


kobo (c) seventy five kobo (d) one
hundred and fifty naira
186. If a lecture room in a given University
A long and twenty - two
is sixty - four metres
n If the University is planning to
metres wide.
s square metres to each student,
allocate eight
how many students will the lecture room
=
accommodate?

(a) 176 students


C (b) 1760 students
(c) 1408 students (d) 148
184. If twenty five percent of candidate for students
post UTME are girls, what fraction of the
candidate are boys?

(a) one - quarter (b) two - third


(c) half (d) three –
quarter

A
n
187. What s yearly insurance fee is paid for
an office valued at four million five hundred
=
thousand naira if the fee is put at one – eighth
of its value?
D
(a) N50062.50 (b) N506.25
(c) N562.50 (d)
185. If the total number of students in a N50006.25
University is seven thousand five hundred and
a class of students that have the cumulative
grade point average of four and above are
tagged scholar. If the fraction of students in
this scholar category is one tenth, how many
students within this University are not
scholars?
175
• 2006 b) 2007 c)
A 2008 d) 2009
188. In a courier outfit, telegram rate is as
Ans=D
follows: ordinary rate for first eighteen words 192.A dozen is 12, a score is ……………
cost three hundred naira each; additional • 20 b) 24 c) 48
words cost twenty naira each, however, d) 144
urgent rate is twice the ordinary rate. What Ans=A
will be the cost of an urgent telegram of 20 193.One of the following is not a musical
words? instrument
a)Guitar b) Flute
(a) N340 (b) N320
(c) N640 (d) c) Microphone d) Trumpet
N680 Ans=C
194.One of the following gases is used to
produce energy in A a living organisms.
n
• Carbon dioxide b) Nitrogen
s
c) Oxygen d) Chlorine
Ans=C
=
195. ……………. is used for the
measurement
D of atmospheric pressure
• Thermometer b)
189.. On a map, two metres represent Barometer c) Hydrometer d)
fifteen kilometres. What length on the map
Sphygmanometer Ans=B
would represent a distance of ninety
196.One of the following is used in
kilometers?
measuring wind speed
(a) 675 metres (b) 337.5 • Wind speed b)
metres (c)12 metres Barometer c) Thermometer d)
(d) 9 metres Anemometer Ans=D
197.Concerning the A Nigerian coat of arms,
the two white
n horses represent our
• s fertile land b) rivers Niger
and Benue c) dignity d)
= pride Ans=C
198.In the Nigerian coat of arms, the red
C
eagle connotes the
• strength and pride b)
190.One of the following is a traditional
dignity c) good land d)
leader in Calabar.
agriculture Ans=A
a)Eze b) Emir c)
199.As petroleum is to Nigeria, what is to
Obong d) Oba Ans=C
south Africa
191.Chief Gani Fawehinmi died in…………..

176
• Coal b) Copper c) 208.Which mountain is the tallest in the
Gold d) Zinc world
Ans=C • Everest b) Kilimajaro c)
Elbrus d) Blanc
200.A decagon has ………….. sides Ans=A
• Seven b) eight c) 209.Which of these towns is in south-west
nine d) ten geopolitical region of Nigeria
Ans=D • Ila-Orangun b) Owo
201.A rectangle box has ……………. and c) Egbe d) Osogbo
……………… equal surfaces Ans=C
• 0 and 6 b) 2 and 6 c) 2 210…………….. is a capital city in the north-
and 4 d) 4 and 4 west of Nigeria
Ans=C • Maiduguri b) Gusau
202.Isosceles triangle has ………….. equal c) Damaturu d) Lafia
sides Ans=B
• no b) two c) 211.Which year did Professor Wole
three d) four Soyinka get a noble prize.
Ans=B • 1985 b) 1986 c)
203.The total sum of angles in a triangle is 1987 d) 1988
………….. Ans=B
• 90 ͦ b) 180 ͦ c) 21. What is the full meaning of
270 ͦ d) 360 ͦ ATM?
Ans=B
A. Automatic teller machine
204.One of the following organisms is odd
• Lizard b) Snake c) B. Authentic teller machine
Alligator d) Toad
C. Argument teller machine
Ans=D
205.All of these animals lay eggs except D. Automated teller machine
• Bat b) Cat fish
c) Snake d) Frog ANS=D
Ans=A
206.Cairo is to Egypt as ……………….. is to
Senegal Instruction: Provide the
• Bangul b) Monrovia c) correct answer to each of the
Dakar d) Yaounde
following questions.
Ans=C
207.Lusaka is a capital city of …………… • The Apex Bank in Nigeria is
• Tunisia b) Uganda c) __________. (A) Zenith Bank
Zambia d) Zimbabwe
(B) First Bank (C)
Ans=C
Intercontinental Bank (D)
177
Central Bank. ANS
D

Instruction: Provide the


correct answer to each of the
following questions.

• The highest rank of an officer 1.Political process is _________


in the Nigerian Navy is (a) solving conflicts in public life
__________. (A) Commodore (b) winning an election (c) being
(B) Commander (C) General in opposition government (d)
(D) Admiral. ANS D mobilizing electorate for voting

Instruction: Provide the ANS A


correct answer to each of the
2.All over the world, people
following questions.
organize themselves into
• AU stands for ______. (A) societies known as
American Union (B) Africa
(a) clans (b) cleavages (c)
United (C) African Union (D) conglomerates (d]states
African Unionism. ANS C
ANS C
4._______ is the instrument
Instruction: Provide the through which the state puts its
correct answer to each of the plans into action. (a)
following questions. government (b law (c)
administration (d) policies ANS
• _________ is the Head of the A
British Commonwealth of
Nations. (A) President 5.The means through which the
Goodluck Jonathan (B) David will of the state is formulated,
expressed and realized is ______
Cameron (C) Queen Elizabeth
(a) law (b) administration (c)
II (D) Barack Obama.
government (d) communication

ANS C
ANS C

6.The sum total of the


legislative, executive and
judicial bodies of a state is
178
known as ________ (a) 12.Every independent country
machineries (b) logistics (c) in the world may be described
strategy (d) political system as ___________ (a) a state (b) a
republic (c) a sovereign (d) a
ANS D continent

7.The effort put in place to


ANS A
preserve individual liberty and
to avoid tyranny is backed up 13.Membership of a state is
by the principle of __________ (a) _________ for everybody born
fair-hearing (b) separation of within a state. (a) optional (b)
powers (c) due process (d) national (c) necessary (d)
justice ANS B compulsory ANS D

8.The control exerted on the 14.The political authority of the


arms of government is achieved state is _________(a) the party (b)
through ______ (a) cooperation government (c) electorate (d) the
and consensus (b) the mandate people ANS B
of the electorate (c) collaboration
and conciliation (d) checks and 16.The act of the state is indeed
balances ANS D an act of _________ (a) the people
(b) the party (c) the electorates
9.A politically organized body of (d) government (e) elected
people occupying a definite ANS D
geographical territory with an
organised government is _______ 17.All functions of the
(a) an aggregate (b) a state (c) a government are performed on
group (d) an ethnic group behalf of the _______ (a) elected
ANS A (b) state (c) electorates (d) party
(e) people ANS B
10.A definite and generally
recognised boundary of a 18.Government is only a part of
country is known as ________ (a) the ________ (a) people (b) party
a globe (b) a land (c) a territory (c) group (d) individuals (e) state
(d) an estate ANS C
ANS D
11.All residents in a country not
classified as citizens are 24.Politics is often defined as
regarded as _________ (a) aliens the study of ______ (a)
(b) refugees (c) immigrant (d) government (b) electoral
emigrants ANS A processes (c) voting pattern (d)
179
power (e) government and the 30.The monopoly of ______
governed makes government effective (a)
ANS wealth (b) force (c) resources (d)
A influence (e) control
ANS B
25.Power involves a kind of
_______ exercised by one party 31.Obedience must not always
over the other. (a) superiority (b) be based on the threat of force,
high-handedness (c) willingness it must be founded on _______
(d) domination (e) collaboration (a) consent (b) compulsion (c)
contribution (d) confidence (e)
ANS D conciliation
ANS A
26.A person has power when he
can _______ of others. (a) 32.The belief that disobedience
influence the conduct (b) speak to command will attract
wisely (c) take the life (d) dictate punishment is referred to as
the happiness (e) over-rule _______(a) physical power (b)
intension religious feelings (c) political
ANS A consent (d) coercive power (e)
collective power ANS D
27.The power that comes from
established authority is ______ 33.The influence that is based
(a) a brute force (b) political (c) on the control of means of
legitimate (d) social (e) economic production and distribution of
ANS B good and services is known as
________ (a) physical power (b)
28.The sources of power include distributive power (c) economic
the following control except power (d) coercive power (e)
________ (a) force (b) minority (c) resource control
wealth (d) skill (e) social status
ANS B ANS C
29.Physical restraint or coercion 34.Election confer _____ on a
through control over the ruler. (a) influence (b) wealth (c)
agencies of the state is known control (d) commitment (e)
as _______ (a) social status (b) power
skill (c) influence (d) force (e) ANS D
control
ANS D
180
35.The rightful and legitimate standards and procedure. (a)
power is known as _______ (a) accepted (b) respected (c)
influence (b) control (c) effective (d) legitimate
authority (d) force (e) liberty ANS
ANS C D

36._______ secured through the 45.Legitimacy may pertain the


threat of punishment is following, except ________ (a)
unstable. (a) power (b) control government institutions (b)
(c) commitment (d) beliefs (e) government functionaries (c)
obedience ANS B government by minority (d)
policy decisions (e)government
40.The following are sources of actions ANS
power except _______ (a) C
education (b) skill (c) prestige (d)
charisma (e) fear 46.Loss of legitimacy may result
ANS D from the following, except
_____(a) transparency (b)
41.When citizens oppose an corruption (c) arbitrary powers
exercise of power, it lacks ______ (d) injustice ANS A
(a)authority (b influence (c)
effects (d) clarity (e) enforcement 47.Legitimacy is the massive
ANS A support for a particular
government to _________ (a) rule
42.The belief in the legitimacy of without tenure (b) collect taxes
an authority that has always (c) award contracts (d) wield
existed produces ________ power
authority. (a) customary (b) local ANS D
(c) modern (d) traditions.
48.________ elections represent
ANS D one popularly accepted way of
deciding who should rule. (a)
43.Charismatic leader can show governorship (b) parliamentary
the following except _______ (a) (c) local government (d) open
passion (b) magical power (c) ballot
revelation (d) heroism ANS B
ANS A
49.A general uprising of an
44.For a government to be aggrieved electorate or the
_______, it must be established military leading to removal of a
according the rules, principles, government is called _____(a)
181
revolution (b) devolution (c) A B C
evolution (d) coup de tat D
ANS Other Another
D Each other One another ANS: B

50.__________ is fundamental to 26. What will a rolling stone never gather?


the maintenance of political
A B C
order in society. (a) democracy D
(b) legitimacy (c) accountability
ANS B Rose Mass Moss
Boss ANS: C
51________ is the most
27. What is the mother of invention?
important characteristics of a
state. (a) sovereignty (b) A B C
legitimacy (c) accountability (d) D
majority rule Difficulty Necessary
ANS Necessity Necessitate ANS: C
A
28. What should you make while the sun
shine?

A B C
52.The supreme and final legal D E
authority beyond which no
further legal power exists is Gay Say Day
Hay Ray ANS: D
________ (a) sovereignty (b)
legitimacy (c) independence 30. What should you do before you leap?
(d)participation
A B C
D

ANS Listen Hear Think


A Look ANS: D

24. I cannot ----------------- any noise ------------- 31. With what do we catch a thief?

A B C A B C
D D

Hare/here Ear/Hair Hear/here Gun Police


Hear/hear ANS: C Soldier Thief ANS: D

Answer with one word: 32. What should you let sleeping dogs do?

25. What does one good turn deserve?


182
A B C (b) Because we have plenty of
D milk.

Lay Sleep (c) Because it is the easiest


Lie slide food to prepare.
ANS: C
(d) Because no other food is
33. Who catches the worm? so nourishing to babies. ANS: D

A B 37. Meat is cooked before it is eaten because:


C D
(a) It tastes better that way.

Early bird Late Bird (b) It smells better when


Early rise Eagle cooked.
ANS: A (c) Housewives like cooking
34. What isn’t robbery? meat.

A B (d) Cooking kills’ germs.


C D ANS:D
E 38. Which tells you best what plenty is?
Economy Exchange (a) A great deal.
Banking Borrowing
Lending ANS: (b) Enough.
B
(c) More than enough.
35. We judge people by what they do rather
(d) All that you want.
than what they say. Why?
ANS: A
(a) Words are sometimes
39. The words telegraph, telephone,
hard to understand.
television, all contains- “tele” what does it
(b) Words often have more mean?
than one meaning.
(a) It tells us that these three
(c) What people do, tells us things are worked by electricity.
what they are really like.
(b) It tells us that these things
(d) What people do, is not are scientific.
always the right thing, ANS: C
(c) It tells us that these things
36. Why do we breast-feed? enable one to deal with things at a distance.

(a) Because it is the cheapest (d) It simply means “tell or


way of feeding babies. say”. ANS:C

183
40. If bread in code is csfbe, then cbe A B
means................. C D

A B C D
Broilers Layers
Chicks Producers
Bed Ear Bad Red
ANS: A
ANS: C
46. What is the name given to crops grown
41. If scraping in code is rbqzohmf, then fzor purposely for sale as raw materials?
means....................
A B
A B C D C D

Grin gaps gasp gain Tree crops Forest crops


ANS: B Food crops Cash crops
42. If ynncyjgle in code is appealing, then ANS: D
nyle means............... 47. Which of this is a system used to bring
A B C D water from streams to farm lands?

A B
Pane Pale Pang Peal C D
ANS: C

43. If amlypqcjw in code is coarsely, then Irrigation Ridging


alpq means............... Ploughing Manuring
ANS: A
A B C D
48. A diagram that shows the walls of a house
drawn to scale is known as the --- of the
Case Coal Clay Core
ANS: A house.

44. If ftgcokpi in code means dreaming, then A B


ogcp means................. C D

A B C D
Sketch Drawing
Manual Plan ANS:
Maid Made Mean Mend D
ANS: C
49. The metals can be beaten into any shape
45. Chicken which are raised purposely for because they are -----
flesh are called----

184
A B A. Botany B. Horticulture C.
C D Forestry D. Game reserve
ANS: B

Malleable Ductile 58. Which of the following can be produced


Brittle Soft from the livestock farm?
ANS: A
A. Eggs B. Flowers C. Meat D.
50. Which of these are used to hold together Sugar ANS:A
heavy machine parts?
59. What is used to measure angles in
A B technical drawing?
C D
A. Set-square B. T-square C.
Protractor D. Compass
Clamp Screws ANS: C
Bolt & knots Strings
ANS: C 62. What is the name given to air in motion?

52. In case of lack of lubricant, the engine A. Speed B. Cloud C.


may cease. This is called an engine Wind D. Gas ANS:
C
A. Break B. stop
63. Which instrument do scientists use to
C. Lock D. knock
ANS: D view the stars?

54. Why must fresh palm wine be left for A. Periscope B. Microscope C.
three days to one week before the alcohol is Telescope D. Kaleidoscope E.
Binoculars. ANS:C
made from it?

A. To make it ferment B. To make it rot 64. What is the term used for the type of
C. To make it smell movement in which an organism moves from
one place to another?
D. To make it more sweet
ANS: A A. Crawling B. Running C.
Locomotion D. Transition
56. During the purification of water at the ANS: C
water works, alum is added to it to.....
65. What is the name given to the yellow part
A. Improve its taste B. Kill bacteria of the egg?
C. Make it clear D. Cool it
ANS: C A. Yolk B. Albumen C. Shell D.
Abdomen ANS:
57. The planting and caring of ornamental A
plants to decorate the surrounding of a house
66. The period in which the hen lies on its
is known as......
eggs until it aches is known as....... period

185
A. Hibernation B. Incubation 73. What is the name given to the process by
C. Breeding D. Rousting which food substances are broken down in
ANS: B forms to be absorbed in to the body?

67. How long does the chicken lie on its eggs A. Respiration B. Digestion C.
before it aches? Reproduction D. Growth
ANS: B
A. 10 days B. 12 days C. 30
days D. 21 days 74. Escape of water from the leaf is known as
ANS:D -----

68. Which part of the egg of a chicken A. Carbon dioxide B. Iron


develops into the young ones? C. Osmosis D.
Transpiration ANS: D
A. Yolk B. Yellow matter C.
White matter D. Air space 75. One of the major problems faced by the
ANS: A men who travel into space is -----

69. Animals that give birth to their young A. Hunger B. Lack of water
ones alive are called------ C. Extreme cold D.
Lack of weight ANS: D
A. Birds B. Reptiles C.
Amphibians D. Mammals 76. A reflection in a mirror is called ------
ANS: D
A. Darkness B. Image C.
70. What term is given to the changes insects Shadow D. Colour
undergo in their life history? ANS: B

A. Recuperation B. Rousting C. 79. Which of the following is a respiratory


Metamorphosis D. None of the organ?
above ANS: C
A. Tongue B. Eye C.
71. What is the major product of the process Nose D. Lungs
of respiration in living things? ANS: C

A. Carbohydrate B. Protein 80. Which of this is given off as a by product


C. Energy D. Carbon during respiration?
dioxide ANS: D
A. Oxygen B. Carbon dioxide
72. What is the name of the white part of a C. Glucose D. Urea
boiled egg? ANS: B

A. Yolk B. Abdomen C. Albumen 81. Which of the following is not a stage in


D. Egg white ANS: the life history of the mosquitoe?
C
A. Imago B. Larva C. Pupa D.
None of the above
ANS: A

186
82. When the moon passes between the A. 5:7 B. 3:4 C1:3
earth and the sun and all the three D. 5:12 ANS:
are in a straight line we have.............. D

A. Eclipse of the earth B. 91. A lucky draw price of N150 was shared
Eclipse of the moon C. Eclipse of among three winners in the ratio
the sun D. Cateling of the sun 4:5:6. How much did each get?
ANS: C
A. 30, 40, 70 B. 40, 50, 60 C. 50,
83. What structure in the fish serves the 60, 40 D. 20, 50, 70 ANS:
same purpose as the lungs in human B
being?
492. Which of the following is a square
A. Scales B. Lateral line number?
C. Fins D. Gills
ANS: D A. 2 B. 10 C.
400 D. 600
86. Scientists who study the weather are ANS:C
called.........
93. n x n = 900. n is -------
A. Geographers B. Archaeologist
C. Astrologers D. A. 20 B. 30 C. 40
D. 50
Meteorologist ANS: D
ANS:B
87. Which of the following is a cereal?
94. What is the square root of 9/81
A. Maize B. Cocoa C.
Coffee D. Tea ANS:A A. 2 / 10 B. 3 / 9 C. 3 /
8 D. 2 / 9
88. An organism that lives on a living ANS:B
organism and eventually causes harm
to it is called...... 95. Simplify(√49) 2

A. Herbiviour B. Carnivour C. A. 7 B. 49 C. 6
Parasite D. Pest ANS: D. 8
C ANS: B

89. I have N12 and my brother has N24. what 96. What is the total charge for a distance of
is the ratio of my money to my 4km if a taxi charges 50k per
brothers? kilometer.

A. 3:4 B. 1:2 C. 2:4 A. N 5.00 B. N 2.00 C. N


9.00 D. N 4.00 ANS:
D. 1:3 ANS:B
B
90. In a school, 25 teachers out of 60 own
cars. What is the ratio of the teachers 97. The rent for a house is N 600 per month.
who own cars to those who do not? What is the total rent for a year?

187
A. N 2720 B. N 7200 C. N
1720 D. N 720
ANS: B

98. A clock losses 10mins every day. How


much slower will it be after 8 days?

A. 40mins B. 30mins
C. 1hr 20mins
D. 1hr 10mins
ANS:C

99. If the average rain fall per month is


12.2cm, what is the total amount of
rainfall for the whole year?

A. 140.2 B. 146.4 C.
164.42 D. 124.8
ANS:B

100. Round off 0.48 to 1 decimal place

A. 0.48 B. 0.4 C. 0.5


D. 0.50 `
ANS:C

101. Round off 1.56 to 1 decimal place

A. 1.6 B. 1.56 C.
0.56 D. 1.5
ANS:C

102. Simplify 7+y = 20. Find y

A. 18 B. 27 C. 13
D. 14
ANS:C

103. Simplify 10 + p = 30. Find p

A. 30 B. 10 C.20
D. 40
ANS: C

188

You might also like